Home

  • Recently Active
  • Top Discussions
  • Best Content

By Industry

  • Investment Banking
  • Private Equity
  • Hedge Funds

Real Estate

  • Venture Capital
  • Asset Management

Equity Research

  • Investing, Markets Forum
  • Business School
  • Fashion Advice
  • Interview Questions

Investment Banking Interview Questions and Answers

101 common technical, fit, behavioral, and logic investment banking interview assessments with sample answers

Rohan Arora

Mr. Arora is an experienced private equity investment professional, with experience working across multiple markets. Rohan has a focus in particular on consumer and business services transactions and operational growth. Rohan has also worked at Evercore, where he also spent time in private equity advisory.

Rohan holds a BA (Hons., Scholar) in Economics and Management from Oxford University.

Patrick Curtis

Prior to becoming our CEO & Founder at Wall Street Oasis, Patrick spent three years as a Private Equity  Associate for Tailwind Capital  in New York and two years as an Investment Banking Analyst at Rothschild.

Patrick has an  MBA  in Entrepreneurial Management from The Wharton School and a BA in Economics from Williams College.

IB Interview First Impressions - Carrying Yourself

  • ​Common First Investment Banking Interview Questions: Crafting Your Story

30 Common Investment Banking Technical Questions

  • WSO's Bonus IB Interviews Tip From An Interviewer

14 Bank-Specific Hard Technical Questions

16 ib product group-specific questions, 16 common ib industry-specific questions, 12 most common behavioral fit questions, 7 exclusive bank-specific behavioural/fit questions.

  • 6 Logical Puzzles - Interview Brain Teasers
  • Questions To Ask The Interviewer At The End

Bonus Interview Preparation Tips

Hirevue investment banking tips.

  • M&I 400 IB Questions Guide Vs WSP's The IB Interview Guide (The Red Book) Vs WSO's IB Interview Prep Course

Full WSO IB Prep Guide & Additional Resources

  • List Of Bulge Bracket Investment Banks And Boutiques

The Investment Banking (IB) interview process is highly competitive and designed to rigorously filter out potential candidates. Consequently, answering the behavioral, technical, and logical questions that are asked in the interview with proven answers that we provide is key to converting an interview into an offer. 

The following free WSO IB interview guide is a comprehensive tool designed to cover every single aspect of the interview process, guiding you from the very beginning to the very end. 

  • This guide features 101 of the most common technical, behavioral, logical, and group-specific questions that are asked by investment banking professionals to candidates during the hiring process as well as sample answers to each one of them. It is a great place to start your preparation before investing in our more comprehensive IB interview course .
  • This resource includes 21 bank-specific questions from bulge bracket investment banks (Goldman Sachs, J.P Morgan, Citi, etc.).
  • This interview guide consists of 16 sections which cater to various phases of the interview process.

Before we begin, we wanted to make sure you know about WSO Academy, a program that guarantees you are job in high finance (including investment banking)...

WSO Academy

The Only Program You Need to Land in High Finance Careers

The most comprehensive curriculum and support network to break into high finance.

investment banking case study interview questions

We’ve all heard about it at one point or another. Forbes has written on it. “First impressions are the best impressions.”

Within just a few seconds of meeting, people will form a solid opinion of who you are. Perfecting your first impression while carrying yourself with a healthy balance of confidence and humility lays the foundation and tone for the rest of the interview. The following section has been written by Patrick Curtis, CEO of WallStreetOasis, based on his vast experience of interviewing candidates for investment banking positions. 

Read it over, perfect your entry and learn how to leave a lasting impression on your interviewer from the get-go.

How to Carry Yourself

  • The biggest thing I am looking for is humble confidence - someone I would like to grab a beer with.
  • Listen, listen, listen!!! So many mistakes happen just because of not listening carefully and not being in the moment.
  • Be punchy, brief, and learn how to end a sentence. I can't tell you how many times people have gotten into trouble by rambling off into some ass-backward irrelevant tangent. Learn to be comfortable with a little silence here and there while we absorb your answer.

​ Common First Investment Banking Interview Questions: Crafting Your Story

There are no excuses for not perfecting what is in your control. Irrespective of the bank, the position, or your region, you can be sure these 2 questions will be asked as they’re a standard in the industry. 

Anticipating both of these questions beforehand, crafting a compelling narrative around them, and selling yourself on it will make you stand out from amongst the pool of potential candidates. 

1. Walk me through your background/resume

Dial-in a cohesive 90-second resume walkthrough that focuses on the positive and motivating factors behind every transition (school to job, job to better job, most recent job to grad school).  A good example:

I went to school to learn how to design cars, but after my first internship I realized that I like interacting with clients directly and pursued full-time roles in B2B sales. In these sales roles, I developed solid selling skills as well as gained exposure to a, b, and c. Since I wanted to continue honing those skills and branch out to focus on x, y, and z, I am seeking a new role/promotion which provides that opportunity…

Be deliberate. Every move you made should have a reason (preferably that you initiated). Don't be negative. Never say you left because you were bored or "wanted to try something new."

2. Why investment banking?

The answer to this question should be tailored uniquely to you and to the firm you are interviewing with. While answering this question, it is key to capitalize on your previous professional/leadership experience, highlight it and create a logical path as to why you are now trying to break into investment banking. A good answer to this question is addressing the three main reasons, illustrated below with an example each:

investment banking case study interview questions

Given the variety of professional backgrounds that candidates come from, WSO has created a dedicated page to answer this question.

Did you know?

WSO’s “Why investment banking?”  article covers 43 sample answers , tailored for students and professionals from backgrounds ranging from law to consulting.

Technical questions are a critical component of almost every investment banking recruiting process. You WILL be asked these questions, and your interviewers will expect detailed and accurate responses. The following section features 30 of the most common IB interview questions, with a detailed sample answer for each of them. 

At the end of these 30 questions, we also have provided you with 14 exclusive bank-specific technical questions (from 7 bulge bracket banks) to kickstart your mock interview training.

1. What are the three main financial statements?

Sample Answer: The three main financial statements are

  • Income Statement
  • Balance Sheet
  • Statement of Cash Flows

The Income Statement discloses a company's revenues and expenses, which together yield net income over a period of time. The Balance Sheet discloses a company's assets, liabilities, and equity on a specific date. The Cash Flow Statement starts with net income from the Income Statement; then adjusts for non-cash expenses, non-operating expenses like capital expenditures, changes in working capital, or debt repayment and issuance, to arrive at the company's closing cash balance.

2. How are the three main financial statements connected?

Sample Answer: Net income flows from Income Statement into the Cash Flow Statement (CFS) as Cash Flow from Operations. Net income less dividends are added to retained earnings from the prior period's Balance Sheet (BS) to come up with retained earnings as on the date of the current period's BS. The opening cash balance on the CFS is from the prior period's Balance Sheet while the closing cash balance on the CFS is the balance on the current period's Balance Sheet.”

The following chart gives you a more comprehensive overview of how the 3 financial statements are connected to help visualize and present better for your interview:

investment banking case study interview questions

3. If you could use only one financial statement to evaluate the financial state of a company, which would you choose?

Sample Answer: The cash flow statement because it shows the actual liquidity of the company and how it is generating and using cash. The balance sheet just shows a snapshot of the company at a point in time, without showing the performance of the company, and the Income statement has several non-cash expenses that may not be affecting the company's health and can be manipulated. Overall, the key to a great company is generating significant cash flow and having a healthy cash balance, both of which are disclosed in the CF statement.

4. How would a $10 increase in depreciation expense affect the three financial statements (assuming a 40% tax rate)?

Sample Answer: In the income statement, the depreciation increase of $10 is set off by a reduction of $4 on taxes as depreciation is a tax-deductible expense for the net reduction in net income of $6. In the cash flow statement, net income is reduced by $6, depreciation is increased by $10, net cash from operations and total cash is increased by $4. This increase in cash is because depreciation is a non-cash expense that has no impact on cash while the reduction in taxes affects the cash flow. In the balance sheet, property, plant, and equipment balances reduce by $10, cash balance increases by $4, and retained earnings reduce by $6 due to the reduction in net income. 

The following points summarize this:

On the income statement

  • $10 depreciation expense, 40% tax rate
  • Reduction in net income of $10 x (1 - 40%) = $6

Reduction in net income flows to cash from operations

  • Net income reduced by $6
  • Depreciation increases by $10
  • Net increase in cash from operations of $4
  • Ending cash increases by $4

Ending cash flows onto th e balance sheet

  • Cash increases by $4
  • Property, plant, and equipment lose $10 in value
  • Net decrease in assets of $6, matches the net drop in shareholder equity due to the reduction of retained earnings from the $6 is net income

5. “Walk me through the Income Statement”

Sample Answer: The first line of the Income Statement represents revenues or sales. From that, we subtract the cost of goods sold, which gives gross margin. Subtracting operating expenses from gross margin gives us operating income (EBIT). We then (add/subtract) interest expense (income), taxes, and other expenses (income) to arrive at Net Income.

6. What is Enterprise Value?

Sample Answer: Enterprise Value (EV) is the value of an entire firm, both debt, and equity. This is the price that would be paid for the company in the event of acquisition without a premium.

EV = Market Value of Equity + Debt + Preferred Stock + minority interest - Cash

investment banking case study interview questions

7. What is WACC and how do you calculate it?

Sample Answer: WACC is the acronym for Weighted Average Cost of Capital. It reflects the overall cost for a company to raise new capital, which is also a representation of the riskiness of investment in the company (higher the risk, higher the cost of capital). It is commonly used as the discount rate in a discounted cash flow analysis to calculate the present value of a company's cash flows and terminal value .

The formula below helps you calculate the WACC of a company if you are put on the spot and asked to calculate it as part of your technical interview:

investment banking case study interview questions

where, E = Market value of equity D = Book value of debt P = Value of preferred stock K E = Cost of equity (Calculate using CAPM) K D = Cost of debt (Current yield of debt) K P = Cost of preferred stock (Interested rate on preferred stock) T = Corporate tax rate

8. What is EBITDA?

Sample Answer: EBITDA stands for Earnings before Interest, Taxes, Depreciation, and Amortization. It gives us a good idea of a company's profitability and is a quick metric for free cash flow because it will allow you to determine how much cash is available from operations to pay interest, CAPEX , etc.

EBITDA = Revenue - Expense (except depreciation and amortization)

It is also often used for rough valuations in a comparable company or precedent transaction analysis as part of the EV/EBITDA multiple.

9. Would you be calculating enterprise value or equity value when using a multiple based on free cash flow or EBITDA?

Sample Answer: EBITDA and free cash flow represent cash flows that are available to repay holders of a company’s debt and equity, so a multiple based on one of those two metrics would describe the value of the whole business from the perspective of all its investors. 

A multiple such as the P/E ratio, based on earnings alone, represents the amount available to common shareholders after all expenses are paid, using which you would be calculating the value of the firm’s equity.

10. Can a company have a negative book equity value?

Book equity value is the accounting value of equity derived by subtracting the value of a company's liabilities from its total assets. It is the total shareholder’s equity, an amount shown as “Total Equity” in the Balance Sheet of the company.

Sample Answer:   Yes. If there are large cash dividends or if the company has been operating at a loss for a long time.

11. What is typical of an LBO (leveraged buyout) transaction?

A firm (usually a PE firm) uses a high amount of debt (70 - 90%) to finance the purchase of a company, then uses the company's cash flows to pay off that debt over time. 

The acquired company’s assets may be used as collateral. Ideally, the original debt of the acquired company would have been partially retired at the time of exit.

In the context of a private equity investment, the debt acts as a way to magnify returns (boost IRR for the fund), but it can also backfire if the acquisition turns south.

12. Why would a company issue equity rather than debt to fund its operations?

Sample Answer: There are many reasons why a company would want to issue equity instead of debt. Some of them are:

  • If the company feels its stock price is inflated, it can raise a relatively large amount of capital with comparatively minimal dilution to existing shareholders.
  • If the projects the company is looking to invest in do not produce immediate or consistent cash flows to pay its debt.
  • If the company wants to adjust the cap structure or pay down debt.
  • If the owners of the company want to sell off a portion of their ownership.

13. How is it possible for a company to have a positive net income but go bankrupt?

Sample Answer: This is possible if working capital erodes (such as increasing accounts receivable, lowering accounts payable, lower inventory turnover) or the company is growing so fast that it’s unable to raise enough capital to fund operations. Another possibility is the existence of financial fraud.

14. What are some ways you can value a company?

Sample Answer: The three most common ways of valuing a company are: Comparable companies or multiples analysis: This is the most common way to value a company. This method attempts to find a group of companies that are comparable to the target company and to work out a valuation based on what they are worth. The idea is to look for companies in the same sector and with similar financial statistics (Price to Earnings, Book Value, Free Cash Flow, EBITDA, etc) and then assume that the companies should be priced relatively similarly.

Market valuation or market capitalization: In this method, the market value of equity is used and hence can only be used for publicly traded companies. It is calculated by multiplying the number of shares outstanding by the current stock price.

Discounted cash flow analysis: This method involves calculating the sum of the present values of all future cash flows to give the value of the entire company including debt and equity, which is also called enterprise value .

15. Which of the valuation methodologies will result in the highest valuation?

Here is a list of the four valuation methodologies organized from highest valuation to lowest valuation:

  • Precedent Transaction - Since a company will pay a control premium and a premium for synergies coming from the merger, values tend to be high.
  • Discounted Cash Flow - Those building the DCF model are frequently optimistic in their projections. 
  • Market Comps - Based on other similar companies and how they are trading in the market. No control premium or synergies.
  • Market Valuation - Based on how the target is being valued by the market. Just equity value, no premiums or synergies.

16. Why might there be multiple valuations of a single company?

Sample Answer:   Each method of valuation will generate a different value because it is based on different assumptions, different multiples, or different comparable companies and/or transactions. Generally, the precedent transaction methodology and discounted cash flow method lead to higher valuations than comparable companies' analysis or market valuation does. 

The precedent transaction result may be higher because the approach usually will include a “control premium” above the company’s market value to entice shareholders to sell and will account for the “synergies” that are expected from the merger. 

investment banking case study interview questions

17. Walk me through a DCF.

In an interview, it is important to keep your technical overview at a high level. Start with a high-level overview and be ready to provide more detail upon request.

Sample Answer:  

  • Project out cash flows for 5 - 10 years depending on the stability of the company
  • Discount these cash flows to account for the time value of money
  • Determine the terminal value of the company - assuming that the company does not stop operating after the projection window
  • Discount the terminal value to account for the time value of money
  • Sum the discounted values to find an enterprise value
  • Subtract the present value of debt (this is generally the market value of debt) and then divide by diluted shares outstanding to find an intrinsic share price

Common questions that follow this are:

Why do you multiply by (1-tax rate)?

Sample Answer: You do this because interest expense (the cost of debt) is tax-deductible so you need to account for the benefit provided by this "debt tax shield."

What is the cost of equity?

Sample Answer: The cost of equity is usually calculated using the Capital Asset Pricing Model (CAPM). CAPM = Risk-free rate + Beta * (Expected market return - Risk-free rate)

What is the exit multiple method for determining the terminal value?

Sample Answer: Find an industry average multiple and multiply it by final year revenue (if using EV/Revenue) or final year EBITDA (if using EV/EBITDA).

18. What is an Initial Public Offering (IPO)?

Sample Answer 1: An IPO is the first public sale of stock in a previously private company. This is known as “going public.” The IPO process is incredibly complex, and investment banks charge high fees to lead companies through it. Companies go public for several reasons—raising capital, cashing out for the original owners, and investor and employee compensation. Some negatives against “going public” include sharing future profits with public investors, loss of confidentiality, loss of control, IPO fees to investment banks, and legal liabilities.

investment banking case study interview questions

19. What is the difference between accounts receivable and deferred revenue?

Sample Answer: Accounts receivable is revenue, which has been earned and recognized because the product has been delivered, but the customer has not yet paid the cash. Deferred revenue is cash that has been collected for products that have not yet been delivered, so the revenue has not yet been recognized. Accounts receivable is an asset on the Balance Sheet, whereas deferred revenue is a liability.

20. When calculating enterprise value, do you use the book value or the market value of equity?

Sample Answer: You should use the market value of equity always because the book value is not adjusted once it is recorded in the books at the time of issue of the shares. It is common to very often see a share priced in the hundreds or thousands having a face value of $1 or $10. This is due to the historical nature of accounting. Hence, the book value of equity is useless for any kind of valuation, and market value is the preferred metric to use.

21. If enterprise value (EV) is $80mm, and equity value is $40mm, what is the net debt?

Enterprise Value = Equity Value + Net Debt + Preferred Stock + Minority Interest.

Sample Answer: If we assume there is no minority interest or preferred stock, then Net Debt will be $80mm – $40mm, or $40mm.

22. What is the difference between cash-based accounting and accrual-based accounting?

Sample Answer: Cash-based accounting recognizes sales and expenses when cash flows in and out of the company. 

Accrual-based accounting recognizes revenues and expenses as they are incurred regardless of whether cash flows in or out of the company at that exact time.

Accrual-based accounting is the more common method for large corporations.

23. What are the major factors that drive mergers and acquisitions?

investment banking case study interview questions

24. All else equal, should the WACC be higher for a company with a $100 million market cap, or a company with a $100 billion market cap?

Sample Answer 1: Normally, the larger company will be considered “safer” and therefore will have a lower WACC all else being equal. However, depending upon their respective capital structures, the larger company could also have a higher WACC.

Sample Answer 2: Without knowing more information about the companies, it is impossible to say. If the capital structures are the same, then the larger company should be less risky and therefore have a lower WACC. However, if the larger company has a lot of high-interest debt, it could have a higher WACC.

25. What is Beta?

Sample Answer: Beta is a measure of the volatility of an investment compared with the market as a whole. The market has a beta of 1, and hence, investments that are more volatile than the market have a beta greater than 1 while those that are less volatile have a beta less than 1.

26. How/why do you lever or unlever Beta?

Sample Answer: Unlevering beta allows us to remove the effect of debt in the capital structure. This shows us the beta of the firm's equity had it not used any leverage in its capital structure. Also, if we are trying to do a market comparison with a company that's not on the market (so no beta), you can take a comparable company and unlever its beta and use this unlevered beta as a proxy for the unlisted company's beta.

investment banking case study interview questions

27. What is net working capital?

Net Working Capital = Current Assets – Current Liabilities

Current assets include items on the Balance Sheet like inventory, accounts receivable, prepaid expenses, and other short-term assets. Current liabilities include items such as accounts payable, accrued expenses, deferred revenue, and other short-term liabilities.

An increase in net working capital means more cash is tied up in the operations. This could be from increasing current assets like inventory or accounts receivable. If you increase inventory, for example, it is not (yet) a cost on the Income Statement, but still blocks the cash that was used for purchasing the inventory which needs to be accounted for on the CF statement. This is why in calculating free cash flow you subtract an increase in net working capital.

A decrease in net working capital means less cash is tied up in operations. This could happen due to changes such as increasing accounts payable or reducing inventory. If you reduce inventory, it means you are selling more goods than you are producing, which means you are realizing a cost on your Income Statement. If you are increasing accounts payable, you are preserving your liquidity by taking a little bit longer to pay your vendors for your raw materials/inputs.

Sample Answer: Net Working Capital is calculated as current assets minus current liabilities. It is a measure of a company’s ability to pay off its short-term liabilities with its short-term assets. A positive number means they can cover their short-term liabilities with their short-term assets. A negative number indicates that the company may have trouble paying off its creditors, which could result in bankruptcy if cash reserves are insufficient and further financing cannot be arranged.

28. What happens to free cash flow if net working capital increases?

Intuitively, you can think of working capital as the net dollars tied up to run the business. As more cash is tied up (either in accounts receivable, inventory, etc.), free cash flow will be reduced.

Remember that if the assets go up in value (denoting a purchase of assets), this is a use of cash; and if a liability goes up (denoting funds received), it is a source of cash.

Sample Answer: You subtract the change in Net Working Capital when you calculate Free Cash Flow, so if Net Working Capital increases, your Free Cash Flow decreases and vice versa.

29. How can a company raise its stock price?

Sample Answer: There are many ways a company can raise its stock price, a few of which are:

  • A company can repurchase stock, which lowers the number of shares outstanding and therefore increases its value per share.
  • It can improve operations to produce higher earnings, causing its EPS to be higher than anticipated by industry analysts, which will send a positive signal to the market.
  • It can announce a change to its organizational structure such as cost-cutting or consolidation, which would lead to increased earnings in general.
  • It could announce the institution of a dividend policy or an increase in an existing dividend.
  • It can announce an accretive merger or an acquisition that will increase earnings per share.

30. If you were the Chief Financial Officer (CFO) of a Fortune 500 company, what would be your concerns? Explain from a high level what the long-term financial implications are for your company.

Sample Answer: Fortune 500 companies are usually in the mature stage of their business lifecycle. This means they have stable growth accompanied by a good amount of stable cash flows and balances. As a CFO of one, I would look out for signs of declining products or services to be discontinued while also actively keeping an eye out for opportunities to expand and grow, either through mergers and acquisitions or by increasing the spending on internal research and development. 

investment banking case study interview questions

WSO’s Bonus IB Interviews Tip From an Interviewer

Walk me through every calculation you are doing. I want to hear you think out loud. 

  • The process matters far more than your answer and gives you a chance to demonstrate a grip on the concepts.

I sometimes just ask for a simple calculation, along with "Are you sure?" to put on a little pressure and see how you respond. 

  • I am looking here for you to take five seconds, double-check your math, and answer with a confident, "Yes, I am sure."

Don't be afraid to say, "I don't know" to a tough technical question. 

  • As an analyst, I expect you to ALWAYS tell me when you don't know something, and never wing your way through an answer. 
  • If you give an, "I don't know" follow it with, "But here is what I am thinking. Tell me if I am on the right track." 
  • And walk me through your thoughts. This is a great analyst quality because it shows that you'll think about a problem critically before you call me and ask about it.

Being able to clearly, confidently, and consistently answer the 30 technical questions bank-specific questions above will undoubtedly give you a competitive edge over the applicant pool. However, to achieve full technical mastery, it is critical you expect technical questions that are specific to each of the investment banks.

The following section features 14 exclusive questions (2 per investment bank) for 7 of the biggest investment banks in the world, to help kickstart your training process for your interviews and superdays .

The following questions have been taken from WSO's company database which is sourced from the detailed experiences of more than 30,000 people with IB interviews. The WSO IB Interview Course includes access to over 4,000 interview questions across 400+ banks (no other resource comes close).

Goldman Sachs IB Interview Questions

At a high level, there are 5 steps to an LBO:

  • Calculate the total acquisition price, including the acquisition of the target's equity, repayment of any outstanding debt, and any transaction fees (such as the fees paid to investment banks and deal lawyers, accountants, consultants, etc.)
  • Determine how that total price will be paid, including: - Equity from the PE sponsor,  - Roll-over equity from existing owners or managers,  - Debt, seller financing, etc.​
  • Project the target's operating performance over ~5 years and determine how much of the debt principal used to acquire the target can be paid down using the target's FCF over that time.
  • Project how much the target could be sold for after ~5 years in light of its projected operating performance; Subtract any remaining net debt from this total to determine projected returns for equity holders.
  • Calculate the projected IRR and MoM return on equity based on the amount of equity originally used to acquire the target and the projected equity returns upon exit.

Web picture of stocks

  • An increase in cash flow causes an increase in future value (FV)
  • An increase in the growth rate of future cash flows

Factors that may cause a company's PV to decrease:

  • Increased discount rate
  • Delay in receiving future cash flows
  • Reduction in the growth rate of future cash flows

JPMorgan Investment Banking Interview Questions

Web picture of factory

Sample Answer 2: The Nigerian election takes place in February. Four years ago, President Muhammadu Buhari gained power on a surge of optimism, pledging to restore security and end corruption. His Presidential record has been mixed, and his popularity and health have declined (he recently denied rumors of being replaced by a body double). The old regime may regain political power, impacting the free flow of goods through the country.

An increase in the interest rates will affect the cost of borrowing for companies. This means a lesser amount of funding from banks, which leads to companies having slower growth on average as compared to before the interest rate hike.

The higher cost of borrowing will also affect DCM. I would expect companies to issue fewer bonds or maintain the same capital structure but cut back on other expenses e.g. layoffs. Given the slower growth of companies, I would expect lesser interest from investors on IPOs. The slower growth and low valuations will then lead to an increase in M&A by strategics. On the other hand, the higher cost of borrowing might reduce the amount of leveraged M&A activity at the same time.

Overall, I feel that the increase in interest rates will affect M&A and capital markets negatively, and thus hiring will be down next year.

Morgan Stanley Interview Questions

Web picture of airplanes

  • Retail or Airlines: EV / EBITDAR (Earnings Before Interest, Taxes, Depreciation, Amortization & Rental Expense)
  • Energy: EV / EBITDAX (Earnings Before Interest, Taxes, Depreciation, Amortization & Exploration Expense), EV / Daily Production, EV / Proved Reserve Quantities
  • Technology (Internet): EV / Unique Visitors, EV / Page views

Note: Feel free to use multiples that you have picked up from other sources. These are for illustrative purposes.

An earthquake would cause the country’s GDP to immediately decline sharply due to the immediate effects of the earthquake as a lot of productive resources may be put out of use. But then the GDP growth will start to increase to an above-average level as there would be an increased amount of spending on rebuilding the infrastructure.

Citigroup Interview Questions and Sample Responses

Yes, a company could have a negative book equity value if the owners are taking out large cash dividends or if the company has been operating for a long time at a net loss, leading to the company having to take on debt to fund loss of cash. Eventually, equity can be negative implying that the entire operation is funded by debt.

This question is a lot more broad, giving you a lot of room to work with. A common method of answering this question would be bringing up 2-3 different types of financial risk concepts, giving a straight definition as to what they are, and following up with an example to demonstrate applied understanding.

investment banking case study interview questions

Credit Risk - This is the risk of a possible loss being incurred by a business or an individual, should their borrower fail to repay a loan or meet contractual obligations. It is impossible to quantify credit risk and precisely predict which borrowers will default on loans, but there are risk management teams built to minimize a business’ risk and manage their credit exposure. An example of credit risk would simply be a bank lending a citizen a loan of $100,000 to start their business as an entrepreneur, on which the bank incurs the risk of not having the loan repaid should the citizen’s business go bankrupt.

Another related type of risk would be,

Interest Rate Risk - This is the risk incurred where there may be a reduction in the value of investment assets should the interest rate environment change drastically in a short period. An example of this would be that if interest rates increased, the value of fixed-income investments would decrease.

BoA Sample Questions on Superday

Terminal Value or TV is the value of any investment at the end of the investment period. This will usually assume a constant growth rate into the future. it can be calculated by applying an exit multiple to the company’s last projected year’s EBITDA, EBIT, or Free Cash Flow (multiples method). Alternatively, the Gordon Growth method can be used to estimate TV based on its growth rate into perpetuity.

The formula for calculating TV without accounting for growth is: Expected cash flow / (1 + Required rate of return)^Time

The formula for calculating TV using Gordon Growth is: Terminal Value = Expected dividend / (Required rate of return – Growth rate).

The investment banking division is sometimes referred to as corporate finance and is broadly split into 2 sectors, products and industries. The purpose of both is to provide advisory on transactions, mergers, and acquisitions and to arrange (and sometimes even provide) financing for these transactions.

Investment banking product groups are broken down into:

Web picture of Bank

  • Leveraged Finance (LevFin) - Issuance of high-yield debt to firms to finance acquisitions and other corporate activities.
  • Equity Capital Markets (ECM) - Advice on equity and equity-derived products (IPOs, shares, capital raises, secondary offerings, etc.)
  • Debt Capital Markets (DCM) - Advice on raising and structuring debt to finance acquisitions and other corporate activities.
  • Restructuring – Improving the structures of a company to make it more profitable or efficient.

Credit Suisse Interviewing Questions

M&A seems to be off the table because REITs have low cash balances and can't do stock issuances because they would be dilutive. Therefore, my advice would be to basically sell assets in non-core markets to raise cash.

Company A’s EPS is $10 / 10 = $1.00.  To complete the deal, Company A must issue 6 ($150 / $25.00) new shares which means that the combined share count after the deal is 16 (10 + 6).  Since no cash or debt was used and the tax rates are the same and the combined net income = Company A net income + Company B net income = $10 + $10 = $20.  The Combined EPS, therefore, is $20 / 16 = $1.25, which is an increase of 25% in the EPS, and this is what is called accretion.

UBS Questions Asked by Interviewers

Thinking of cash flow/share the same way as earnings per share, the PE for A is 2.5 and B is 2.5. As both their EPS are equal, the transaction is neither accretive nor dilutive.

You need to remember that the leverage multiple stands for Debt/EBITDA; so calculating out the leverage multiples, you will see both A and B are leveraged at 2.5, hence, the combined leverage multiple of A and B is still 2.5; if the transaction is equity-financed, the leverage would decrease and the company would be de-leveraged; deals are usually more accretive with debt due to tax deduction on interest expense.

​ Most Common Technical Interview Question - WSO Bonus: ​

15. “pitch me a stock”.

Many interviewers will ask you in one way or another to pitch a stock if you have any experience with trading, a private wealth management internship, a hedge fund internship, or anything that deals with market transactions. If this is you, spend 30 minutes to a couple of hours finding a stock you like and why. Even if it doesn't get asked, it's always better to be safe than sorry. Here's a good explanation of how to answer this question.

investment banking case study interview questions

Do you have an M&A group interview coming up? Or perhaps an ECM superday right around the corner? We’ve got you covered.

The following 16 group-specific questions have been taken from our forums and company database where over 30,000 candidates have reported their interviewing experiences for different divisions within investment banks. 

The following questions and sample answers will help you achieve specialized focus and demonstrate expertise for the group you’re interviewing for.

Mergers & Acquisitions (M&A)

The main reason two companies would want to merge would be the synergies the companies could create by combining their operations. However, some other reasons include gaining a new market presence, an effort to consolidate their operations, gaining brand recognition, growing in size, or gaining the rights to some property (physical or intellectual) that they couldn't gain as quickly by creating or building it on their own.

A strategic buyer is generally a corporation that wants to acquire another company for strategic business reasons such as synergies, growth potential, etc. An example of this would be an automobile maker purchasing an auto parts supplier in order to gain more control of their COGS and keep costs down.

A financial buyer is generally a firm looking to acquire another company purely as a financial investment. An example is a private equity fund doing a leveraged buyout of the company.

Leveraged Finance (LevFin)

Web picture of credit card

  • Credit card

Web picture of car

A car loan is riskier than a home loan because a car loses its value much quicker. To compensate for the higher risk profile and lack of collateral, credit card companies charge much higher interest rates when compared to a typical car loan and mortgage while the risk associated with a lower value of collateral in car loans is why they carry higher interest rates compared to mortgages.

The software company because of recurring revenues from annual contracts that are even more guaranteed than a hardware store, assuming that both companies are mature.

Debt Capital Markets (DCM)

Think about how bonds are priced – based on their discounted future cash flows. If any of those cash flows is in doubt, then the bond's value falls accordingly. (Think of a UST bond as being priced with risk-weighted cash flows of 100%. A BBB bond might be priced with risk-weighted cash-flows of 95%, just as an example - although in reality the bonds are priced with a spread/all-in yield that implicitly contains the risk, rather than calculating the risk % driving the spread). So any macro event that would impact companies' profitability/cash flow would affect the price of corporate bonds. That said, corporate bond spreads are more driven by micro factors than by broader economic trends unless those economic/systemic factors are very pronounced.

The price and yield of a bond move inversely to one another. Therefore, when the price of a bond goes up the yield goes down.

The reason for this is that the return on a bond (when annualized, this is called yield) is the difference between its current price and future repayment (generally bonds are redeemed at par). The lower the price, the higher is the return as the repayment is constant regardless of its price. As the price increases, the return reduces thereby reducing the yield. 

Web pic showing relationship between bond price and yield

Let’s understand this better with the help of an example. Let’s assume a bond can be redeemed at a par value of $100 on maturity (one year from now). Let’s now assume that the bond is trading at $80. The yield on the bond can be calculated as 25% ((100/80) - 1). What if the price instead was $90? The yield reduces to 11.11% ((100/90) - 1). Hence, higher prices mean lower yields and vice versa.

Equity Capital Markets (ECM)

An Initial Public Offering or IPO is the very first sale of stock to the public by a private company. This is also known as “going public”. Two kinds of companies will undertake an IPO:

  • Startup companies looking to raise capital and investors
  • Large private companies looking to become publicly traded

To find out more about IPOs, check out WSO’s “What Is An Initial Public Offering (IPO)?” page here .

Since a private company has no market capitalization and no beta, you would most likely use the WACC for a comparable public company adjusted upwards for the lack of liquidity.

Restructuring

This depends on the formal definition of the leverage ratio, but assuming debt/EBIT is implied we can set up two simple equations:

  • Debt/EBIT = 5
  • EBIT / (Debt * Interest rate) = 5

Solving these equations we find that the interest rate is 4%.

Assuming operating income is EBIT, add back depreciation (a non-cash expense) and deduct CAPEX to get Unlevered Free Cash Flow (UFCF) = 35. Then deduct the interest expense but add back interest tax shield, for the net expense of $9 (15 * (1 - 40%)) assuming 40% tax rate. This gives us a Levered Free Cash Flow (LFCF) of $26.

Value, because the payoff will be quicker. In high inflation periods, short-duration equities are favored as cash flows are eroded less by the higher cost of capital imposed by higher inflation. Growth equities need a longer holding period before capital yielding projects are realized, at which point the discount factor will be higher making them subject to more erosion from inflationary pressure.

It depends, if the product portfolio of the company is vastly different with varying risk profiles, then it would not be right to use WACC as the discount rate. The discount rate is the cost of capital. If the risks in each product line are vastly different, so should the cost of capital. Using a broad stroke denominator such as the company's WACC would not be right in this case.

Sales & Trading (S&T)

Two steps. I’d look at what they’re interested in, and then I’d look at how they wanted to change.

Quite impossible for equities, 0 beta would be risk-free like treasuries. You would have to find two industries that were negatively correlated to remove idiosyncratic risks.

Risk Management

They earn revenue through APR, interchange, late fees, and subscription fees and their primary costs are operations and marketing-related expenses.

Tapering is a balancing act to reverse the effects of quantitative easing once its objectives have been achieved. The Fed must consider the right rate of implementation so as to not lead the economy into a recession.

Are you looking for questions that are unique to particular IB group that you are interested in joining? Do you have an interview coming up and have preferences on what industry you want to join? We've got you covered.

The following 16 industry-specific questions have been taken from our forums and company database where over 30,000 candidates have reported their interviewing experiences for different divisions within investment banks.

The following questions and sample answers will help you achieve specialized focus and demonstrate expertise for the industry you're interviewing for.

Consumer & Retail

The answer to increasing your margins while having lower revenue is to cut back on expenses.  Revenue - expenses = gross profit and gross profit/revenue = gross margins. 

The increase in shareholders equity would be equivalent to the increase in assets (x), due to the effect of the equation below: Assets = Liabilities + Shareholders equity.

Assets are recorded at cost (what you paid for them).

Energy and Utilities

The main regulatory hurdle that the power sector faces is distributed electricity generation. Distributed generation poses a threat to existing power utilities because it takes away power demand from them.

However, there are a number of ways in which to handle this emerging trend. One way is to structure tariffs in a way that increases affordability. The other is to offer time-of-use tariff structures that can flatten the duck curve , which occurs when renewables ramp down in the evenings, exactly when electricity demand is highest.

If we assume it’s a single asset company, we can estimate the capital efficiency based on historical data and apply it against the company's CAPEX forecast to reach a "new additions" production estimate. From there, applying the historical asset production decline rate against "base" production and summing the numbers together leads us to a forward estimate.

Financial Institutions Group (FIG)

WACC decreases at low levels of leverage due to the tax shield created by interest payments and then increases exponentially due to the increasing riskiness of investing in a highly levered company.

A DCF involves predicting the unlevered free cash flows that a business will generate, discounting it into the present and then adding it up to get the enterprise value. 

A DDM on the other hand only looks at dividends the company pays, and then divides it using the required rate of return to find the value of equity.

A DCF typically spits out an enterprise value whereas DDM is an equity value based on the present value of projected free cash flow to equity (% of dividends for a bank since they have regulatory capital requirements, which is a limiter on growth assumptions on a bank DDM).

Comps are more market-based and sensitive to environmental trends, such that in times of high valuations (such as now), comps > DCF, and in times of lower than FMV valuations, DCF > comps.

This is because biotech companies don't operate in perpetuity, we assume that once the patent ends, generics will flood the market driving profits close to zero.

Industrials

Levered beta measures the risk of a firm with debt and equity in its capital structure to the volatility of the market. Unlevered beta removes the debt component.

Take terminal year EBITDA, calculate TEV using current EV/EBITDA multiples of comp set, and take the present value of that total.

Natural Resources

1. Upstream 2. OFS 3. Downstream 4. Midstream

Upstream is likely to be the most volatile, due to its association with risk of exploration and sensitivity to prices. OFS is second because it's adjacent to upstream. Downstream is third because demand is relatively inflexible and is subject mostly to price sensitivity. Midstream is the lowest beta because it's a volume business.

A Section 338(h)(10) election blends the benefits of a stock purchase and an asset purchase.

  • Legally it is a stock purchase
  • However, accounting-wise it’s treated as an asset purchase.
  • The seller is still subject to double-taxation – on its assets that have been appreciated and on the proceeds from the sale.
  • The buyer receives a step-up tax basis on the new assets it acquires, and it can depreciate/amortize them so it saves on taxes.

Even though the seller still gets taxed twice, buyers will often pay more in a 338(h)(10) deal because of the tax-savings potential. It’s particularly helpful for:

  • Sellers with high NOL balances (more tax-savings for the buyer because this NOL balance will be written down completely – and so more of the excess purchase price can be allocated to asset write-ups).
  • If the company has been an S-corporation for over 10 years – in this case, it doesn’t have to pay a tax on the appreciation of its assets.

The requirements to use 338(h)(10) are complex and bankers don’t deal with this – that is the role of lawyers and tax accountants.

- Find key ratios LTV, DSCR, Debt Yield - Assess property value - Assess similar properties in the area - Go over potential covenants

Some statistics to consider in such a situation would be,

  • Market Stats (Cap Rate, Rent, Vacancy)
  • Asset Class
  • Value add or Core

Technology / Media / Telecom (TMT)

MOIC’s simplistic calculation tells investors how much money they’re ultimately receiving from an investment while IRR includes the impact of time over which the returns were generated.

Considering the government measures involving lockdowns, in-store purchases are likely to decrease therefore lowering profits. This can, however, be countered by Apple building an extensive or expanding on its current online infrastructure to ensure an optimal and sustainable online shopping experience for customers.

You've reached the behavioral/ fit interview. This means that the firm believes you are smart enough for the job. At this point, the little things matter. Fit questions are a major part of the IB analyst interview. The focus of fit questions is to see who you are and how you would fit into the firm's culture.

This section features 12 of the most common behavioral/fit questions you are likely to be asked during your interviews, followed by an exclusive set of 7 bank-specific (from 7 bulge bracket banks) behavioral questions to support you in tailoring your response to each bank and walking out with an offer.

1. What are some of your strengths?

Web picture of theatre

If you can, bring up a strength that doesn’t appear on your resume but could catch attention. For example, if you have performed in concert or theatre all your life, learning to be poised in front of strangers and/or large groups, it may be a good strength to share.

2. What is the one word that describes you best?

Motivated, smart, driven, humble, efficient. All of these are good options to use. Make sure to have an example to back up whatever word you choose with a specific story.

3. What has been your favorite class in college and what was your grade?

The course you name should have something to do with business/finance/economics, and the grade you report should be a good one. If you say your favorite class was “dancing”, why are you looking to go into finance? Why do they want to hire you?

It is worth noting however, you must have a genuine justification and rationale behind claiming a finance class was your favorite. If you do not have a compelling reason behind your answer, interviewers will call your bluff and see through it easily.

4. “What is the most important thing your resume doesn’t tell me that I should know?”

Talk about a skill that is unique to you (something that makes you memorable) and that cannot be documented on a resume. Think about things like your communication skills, teamwork skills, etc…not your math skills, which can be seen in GPA or SAT scores. Once you decide on the quality you want to present, illustrate it with a story from your life. A common variation of this question is “What separates you from the last person with your GPA from your school?”

Sample answer: Ever since my freshman year of high school, I have loved to perform. I was in the musical each year of high school, and have had a lead role in a play each year in college. This has allowed me to develop a comfort speaking in public situations, and with people, I don’t know or have just met. I think that this will be an extremely valuable skill in finance, speaking with clients, on the phone, and when presenting my work to my coworkers.

5. “Tell me about a time that…”

There are countless variations of this question, from “Tell me about a time you acted with integrity” to “Tell me about a time that you had difficulty dealing with coworkers”. It is key to have a well-rehearsed response for each of them, and a general guideline to follow.

Ideally, you can come up with 6-8 stories that cover the 30-40 basic questions, with only slight modifications. DON’T wing it. For every potential question, map out the story using the SOAR framework.

Describe the Situation (10-15 seconds), Obstacle (10-15s), Action (60-75s), and Result (15-30s). Stories for these questions should be 1.5 - 2 minutes long and focus only on what's important.

6. What is your biggest weakness?

Seeing as to how common this question is (even outside of the finance industry), we have a dedicated page for this question to support you in answering it perfectly during the interviewer. The “Good Responses To Biggest Weakness Questions” page can be found here .

7. What are you looking for in this job?

The interviewer wants to make sure you are aware of what this job entails, and what most analysts get out of the experience. You should acknowledge the long hours and the heavy workload while making it clear that you are ready to take on the challenge. Emphasize the appeal of a great learning experience that you would be unable to get in any other job straight out of school. Explain how you relish the prospect of pushing yourself and being challenged to do your best work in this job, and working with and learning from successful people.

Sample answer: I am going into this as an unparalleled learning experience. Everyone I have spoken to within the industry tells me you learn everything on the job. While my undergraduate studies prepared me for business, I know that most of the skills I need will be acquired on the job. I understand the hours and the workload, and I want to work incredibly hard to gain real-world experience that isn’t available in any other profession at this stage in my career. I know these skills will prepare me for anything I want to do later in my career.

8. What has been your favorite job so far?

If possible, pick a job that requires similar skills to the job for which you are applying and explain why those skills or requirements made it you're favorite. Talk about how you were forced to learn on the fly or multi-task or think critically because those are all skills you will need in finance.

9. What has been your least favorite job so far?

Talk about a job where you were bored, or not challenged, or not busy. None of those things will be the case in finance so they won’t be an issue.

10. What competitive activities have you participated in, and have they been worthwhile?

Finance is competitive. Firms are always competing for business and colleagues can even be competitive with one another (although most won’t admit it). You need to show that you are comfortable in competitive situations, but still can act with class and show respect.

Sample answer: I played varsity football in college. We won the conference 2 consecutive years and played at the NCAA tournament. Working with my teammates to accomplish a common goal and beat the competition was an amazing experience, really exhilarating.

11. If you had a million dollars that you weren’t allowed to invest, how would you spend it?

Make sure your background backs up whatever answer you give. If you have never volunteered in your life, don’t say you’re going to donate your money to a non-profit and go work for them. Have it relate to something you are interested in, and make sure that whatever you are spending it on can cost roughly a million dollars.

12. What are you interested in?

There are two ways that you can answer this question properly and you may want to explore both in your answer. First, you can mention things that you are interested in that are job-related like keeping up with current events, studying for the CFA, etc. Second, you can speak about your hobbies and your interests outside of work. The latter works great if you happen to share an interest with your interviewer. Hint: If you know the names of your interviewer you can do a little research beforehand and see if you can find any common interests so you can push the conversation in that direction.

WSO's Bonus Tip

"Once you get yourself in that room, I don't care about your GPA or what else you have on your resume. I am looking for 3 things, A connection with myself and the firm's culture, Will you be able to do the work? and Do you have a passion for the job?"

As taken from our forum .

Knowing the culture of each bank before walking into an interview is key to clicking with the interviewer and walking out with an offer.

The following section features 7 exclusive questions for 7 of the biggest investment banks in the world, to help you jump start your training for the respective investment banks you are interviewing for.

The following questions have been taken from WSO’s company database which is sourced from detailed IB interviews experiences of more than 30,000 people.

1. What do you like to do outside of school and work? (Goldman Sachs Behavioral Questions)

Ideally, the interviewer is looking for anything you can speak genuinely and passionately about, and support it with examples of your dedication towards it from your past experiences.

investment banking case study interview questions

2. What makes you stand out / What should we remember you by? (JP Morgan Fit Questions)

This is similar to the ‘What are your strengths?’ question. Have a concise 30-second pitch prepared. Concentrate on the three main bullets highlighted in the introduction, and identify three of your traits that manifest those qualities. Examples include things like being extremely driven, never giving up, wanting to learn, looking for challenges, etc. Make sure you take only 20-30 seconds and speak with confidence, but make sure to avoid arrogance.

3. What are your leadership involvements outside of finance? (Morgan Stanley Fit Interview)

It is important to show that you are comfortable taking up a leadership role or working under the leadership of someone else. It is important to be able to do both. Talk about past projects that show you being successful in both types of roles. Talk about your teamwork skills (communication, collaboration, etc.) and how those skills are effective when you are the leader as well as when you are supporting someone else’s leadership.

4. Why Citi? (Citigroup Soft Skills Evaluation)

This question can generally be asked by any bank, and the preparation routine is consistent across all such banks. Ideally, you want to tie in and present an alignment between your interests and values, with the firm’s culture, and support this with examples.

An exceptionally strong way of demonstrating this would be networking with current investment bankers at the bank, and talking about the appreciation you felt towards the characteristics of those you networked with (refer to specific people wherever possible) and concluding with how that makes you feel the bank would be a great place to work.

5. What do you consider the biggest negative about this job? (Bank of America Behavioral Questions)

Your interviewer is giving you a chance to give a “negative” about the job and explain why you don’t see it as a negative. The overwhelmingly popular response to this question is the lack of work-life balance, long hours, very unpredictable schedules, etc. Quickly mention the negative and then move on to why it doesn’t bother you.

Sample answer: I have been fortunate enough to have a lot of contacts who work in finance, and their usual response to this question is the long hours. However, every single person I have spoken with has said that they enjoy their job and they think the hours are worth it. This job will give me 4 - 5 years of work experience in only two years. It’s an opportunity I crave and a learning experience I don’t want to miss. I am ready for the challenges and I want to show that I can handle them.

6. How important do you think it is to be maintaining connections in this industry? (Credit Suisse Fit Assessment Question)

While networking is generally seen as important for advancing one’s professional career irrespective of industry, there is significantly more weight placed on its value within the investment banking industry relative to other industries. Ideally, your answer should acknowledge this fact, and you should support this with examples of you maintaining connections with a variety of professionals and the insights you have learned through their shared experiences.

7. What are your hobbies? (UBS Behavioural Interviews)

This question is quite a personal one, so feel free to expand upon this as per your choice. One key point we’d like to iterate is that it is disadvantageous to “fake” an interest or hobby with the intent of faking a “click” with your interviewer. Interviewers can often see through this, and it could potentially harm your chances of getting an offer.

It is much more beneficial to highlight a hobby that requires a set of transferable skills or values to IB, such as competitive sports (which involve having a strong work ethic), and passionately speaking about them.

IB Technical Interview Course

Everything You Need To Break into Investment Banking

Sign Up to The Insider's Guide on How to Land the Most Prestigious Jobs on Wall Street.

investment banking case study interview questions

  6 Logical Puzzles - Interview Brain Teasers

Logical puzzles, brainteasers, and riddles are an important part of the interview process as they allow the interviewer to determine your critical thinking abilities.

For this section of the interview, interviewers aren’t focused on whether you get the right answers or not. Rather, they are interested in your thought process while solving the riddles you are presented with.

Given this, it is key to walk your interviewer through your thinking as you progress through the riddle, who may even probe you with questions to assist you. Giving them a rundown of your thoughts and occasionally asking if you’re headed in the right direction demonstrates your capabilities to reflect, and approach a problem with composure.

It is still, however, extremely useful to anticipate these logical puzzles beforehand to avoid being put on the spot and caught off guard in the interview. The following section has 5 commonly asked logical puzzles that you can prepare for beforehand to impress your interviewer.

1. A room with no windows has 3 light bulbs. You are standing outside with 3 switches that control each of the three bulbs. If you can only enter the room once, how can you determine which switch controls which bulb?

Answer: First, turn on two switches: call them Switch 1 and Switch 2. Leave them on for a couple of minutes to let them get nice and hot. Then, turn off Switch 1 and enter the room. The bulb that is lit should be the one that is controlled by Switch 2. Of the remaining two bulbs, the hot one is the one controlled by Switch 1. The last one, off and not hot, is controlled by Switch 3.

2. What is 17 squared? What’s 18x22?

Answer: Don’t worry; they want to know how you will handle this question, and it is not difficult if you think about it correctly.

Think 17 x 17 is just 17 x 10 plus 17 x 7. You know 17 x 10 is 170. Now 17 x 7 is 10 x 7 and 7 x 7. This gives you 170 + 70 + 49, or 289. Whatever you do, don’t panic!

Now see if you can do 18 x 22: 18 x 20 + 18 x 2. Easy, 360 + 36 = 396.

As far as brainteasers go, this is a rather common one. You will do better if you have practiced these types of questions.

investment banking case study interview questions

3. How many NYSE-Listed companies have 1 letter ticker symbols?

Answer: It could be 26 (letters in the English alphabet), but it is only 24 because I & M are saved for Intel and Microsoft, in case they change their minds.

4. How many gas stations are there in the United States?

Answer: With a question like this, the interviewer is looking at your thought process, not that you can figure out how many gas stations are in the U.S.

The easiest way to go about answering a question like this is to start small and work your way to the bigger question. Think about your town. Say your town has 30,000 people, and you have 5 gas stations serving that area.

investment banking case study interview questions

So you have 7,500 towns with 5 gas stations and 2,500 “towns” with only 1.

Do a little mental math and you get the number of 40,000 gas stations in the U.S.

5. A stock is trading at 10 and 1/16. There are 1 million shares outstanding. What is the stock’s market cap?

Answer: This is just a test of your mental math. If a fourth is .25, an eighth is .125, and a sixteenth is .0625... The stock price is 10.0625 and the Market Cap is 10.0625 million.

6. How many degrees are there between a clock’s two hands when the clock reads 3:15?

Answer: The quick thought would be 90 degrees, but it isn’t. If the clock is 360 degrees, the minute hand will be exactly at the 90-degree mark. The hour hand will be ¼ of the way between the 3 and the 4. Since there are 12 numbers, the 3 and the 4 are 30 degrees apart, making the hour hand 7.5 degrees beyond the 3, and 7.5 degrees from the minute hand. The picture below helps illustrate this better.

investment banking case study interview questions

Questions To Ask The Interviewer at the End

After approximately an hour of drilling from the interviewer, here you are, at the very end. The interviewer looks at you and asks, “Do you have any questions for me?” This is your last and best chance to leave a lasting impression, so ask a thoughtful question for which the answer cannot be found with a simple google search.

Some of the questions WSO recommends are

  • I bet you were in my shoes a few years ago - what initially attracted you to X bank?
  • How would you describe the culture here?
  • Can you talk about your role in the last [M&A, ECM, DCM] deal you worked in and what specifically you learned from it?

These questions have been taken from “What Are Some Good Questions To Ask The Interviewer At The End Of The Interview?”, posted by WSO user @soontobe. Check out his full post with additional questions here .

Once you have asked your question, listen sincerely to the interviewer’s response. They want to see if you are genuinely interested, and not simply asking for the sake of asking. Take down notes occasionally if you have to.

As you wrap up, thank the interviewer once again for their time and shake their hands firmly. Sometimes an interview may go extremely well, while at other times not so much so. Irrespective of the situation, show respect and leave on a positive note with the interviewer. A thank-you email a few hours after the interview is generally appreciated.

Resources to learn more about sending thank you emails after interviews:

  • The Muse - How to Write an Interview Thank You Note Email Template
  • Indeed - Sample Thank You Letter After Interview
  • Zety - Thank You Email After an Interview
  • The Balance Careers - Thank You email After Job Interview
  • Career Sidekick - Thank You Note After Interview

Having mastered all the technical and behavioral/fit questions, as well as logical riddles above, you are in an exceptionally strong position for your IB interviews.

However, there is much more to excelling in IB interviews than simply memorizing sample questions and answers. In a pool of highly qualified and competitive candidates, interviewers look for more than someone who simply aces technicals.

They look for a candidate who would fit right into the company, who are able to stand out and showcase their strengths in a way their competition can’t match.

Given this, it is no surprise that the most successful candidates go above and beyond in the preparation process, taking the time to master the art of selling themselves in an interview and clicking with their interviewers. They build upon their social skills, and utilize these soft skills to steer the direction of the interview in their favour, giving them an upperhand which cannot be matched.

When considering this, it is not a stretch to say one’s soft skills play just as much as a role in their interview’s outcome as their technical skillset.

If you’d like to get started on learning these powerful tips, here are some additional resources from WSO’s hall of fame.

3 Tools That Will Take You A Long Way

  • Read Dale Carnegie's How To Win Friends and Influence People : If you have read it, read it again. Take the time to practice every single one of the principles over and over again. You can become excellent at communicating with this one book, and if you are ambitious, please check out Influence by Robert Cialdini .
  • Master selling: Again, people think of selling as something you either have or you don't. WRONG. Selling is a skill that brings together preparation and the skills of communications and influence. Read How to Master The Art of Selling by Tom Hopkins . If you are serious, consider creating a Selling Black book to sell your most valuable commodity - YOU.
  • If you're bold, dip your toe in the water and learn the most advanced skills of influence - NLP and conversational hypnosis. As part of this, you will learn nuances of language and non-verbal communications that few people even know about. It's hard to learn from a book, so feel free to google for courses on NLP.

Note: This tip was provided by WSO user @geoffblades, A former investment banker at Goldman Sachs and investor at the Carlyle Group. The full writeup by him, “How Do You CRUSH Your Interviews?”, can be found here . 

Show Them What They Want to See

Your general personality will come out throughout the interview - there is no hiding it. Being able to do the work well is shown through the confidence you display while answering the general questions along with how well you answer the technicals. 

Showing passion is a little more difficult, but it comes out in different ways. A few things that make the interviewer say “This kid knows his stuff” are:

  • How excited you are to answer questions,
  • The level of detail that you can answer a technical question
  • The types of questions that you ask when I tell you about what I do and the product that I trade. 

Chances are, showing passion comes from a combination of those examples.

Additionally, this is learned more with experience, but there comes a point in the interview where you have the ability to gain control of the interview and steer it in the direction you want. If you're having a conversation about China or Michael Lewis, you can keep it going by talking about another book that you have read. If you are asked a question, you can answer it in a way that almost guarantees the next question, which you will be prepared for. 

The number of choke points in an interview where you can gain control and dictate flow is endless, you just need to learn how to spot them.

Note: These are 2 of 10 tips by WSO user @Gekko21. His full writeup, “Gekko's Guidance (10 Rules To Interviewing) - Part 1” can be found here .

So If You Want To Give Yourself A Leg-Up In Getting An Interview (Written by an interviewer)

Below are a few tips to help give you a boost in your investment banking interview:

  • It helps to have a high GPA but is not a guarantee for anything.
  • You SHOULD address the question of "Is he/she interested in banking?". If you've never interned in finance and are a non-traditional major, you should be actively involved (pref. at a senior level) in the finance clubs, you should participate in finance/modeling training seminars sponsored by your school, or have a section under interests with "Readings'' or "Favorite Books'' that have a finance tinge to them (more “ When Genius Failed ” or “ Fooled by Randomness ” or “ Barbarians at the Gate ”, less “ Monkey Business ” or “ Liars Poker ”)... Wall Street Journal , DealBook , FT , etc... I wouldn't advise adding that section if the rest of your resume already sells your finance interest... otherwise, it’s overkill and you seem uninteresting and boringly uni-dimensional. You want to be well-rounded.
  • Formatting is EVERYTHING and there is NO excuse for typos or inconsistencies in formatting.

The full undergrad recruiting series from our forums is available here:

  • UG Recruiting Part I: How A Resume Becomes An Interview
  • UG Recruiting Part II: The First Round Interview
  • UG Recruiting Part III: The Super Day And Offer

What is Hirevue?

Hirevue is a video conferencing program built to reduce hiring time for their clients whilst still attracting and acquiring talented candidates. Hirevue is used by many major banks including Goldman Sachs, JP Morgan, and Morgan Stanley.

How does Hirevue work?

Hirevue gives candidates a certain set of questions to answer during the interview process, in which their answers are recorded and then analyzed using artificial intelligence (AI). The results provide insights into candidates which investment banks then use to make informed hiring decisions.

The AI tracks and analyzes verbal, as well as non-verbal cues such as facial movements, body language, speech formation, as well as attire, and clothing. These data points are then processed by algorithms and generate results (employability score) allowing employers to predict a candidate’s performance on the job based on their presentation during the interview.

What are the questions asked in Hirevue interviews?

Hirevue aims to predict your capabilities for the job you’re applying for based upon your soft skills and presentation of yourself during the interview. As such, the interview consists of a majority of behavioral questions (as covered above), with some banks not even asking a single technical question to candidates.

The following is a list of 15 sample questions Hirevue has previously asked candidates during interviews, credit to The University of Colorado .

  • Tell me a little about yourself.
  • What are your long-term career plans?
  • What made you leave your previous job?
  • Tell me about your strengths and weaknesses.
  • Why do you want to work here?
  • What makes this position a good fit for you at this point in your career?
  • Tell us how your experience and training have prepared you for this position.
  • Give us your understanding of our organization
  • Tell us about a time when you had to balance multiple priorities. Please give an example that demonstrated how you navigated completing work priorities to attain the best result.
  • Describe a work scenario in which you were faced with competing priorities. How did you juggle them all and still meet everyone’s expectations?
  • Describe a high-pressure situation (either within a work setting, or beyond) that you were put in unexpectedly. How did you adjust and still create a successful outcome?
  • Tell us about a time you were most creative.
  • What do you enjoy about working in customer service? What do you dislike or find challenging?
  • Explain your approach to completing multiple assignments in a workday.
  • Tell us about a mistake you’ve made on the job and what you learned from it.

The following youtube video by Afzal Hussein , an ex-Goldman Sachs employee, shows him covering 18 questions and answers asked by J.P.Morgan’s Hirevue interviews.

WSO’s bonus tips to excelling in Hirevue interviews:

Web picture of suit

  • Dress appropriately (especially for investment banking interviews) This is pretty self-explanatory, make sure you are dressed well and look presentable.
  • Rehearse and practice You’re at an advantage with Hirevue as you can anticipate which questions are going to be thrown at you. Prepare for these questions beforehand, rehearse and practice your stories. Record yourself and replay it, find places to improve, and repeat the process until you are able to consistently and confidently answer questions. Ideally, the actual Hirevue interview itself should simply be a rehearsal of your past practice sessions, nothing should be catching you off guard. If you want the upper hand, ask your close friends and other respectable people in the industry to review your recordings and offer feedback. Perfect your delivery, practice makes perfect.

Web picture of microphone

M&I 400 Investment Banking Questions Guide vs WSP’s The Investment Banking Interview Guide (The Red Book) vs WSO’s Investment Banking Interview Prep Course

One question we receive a lot from students and professionals alike, given the many courses flooding the market, is which interview course is the best in the industry for breaking into investment banking. The following table and comparison present a comparison between the top three resource providers: the Wall Street Oasis (WSO), WallStreetPrep (WSP), and Mergers & Inquisitions (M&I). This comparison explains why we believe WSO’s IB Prep Course remains the gold standard in the industry, with features unmatched by competitors.

With that being said, WSO leads the industry for IB recruiting as the IB prep course gives you access to thousands of interview insights by actual candidates all across the world across a variety of divisions in finance. The advantages and insights gained by this are simply unmatched by our competitors, and we believe you’ll feel the same way as us once you’ve got our guide as well.

Many of the sample answers in the guide above were taken from WSO’s very own Investment Banking Interview Prep Course , which features:

  • 7,548 questions across 469 investment banks
  • 3 Modules to master technical + fit + networking
  • Access to Company Database for 12 months

Think about it - if this page can set you miles ahead of the competition, imagine what our complete guide can do for you.

Receiving the title of, “The Insider's Guide on How to Land the Most Prestigious Jobs on Wall Street” , the WSO IB Interview Prep Course will walk you step-by-step through the interview process, and place you in the strongest position to land the job. Click the button below to check it out.

Additional resources from our forums:

  • Investment Banking Resume Template - Official WSO CV Example
  • Investment Banking In The UK - An Overview
  • Investment Banking Analyst: A True Day In The Life
  • P/E Vs. EV/EBITDA - Advantages/Disadvantages?
  • Investment Banking Vs. Capital Markets - How Different Are They?
  • WSO Financial Dictionary
  • Some Observations From An MD

List of Bulge Bracket Investment Banks and Boutiques

Currently, the bulge bracket consists of the following banks:

  • Goldman Sachs (GS or Goldman) | GS Overview | GS Site
  • JPMorgan Chase (JPM) | JPM Overview | JPM Site
  • Morgan Stanley (MS) | MS Overview | MS Site
  • Credit Suisse (CS) | CS Overview | CS Site
  • Bank of America Merrill Lynch (BofA) | BofA Overview | BofA Site
  • UBS | UBS Overview | UBS Site
  • Deutsche Bank (DB) | DB Overview | DB Site
  • Barclays Capital (BarCap) | BarCap Overview | BarCap Site
  • Citigroup (Citi) | Citi Overview | Citi Site

Boutique IBs can range in size from reasonably large, global firms to tiny one-person firms. Below is a list of some of the larger and more well-known boutiques, sometimes referred to as the 'elite boutiques':

  • Piper Jaffray (Now Piper Sandler) | Piper Jaffray Overview | Piper Jaffray Site
  • Rothschild | Rothschild Overview | Rothschild Site
  • Moelis | Moelis Overview | Moelis Site
  • Jefferies & Co. | Jefferies & Co. Overview | Jefferies & Co. Site
  • Blackstone | Blackstone Overview | Blackstone Site
  • Lazard | Lazard Overview | Lazard Site
  • Qatalyst Partners | Qatalyst Partners Overview | Qatalyst Partners Site
  • Evercore | Evercore Overview | Evercore Site
  • Houlihan Lokey | Houlihan Lokey Overview | Houlihan Lokey Site
  • Centerview | Centerview Overview | Centerview Site

Additional interview resources

To learn more about interviews and the questions asked, please check out the additional interview resources below:

  • Private Equity Interview Questions and Answers
  • Hedge Funds Interview Questions and Answers
  • Finance Interview Questions and Answers
  • Accounting Interview Questions and Answers
  • FP&A Interview Questions and Answers

investment banking case study interview questions

Get instant access to lessons taught by experienced private equity pros and bulge bracket investment bankers including financial statement modeling, DCF, M&A, LBO, Comps and Excel Modeling.

or Want to Sign up with your social account?

Investment Banking Case Study Interviews

Best Investment Banking Interview Prep Courses

What are investment banking case studies, why do investment banks ask case study questions, how to tackle a case study question, how to prepare for investment banking case studies, tips to help you impress, investment banking case study interviews.

Updated October 11, 2023

Edward Melett

All products and services featured are independently selected by WikiJob. When you register or purchase through links on this page, we may earn a commission.

1. Investment Banking Interview Skills - Financial Edge

Financial Edge is the firm trusted by the world’s top 4 Investment Banks (as rated by the FT) to train their new analysts. Founded by ex-bankers and a team of experienced instructors, they deliver technical training programs in-house and online through their range of Wall Street-recognized certified courses.

The course takes about 36 minutes to complete with topics like what an investment banker does, the role of an investment banker and preparing for the recruitment process.

Price: £32.50

Visit Investment Banking Interview Skills - Financial Edge

2. The Complete Investment Banking Course 2023 - Udemy

The Complete Investment Banking Course 2023 by Udemy is a comprehensive online course that teaches the fundamentals of investment banking. The course covers topics such as financial modeling, valuation, mergers and acquisitions, how to pass investment banking interviews, and other core skills required for a career in investment banking. The course is designed by industry experts and includes practical exercises, case studies, and quizzes to help learners fully understand the concepts. By taking this course, learners can gain the skills and knowledge necessary to pursue a career in investment banking.

This course includes 11.5 hours of video and 172 downloadable resources.

Price: $13.74

Visit The Complete Investment Banking Course 2023 - Udemy

3. The Ultimate Guide to the Technical Finance Interview - Wall Street Prep

The Ultimate Guide to the Technical Finance Interview by Wall Street Prep is a comprehensive resource for those looking to prepare for a finance interview. It covers a wide range of topics, including financial statement analysis, valuation techniques, and corporate finance. The guide provides in-depth explanations, step-by-step instructions, and real-world examples to help readers understand these complex concepts. It also includes practice problems and interview questions to help readers hone their skills and prepare for the interview process. Overall, The Ultimate Guide to the Technical Finance Interview is an invaluable resource for anyone seeking to succeed in a finance interview.

It is a self-study guide with over 275 pdf pages, including 1,000 investment banking interview questions and answers.

Visit The Ultimate Guide to the Technical Finance Interview - Wall Street Prep

When going through the application process for a graduate role at an investment bank, you will likely be faced with a case study interview . Investment banking roles are highly competitive and you must be properly prepared.

This article, along with our comprehensive article on case study interviews , will ensure you know what's coming and can prepare with confidence.

Investment Banking Case Study Interviews

Investment banking case studies are commonly used to assess how a job candidate would perform in a real situation, by presenting them with a theoretical scenario similar to those encountered working in the role.

You will normally encounter case studies at the final stage of the application process, likely during an assessment centre or final-stage interview.

Most investment banking case study questions centre on  acquisition , raising capital or company expansion. The case may be given to you ‘blind’ on the day of your assessment centre with only a short amount of time provided for preparation.

If the case is likely to involve deep analysis, financial modelling or company valuations, you will likely be given the case in advance, to give you more time to work on it before the assessment day.

In some cases, the investment banking case study may be presented as a group task . This is an opportunity for the employer to test how candidates work in teams, as well as their analytical skills .

While  general interview questions can give a surface impression of a candidate’s skills and suitability, case study questions enable interviewers to assess the candidate’s ability to approach a multi-layered client problem . 

Case study questions are not only testing a candidate’s analytical skills but also their ability to reason and communicate. They are useful for assessing how candidates tackle complex problems, make key judgements and present their recommendations.

With many case study questions, there will be more than one way to successfully approach the problem and more than one possible solution. Investment banks are looking for decisive candidates who can present their solution clearly and logically, and defend their decisions under scrutiny.

These questions are not designed to test in-depth sector knowledge but are an opportunity to display the commercial awareness investment banks seek. 

Facing a case study exercise can be daunting. Here are some key tips to help you get started:

  • Read through the scenario carefully – at least twice – before beginning to form an opinion on how the problem should be tackled. This will ensure that no intricacies are missed and your response addresses all facets of the case.
  • Review the information presented and make informed and necessary assumptions where needed.
  • Take time over your analysis:  consider all possible courses of action before settling upon your recommendations.
  • If financial information is provided, take note of this carefully. Do not shy away from performing calculations to support your recommendations.

Candidates will likely be asked to give a presentation of their findings at the end of the task.

When preparing your presentation, make sure you outline the path you have taken clearly. It is important to display why the choice was made to discard other courses of action in favour of the selected recommendation.

The case study questions are not necessarily about finding a correct answer, but rather the thought process and analytical skills used to tackle a problem.

What to Expect During a Case Study Interview

Case studies for investment banking interviews come in two forms:

  • Take-home case studies
  • Blind or on-the-spot case studies

Take-Home Case Studies

Take-home case studies are usually given to candidates up to a week before they are scheduled to attend an interview or assessment day .

You will be expected to work on it at home and then present your analysis and recommendations in a 30 to 45-minute presentation when attending your interview. The level of analysis must reflect the amount of time allowed to complete the task.

Due to the extra time allowed, a take-home case study will usually be a financial modelling case study which will require more in-depth research. Financial modelling and simple valuation will be required.

This may be preparing a straightforward merger, presenting the process for a Leveraged Buy Out (LBO) or performing an FCFF (free cash flow to firm) valuation.

Candidates will need to be confident with valuation multiples; for example, looking at equity and enterprise value to determine accurate, over- or under-valuation.

For more help and advice on investment banking valuations, see our article on Company Valuation Interview Questions .

Blind Case Studies

Blind case studies will be given to the candidate on the day of an assessment centre or later-stage interview.

During a blind case study assessment, a candidate will normally face a decision-making case study, which will not require in-depth research.

These may focus upon whether a particular merger should take place, how to achieve the best valuation or identifying sources through which capital could be raised. These questions are looking more at a candidate’s problem-solving skills and commercial awareness , rather than deep analysis.

Candidates will be given around one hour to complete their analysis and approximately 10 minutes to present. They should be prepared for a substantial round of questions after completing their presentation.

Investment Banking Case Study Interviews

Regardless of which investment banking case study type a candidate is presented with, the thought process and deliverables are the same. The best way to prepare for an investment banking case study is to practise completing similar questions .

Investment banks do not tend to publish case study questions for practice. However, it is possible to formulate your own questions by looking at business scenarios involving possible mergers, necessary valuations or raising of capital.

As mentioned above, candidates will need to be confident with valuation skills . They will also need to display a good level of commercial awareness . Presentation skills and successful team working also need to be considered.

Before your case study interview, keep up-to-date with developments in the investment banking sector, both relating to company conduct and external impacts. This background knowledge is key when demonstrating your knowledge and enthusiasm for investment banking.

Thinking about work experience completed and even any extra-curricular activities from a commercial perspective is also a good way to start to interrogate different types of client relationship and look at the common factors for success.

Candidates should ensure that they can display a full understanding of key business concepts and can use the associated terminology comfortably in a conversation. Make sure you can confidently discuss the following:

  • Profit, revenue, and fixed and variable costs in a business;
  • Client, stakeholder and competitor landscapes;
  • How external/internal factors can influence these and the financial dynamics that rely upon them.

Candidates must practise streamlining their thought process so judgements can be made under time pressure if required.

Candidates should also practise presenting their process of analysis clearly and justifying the recommendations they have made, to prepare for the scrutiny of the interviewer.

Example Case Study Questions and Answers

“A client owns her business (worth £400m) outright and is looking to release some liquidity while retaining a percentage of ownership. What recommendations would you make to the client to achieve maximum valuation?”

Accompanying information is likely to be provided with a question of this kind, such as a company overview and data on the company’s performance over the last three years. Go through any financial data carefully and make predictions as to what organic growth would look like for the company.

If you are given the relevant information, consider the breakdown of the current valuation.

Consider the industry of the client and predicted sector trends:

  • How does the valuation compare within the industry?
  • Is the current valuation based upon solid industry predictions?
  • Would it be wise to give up more equity based on stagnant industry growth?
  • Is the sector growing and predicted to continue to do so?

Think about whether any steps could be taken to make the company of increased appeal – consider current client portfolios and projects, etc.

Consider how you would advise the client to negotiate:

  • What percentage of equity should they be willing to give up?
  • What figure would you advise the client to settle on as their effective reserve price?

“A public company approaches you as they wish to raise capital. Recent earnings and the quarterly report have been reviewed and found to be in line with analyst predictions but the company is currently trading at an all-year low. The company’s management has designed a project it believes will substantially increase its EBITDA and wishes to raise the capital to advance. How should the company go about raising the capital necessary?”

For this question, you would be given an overview of the company and their financial statements to review.

You must consider whether the company should raise debt or equity.

Consider the share price, volume of shares outstanding and the market cap:

  • What impact would issuing new shares have on the company in this landscape?
  • Would equity financing be a sustainable option regarding ownership dilution?
  • How would the impact now differ from that if the share price were back at ‘normal’?

Look over the financial statements provided:

  • If these are indeed in line with management predictions, would raising debt be a better option?
  • How much might they raise?
  • What future issues may raising debt cause?
  • How might interest expense be mitigated?

Organise your ideas clearly for your presentation, working through your thought process and the questions you posed to reach your recommendation. Around five PowerPoint slides should be ample.

“A renewables company is looking to make a significant acquisition. You have been asked to review the deal and make your recommendations.”

You will be given an overview and the financial information for both the buying company and the potential acquisition. You will also be provided with information relating to other companies similar to the proposed acquisition.

You will need to decide if the target company should be purchased and, if so, at what price.

You will be expected to use the data provided from comparable companies and apply multiples to the target company to settle upon a purchasing price.

Consider the financial situation of the purchasing company:

  • If required, can they raise the necessary capital through raising debt or equity, or do they already have the funds necessary?

Consider the benefits of acquiring the target company:

  • What will the company contribute in terms of profit and revenue?
  • What other benefits may the acquisition bring the buyer?

Think about how any risks to the buyer can be mitigated.

Present your strategic recommendation clearly and concisely.

Step 1 . Prepare

Practice completing the type of case study questions you are likely to be presented with. Read about examples of mergers, acquisitions and expansions in the news. Get comfortable with all relevant terminology and financial concepts.

Step 2 . Have conviction

Make a solid recommendation backed up with detailed reasoning. Stand by your choices and explain clearly why you believe the course of action to be preferable.

Step 3 . Remain calm

There may not immediately be an obvious solution. Take time to think the problem through with the information given, making necessary assumptions.

Step 4 . Be logical

Work through the problem carefully and strategically. Present your ideas clearly and with justification. Case study questions are intended to challenge candidates to display their thought process.

Step 5 . Teamwork

If the case study task is a group exercise, make sure all members have the opportunity to contribute and present. Performing well yourself, while facilitating others to perform at their best, displays a good awareness of people management skills.

Step 6 . Confidence

Work on your presentation skills so you can convey your ideas decisively and with confidence. A detailed and meticulous analysis won’t count for much unless it can be communicated effectively.

You might also be interested in these other Wikijob articles:

Investment Banking Case Study Interviews

Or explore the Industry / Investment Banking sections.

Logo for Growth Equity Interview Guide

The Complete Guide to Investment Banking Interview Questions

Picture of Mike Hinckley

Investment banking is an incredibly competitive field to interview for and work in. 

Whether you’re aiming for a small boutique firm or a bulge-bracket firm, you’ll be competing for a position with dozens, or hundreds, of other possible applicants. In your investment banking interviews, your work ethic, hustle, and technical knowledge will be what sets you apart – and your passion for the work and people skills will be what sells the firm on YOU. 

Most of the below investment banking interview questions are directed at candidates entering investment banking at the pre-MBA level. If you are entering investment banking after a few years working in another finance sector or post-MBA, you can expect a bit more technical questions than I cover here. 

The first category of questions you can expect are basic “fit” questions. 

Investment Banking Fit Questions

These questions are your generic interview questions, but they give you a really good chance to sell yourself. This is where your “soft” resume skills can show up well: work ethic, public speaking, and good old-fashioned hustle.  

Walk me through your resume.

This is not supposed to be your full life history – your interviewer has already looked over your resume and knows what’s there. Keep it brief, and pull out specific items of interest. 

If you’ve had some real finance experience – e.g. if you managed your finance club’s practice investment fund – talk about your results, the most interesting thing you learned, or some other important bit. 

If the interviewer asks specific questions about resume items, feel free to go into more detail. If you have experience listed that isn’t specifically related to investment banking, be prepared to tie it to the specific job you’re interviewing for. For example, if you were a club or professional fraternity president, or a sports team captain, highlight the leadership experience you’ve gained and how it could help you on the job. 

If you’ve done your research about the firm (and even the specific interviewer if you’re lucky) you can use this question to highlight specific things about your resume that would impress that person specifically. If you looked up your recruiter on LinkedIn before the interview, you might see items in their CV that can tie to yours – maybe they were in the same fraternity, or you both have a specific interest in international banking. Only do this if it’s genuine; it’s not something that can be forced. 

Don’t be afraid to conduct this type of research ahead of time. To some degree, they’re doing it to you – you should feel free to find out more about them too. Just don’t divulge so much that you come across as creepy. 

Why should we pick you?

Investment banking is so competitive that you have nothing to lose by giving a positive, confident pitch about yourself. 

In your preparation, you should ask yourself “What (actually) sets you apart? What makes you unique?” 

Create a one- or two-sentence description of yourself. Your tagline, your slogan, your elevator pitch; whatever you want to call it, it’s a powerful description of how you’re ready to bring value to the world. 

And remember that the biggest thing they are looking for is work ethic and hustle so make sure you show in your pitch how passionate and excited you are for this job.

Why investment banking? 

Recruiters want to know you’re serious about investment banking. Your answer to this question reveals a lot about why you’re sitting in that chair in the recruiter’s office. 

Are you looking for an interesting, challenging career? Are you ready to work long hours?

Do you want a job with prestige and a big paycheck? (Hint: even if you do, don’t talk about that during your interviews).

If you’re going into investment banking because you hope to springboard to a different area of finance in a few years, like VC, growth equity, or private equity, that’s fine. It’s a very popular path for young finance professionals. However, the company you’re interviewing for now doesn’t want to know about your future plans to leave them! So don’t discuss this in your interviews. 

Their main concern is that you will be a reliable, hardworking, creative employee – and stay long enough that you can be an asset to the company. 

If you have a specific interest or life experience that has drawn you to investment banking as a career, feel free to give a more personal reason why it’s important to you – but again, it’s not the time for a long story. 

For a deeper dive on why investment banking , check out this article.

What kind of work environment do you do well in?

Everyone is different, and every bank is different (or wants to think they are). 

Interviewers are looking for candidates who thrive in a fast-paced, ambitious, and competitive environment. In your answer, make sure to back it up with examples from your past experience.

What are your hobbies?

This is a good opportunity to let your personal side show a bit. 

Whether you like mountain climbing, oil painting, or something else, this is a nice way to show your employer that you are a well-rounded person with unique interests outside of work. 

If your favorite hobby is truly reading the Wall Street Journal and keeping up with the stock market, absolutely say so – but you don’t have to pretend it is just for the sake of the job. 

Keep the answer brief – this answer is likely not going to get you the job.  So, answer succinctly and let them see a new side of you.  Then move on.

Why This Bank?

To answer this question, do your research before the interview. 

Show your interviewer that you’ve researched all the external facing information about the bank.  For instance, what does the website say about the bank’s strategy?  What did the CEO talk about on the recent earnings call?  

This is also a good opportunity to show off any networking you’ve done. If you attended a campus event or spoke to a recruiter before interviewing, this is a good time to drop names and mention what you learned about the company. “I spoke to X at the Y event, and he told me about this company’s role in Z. I’m especially interested in Z, and am looking forward to working on Z- related projects in the next few years.” 

If you have specific social ethics or values, you can mention how they line up with the company’s CSR or ESG goals – but this shouldn’t necessarily be the main reason you chose the firm.  But don’t go overboard here, unless you’re specifically interviewing for an ESG role.

Investment Banking Behavioral Questions

I’m not going to go into a lot of detail on these. They’re pretty standard interview questions. Be prepared to answer them, but put most of your preparation into the technical and firm-specific questions.

  • What’s the thing you’re most proud of?
  • What is your biggest strength?
  • What is your biggest weakness and how do you deal with it?
  • How do you deal with stress?
  • Tell me about a time you had to work extremely hard.

Investment Banking Technical Interview Questions 

This is the real meat of an investment banking interview. Knowing the principles of finance and investing inside out will go further towards distinguishing you from other candidates than any discussion of your personal preferences.  

What is the difference between equity value and enterprise value?

Equity value is a component of enterprise value.

The enterprise value is the full value of any business.  Because many businesses have debt, part of this value will be attributable to NET DEBT, while the rest will be attributed to EQUITY.

Enterprise value = Equity + Net Debt

Investors who are interested in valuing or acquiring entire businesses are generally interested in enterprise value. If you are going into private equity where leverage is heavily emphasized as an acquisition strategy, you will certainly rely on  enterprise valuation. 

Think of it like the value of a home.  The enterprise value is the price you sell your home for – let’s say $1 million.  However, because you have a mortgage on the home with an outstanding balance of $300,000, then you only have $700,000 in equity value.  This means when you sell for $1 million you would only get $700,000 in net proceeds (or equity) when you sell.

Same idea as enterprise value vs. equity value.

Screenshot of course preview

  • 6+ video hours
  • Expert instructor

ONLINE COURSE

Assess Business Models Like An Investor

Designed for aspiring private equity, growth, or venture investors

What is EBITDA, and why are EBITDA multiples sometimes preferred to P/E ratios?

EBITDA is earnings before interest, tax, depreciation, and amortization. 

A company valuation based on EBITDA often reflects the true value of a company better than a P/E ratio for one important reason: it excludes capital structure and non-core factors from the company’s earnings. 

Since Net Income includes so many one-off things that may not be reflective of the ongoing business activities, it can skew the company valuation’s valuation, which should only reflect going forward activities. 

What are the main valuation methods for companies?

A few of the primary methods for companies are discounted cash flow (DCF), comparables or multiple analysis, and asset-based valuation. 

Discounted cash flow is very common among investment bankers, as it gives the theoretical value of any company.  It essentially calculates the value of the business by projecting forward its cash flows and then discounting them back to today’s date, using a discount rate.  The downside of this method is that it relies on several assumptions and can be too theoretical to use in practice.  

Comparable analysis compares the sale price or market valuation of the company in question to other similar companies in the market. There are two kinds of comparables – “trading” comps, which compares public companies, and “acquisition” comps, which compares M&A deals.  The companies have to be similar in terms of industry, size and yearly earnings to be comparable in value. This valuation method is extremely common since it shows the actual market prices of real companies in the market, rather than some theoretical value as in the case of DCF.

Another form of valuation is called “replacement value.” If a company is being sold, one way to value it is by calculating the cost required to completely rebuild or duplicate the company.  If a company wants to sell for $400 million, but buyers know they can rebuild its assets for just $50 million, this may constrain the valuation the company can achieve in its sale process.

Which has a higher cost of capital: debt or equity? Why?

Equity financing has a higher cost of capital than debt financing. 

This is because the providers of the financing take on a greater risk in providing equity than they would in providing debt. They will demand a higher return on their money to compensate for the increased risk. 

Companies have a legal requirement to pay back the debt they owe before the equity, and they must make debt payments before they pay dividends to shareholders. This offers a few benefits. First, it decreases the risk that they will default on the debt, creating the possibility of a lower cost of capital. Second, the interest the company will pay on that debt is usually tax-deductible, reducing their cost of capital by their corporate tax rate. 

Equity financing has none of these benefits. The company has no legal obligation to pay shareholders. This alone creates more risk for the shareholders, and they will demand a higher return in exchange. Payments to shareholders are also not tax-deductible as interest on debt financing is. Because of this, even equity financing that provides the same rate of return as debt financing will always be more expensive to the company because of the tax rate. 

Explain what net working capital is, and how it impacts cash flow.

Net working capital is the difference between a company’s current assets (excluding cash) and current liabilities (excluding current portion of long-term debt). 

It shows two important things: 

First, it shows how easily a company would be able to meet their current liabilities with their current assets, should they need to in an emergency. 

Second, working capital can be a significant source (or use) of cash for the business in ongoing operations. Changes to the net working capital will increase or decrease the unlevered free cash flow of the firm.  

Walk Me Through a DCF

This is another technical question, and one that requires slightly more preparation than the others. 

The key here is preparation – the interviewer wants to see not only that you have the technical knowledge, but that you are mentally organized enough to discuss it fluently and intelligently. 

To answer this question in an interview, follow a basic outline in your head that you can flesh out as you go. Something like this: 

Define DCF: A DCF, or discounted cash flow model, shows the value of future cash flows from a company over a period of time, adjusted by a discount rate back to the present time period. 

State the purpose of a DCF: The end goal of a DCF is to determine the theoretical enterprise value of a company . This gives potential investors a realistic idea of the value in the company independent of market forces. This is a solid anchor for valuation and pricing estimates. 

Explain how a DCF works: Before you can find the discount part of a DCF, you have to find the cash flow part. You will need to project forward a cash flow statement to find free cash flow for the end of each period – usually five years. To do this, you will need the EBITDA and other financial metrics affecting cash besides EBITDA to get down to Free Cash Flow for each projected year (e.g. interest, working capital, taxes, capital expenditures, etc.). .

Once you have the ending free cash flow for each period, you need to calculate the discount rate in order to find the present value of each year’s cash flow. The discount rate is calculated by finding the WACC (weighted average cost of capital) for the company. This is a weighted average of the returns that debt and equity investors can expect from the deal. If the company is highly leveraged, the WACC will generally be lower than if it is financed with larger amounts of equity. 

Now that you have the WACC and the free cash flow for each year, you then have to discount each year’s cash flow separately back to the present day, based on the WACC and the time period for each cash amount. 

Finally, you’ll need to calculate the “terminal value” of the enterprise.  This is the value of all cash flows that occur outside of the projection period (e.g. if the projection period is 5 years, then the terminal value is for year 6 and beyond).  You can calculate this value either through a perpetual growth rate or by applying a valuation multiple to the FCF in that year.  Likewise, you will then apply the discount rate to see the present day valuation of the terminal value.

Once you have all the present-day equivalents of the cash flows, you can add them all together to obtain an approximate company valuation. 

Discuss pros and cons of a DCF:

  • Pros: Assuming that the WACC and assumed growth rate are accurate, the DCF is one of the best ways to obtain a company valuation. It eliminates the impact of prevaling market forces.
  • Cons: The Con is the same as the Pro:  Market forces.  While the DCF eliminates the impact of market forces outside the company’s control, these forces CAN and DO easily change the valuation of companies overnight in the real world.  Also, the DCF relies on several assumptions (WACC, discount rate, etc.) that are theoretical and hard to justify. 

How are the three main financial statements linked?

Like the last question, this is one that is best answered with a basic framework that you can build off of. 

Describe the three main financial statements: 

  • Income statement: shows all revenue and expenses for the past fiscal period
  • Balance sheet: shows all assets, liabilities, and equity at the end of the period
  • Cash flow statement: translates the accrual accounting basis to actual cash on hand for the period

Describe how they impact each other: 

The first statement you have to compile is the income statement. This shows all the revenue earned and expenses incurred.

The balance sheet , of course, is impacted by the income statement because net income (or loss) from the period flows through to retained earnings in the equity section of the balance sheet, where it can be distributed to owners and shareholders.  

There are other income statement items that impact other balance items. A prominent example is depreciation & amortization and how it impacts the balance of PP&E on the balance sheet.

Finally, the cash flow statement pulls from both the income statement and the balance sheet.  The cash flow statement ultimately measures the change in cash flow for the business.  The change in cash is ultimately quite different from the income it earns, but this statement helps you track the differences.  This statement starts with net income from the income statement, and then it will detail all non-cash items that are counted within net income (e.g. operating activities) or need to be included (e.g. investing or financing activities).  Examples include capital expenditures from the balance sheet, and depreciation from the income statement. 

Then, the change in cash flow measured on the cash flow statement is ultimately reflected back on the balance sheet!

To cross-check the ending cash balance on the balance sheet, you can check that the ending cash balance from the last period plus the current period’s cash from operations, investing, and financing (cash flow statement) are the same. 

Investment Banking Case Study

Your investment banking interview may include a case study. While this is more common in sectors like private equity or growth equity , you may also see it here, especially at interviews above entry-level positions. 

In general, though, investment bankers prefer to test your knowledge with technical questions and follow-ups like the information in this article, rather than actual assignments. 

Private equity interviews may include a detailed take-home case study or in-person valuation model. 

The most common case study you’ll see in an investment banking interview process would be a very light DCF model with little complexity. This is something you might only see at a very small boutique firm, and won’t see at all at larger firms or bulge-bracket banks – they simply don’t have the time or capacity to review case studies on a large scale. 

If you’re going into investment banking as a step to a future career in private or growth equity, the case study is a good thing to remember for the future, but you most likely won’t need to deal with it now. 

Questions To Ask Your Investment Banking Interviewer

This is a good time to ask any questions you may have about company culture or specific answers to questions you may have thought of that you couldn’t find on the website. 

If you did some networking before the interview, you can mention any questions you didn’t get to ask in your networking conversations. “I really enjoyed speaking with X about the work this bank is doing with Y. I’d love to know more about that project.” 

This isn’t the time to appear self-interested in working hours or salary. Think of it as the flip side of the interviewer’s “why should we hire you?” question. Now you get to ask questions to see if you are a good personality and work style fit for the company. You can also ask your interviewer more personal questions.

“What is your favorite part about working for X?” 

 “What were your first couple years working for X bank like? Is that a similar experience to what new hires now can expect?”

Preparing for a big career interview like investment banking can be a bit nerve-wracking if you haven’t done many like it before. Don’t worry – you’ll get over it quickly. Practice in front of a mirror, or record yourself giving confident answers. Investment banking is a rewarding, lucrative career, and you’re ready to do what it takes to get you there.

Preparing for an investment banking interview requires a thorough understanding of the various types of questions that may be asked, such as fit, behavioral, and technical inquiries. 

It is critical to conduct research on the bank you are interviewing with and be able to articulate why you want to work for them.

Technical questions, such as walking through a DCF and understanding the relationship between financial statements, must also be mastered. Furthermore, practicing investment banking case studies and preparing thoughtful interview questions can help you stand out and make a good impression.

You will be better prepared to succeed in your investment banking interview (including your HireVue interview ) and land your dream job if you follow this guide.

  • Articles in Guide
  • More Guides

DIVE DEEPER

The #1 online course for growth investing interviews.

Screenshot of course preview

  • Step-by-step video lessons
  • Self-paced with immediate access
  • Case studies with Excel examples
  • Taught by industry expert

Get My Best Tips on Growth Equity Recruiting

Just great content, no spam ever, unsubscribe at any time

Copyright © Growth Equity Interview Guide 2023

[email protected]

HQ in San Francisco, CA

Phone: +1 (‪415) 236-3974

Growth Equity Industry & Career Primer

Growth Equity Interview Prep

How To Get Into Private Equity

Private Equity Industry Primer

Growth Equity Case Studies

SaaS Metrics Deep Dive

Investment Banking Industry Primer

How To Get Into Investment Banking

How To Get Into Venture Capital

Books for Finance & Startup Careers

Growth Equity Jobs & Internships

Mike Hinckley

Growth stage expertise.

Coached and assisted hundreds of candidates recruiting for growth equity & VC

General Atlantic logo

with Mike Hinckley

Online course

  • Business model breakdowns
  • Practical mental models & frameworks
  • Aspiring PE, growth, or VC investors

Register for Waitlist

Company logo

FREE RESOURCES

Get My Best Growth Equity Interview Tips

No spam ever, unsubscribe anytime

Username or Email Address

Remember Me

Company logo

The 400 Investment Banking Interview Questions

Crack the code to investment banking success with our comprehensive guide to the top 400 Investment Banking Interview Questions. Elevate your preparation with expert insights into technical know-how, financial modeling, and behavioral scenarios. Ace your interview with confidence, armed with a thorough understanding of the questions that matter. Explore our meticulously curated list covering all aspects of investment banking, ensuring you're ready to impress recruiters and secure your dream job. Your journey to acing the interview and securing a lucrative career in investment banking begins with mastering these essential questions.

Securing a job in the competitive field of investment banking can be a challenging task. Apart from having the necessary qualifications and experience, acing the job interview is crucial to stand out from the crowd. Interviewers often ask a wide range of questions to assess a candidate’s knowledge, skills, and fit for the role. In this article, we will explore the 400 investment banking interview questions and provide you with valuable tips to help you prepare and succeed in your job interview.

Preparing for Your Investment Banking Interview: Tips and Best Practices

  • Research the company: Familiarize yourself with the company’s history, recent deals, and market position. This will demonstrate your interest and commitment during the interview.
  • Practice technical questions: Review financial concepts, valuation methods, and accounting principles. Practice solving case studies and be prepared to explain your reasoning.
  • Develop your story: Prepare concise and compelling answers to common interview questions, highlighting your relevant experience, skills, and achievements.
  • Enhance your soft skills: Work on improving your communication, teamwork, and leadership abilities. Practice articulating your thoughts clearly and concisely.
  • Stay up-to-date with industry trends: Read financial news, follow market trends, and stay informed about current events that may impact the investment banking industry.
  • Prepare questions to ask: Have thoughtful questions prepared to ask the interviewer about the role, team dynamics, and the company’s future plans. This shows your interest and engagement.

Understanding the Investment Banking Interview Process

The investment banking interview process typically consists of multiple rounds, including phone screenings, technical interviews, behavioral interviews, and case study interviews. Each round aims to evaluate different aspects of a candidate’s abilities and suitability for the role. It is essential to be well-prepared and familiarize yourself with the types of questions that may be asked in each round. Let’s delve into the various types of questions you may encounter:

Technical Questions: Assessing Your Financial Knowledge

Technical questions form a significant part of the investment banking interview process. These questions aim to assess your understanding of financial concepts, valuation techniques, accounting principles, and market trends. Here are some common technical questions you may come across:

1. What is the difference between equity and debt financing?

Equity financing involves raising capital by issuing shares of stock, giving investors ownership of the company. Debt financing, on the other hand, involves borrowing money from lenders and repaying it with interest over a specific period.

2. Can you explain the concept of the time value of money?

The time value of money refers to the idea that money available today is worth more than the same amount in the future due to its potential earning capacity. This concept is crucial in evaluating investment opportunities and calculating present and future values.

3. How do you calculate enterprise value?

Enterprise value is calculated by adding a company’s market capitalization to its total debt and subtracting its cash and cash equivalents. This measure provides a more accurate representation of a company’s value, considering its debt and cash holdings.

4. What are the different valuation methods for companies?

There are several valuation methods used in investment banking, including discounted cash flow (DCF), comparable company analysis, precedent transactions, and leveraged buyout (LBO) analysis. Each method has its own advantages and considerations, depending on the specific context.

5. How does an initial public offering (IPO) work?

An IPO occurs when a private company offers its shares to the public for the first time. This process involves underwriters, who help determine the offering price and facilitate the sale of the shares. IPOs provide companies with an opportunity to raise capital and become publicly traded.

Behavioral Questions: Assessing Your Soft Skills

While technical knowledge is essential, investment banking roles also require strong interpersonal skills and the ability to work effectively in a team. Behavioral questions aim to assess your soft skills, such as communication, leadership, problem-solving, and adaptability. Here are some common behavioral questions you may encounter:

1. Tell me about a time when you had to work under pressure and meet tight deadlines.

This question assesses your ability to handle stress and deliver results in challenging situations. Provide a specific example from your past experience and explain how you managed your time, prioritized tasks, and achieved the desired outcome.

2. Describe a situation where you faced a conflict within a team and how you resolved it.

This question evaluates your conflict resolution skills and your ability to work collaboratively. Share an example where you successfully managed a conflict by listening to different perspectives, finding common ground, and fostering a positive outcome.

3. How do you handle feedback and criticism?

This question examines your ability to receive feedback constructively and make improvements. Discuss your approach to feedback, such as actively listening, reflecting on the feedback, and taking appropriate actions to grow and develop.

4. Give an example of a time when you demonstrated leadership skills.

This question assesses your leadership abilities and your potential to take charge in challenging situations. Share an example where you took the lead, motivated others, and achieved a positive outcome through effective decision-making and communication.

5. How do you prioritize multiple tasks with competing deadlines?

This question evaluates your organizational and time management skills. Provide an example where you effectively managed multiple tasks by setting priorities, delegating when necessary, and delivering high-quality results within the given deadlines.

Case Study Questions: Assessing Your Problem-Solving Abilities

Case study questions are commonly used in investment banking interviews to evaluate a candidate’s problem-solving abilities and analytical thinking. These questions typically present a hypothetical scenario or financial problem that requires critical analysis and creative solutions. Here are some examples:

1. You are advising a client who is considering acquiring a company. How would you assess the financial viability and potential risks of the acquisition?

This question tests your ability to analyze financial statements, conduct due diligence, and evaluate the strategic implications of an acquisition. Explain the steps you would take, such as analyzing the target company’s financial performance, evaluating synergies, and assessing industry trends.

2. How would you value a start-up company with a limited financial history?

This question evaluates your ability to assess the value of a company based on limited financial data. Discuss the alternative methods you would consider, such as market research, benchmarking against similar companies, and assessing the growth potential and competitive landscape of the industry.

3. You are presented with a declining company. What strategies would you recommend to turn it around?

This question assesses your ability to identify and propose solutions to improve the performance of a struggling company. Discuss potential strategies such as cost-cutting measures, diversifying revenue streams, restructuring operations, or exploring new markets.

4. How would you analyze and value a real estate investment opportunity?

This question tests your ability to evaluate real estate investments based on market conditions, financial projections, and risk assessment. Explain the factors you would consider, such as location, rental income potential, financing options, and market trends.

5. You are advising a client on investing in a foreign market. What factors would you consider and how would you assess the risks?

This question evaluates your understanding of international markets and your ability to assess the risks associated with foreign investments. Discuss factors such as political stability, economic indicators, exchange rate fluctuations, and regulatory considerations.

Acing Your Investment Banking Interview

An investment banking interview can be a daunting experience, but with thorough preparation and practice, you can increase your chances of success. Remember to stay calm, confident, and authentic during the interview. Use the 400 investment banking interview questions as a resource to enhance your knowledge and prepare effective responses. Good luck!

Related Posts:

  • Aritzia Interview Questions: How to Prepare and Succeed When it comes to preparing for a job interview, it's important to do your research and anticipate the questions that may come your way. This…
  • Top Dishwasher Interview Questions: Ace Your Next Dishwasher Job Interview Are you preparing for a dishwasher job interview? Whether you're a seasoned dishwasher or just starting out in the industry, it's important to be well-prepared…
  • Peloton Interview Questions: Tips and Advice for Success Are you preparing for an interview with Peloton, the popular fitness company known for its innovative exercise equipment and virtual workout classes? Landing a job…
  • Common Interview Questions: What to Expect and How to Answer Them Job interviews can be nerve-wracking experiences. Whether you're a recent graduate or a seasoned professional, the pressure of impressing a potential employer can be overwhelming.…
  • Cracking the Interview: A Comprehensive Guide to FTI Consulting Interview Questions Preparing for a job interview can be a nerve-wracking experience, especially when it comes to prestigious consulting firms like FTI Consulting. As one of the…

Leave a Comment Cancel reply

Save my name, email, and website in this browser for the next time I comment.

Show Menu

  • Video Tutorials
  • Knowledge Base
  • Group Licenses
  • Why Choose Us?
  • Certificates

User Avatar

Investment Banking Interview Guide 4.0

Smiling Banker

Here’s How to Gain an ‘Unfair Advantage’ Over Other Candidates in Investment Banking Interviews

– Without Memorizing Hundreds of Repetitive Questions and Answers

Pitch yourself like a pro

Use our templates to answer the “Walk me through your resume/CV” question

Ace the technical questions

Learn the concepts so you don’t have to “memorize” anything

Prepare efficiently for "fit" questions

Re-use the same few stories to answer any fit/behavioral question

Test yourself and practice for case studies

You’ll get 17+ case studies and 218+ quiz questions with full answer keys

View short course outline  or scroll down for the details

Join Our Community Join the 56,763+ students and professionals who have already used our training to win interviews and job offers Prepare Quickly Get study plans for 4 hours, 2 days, and 1 week, plus an "interview checklist" with the key points you need to know Get Expert Support Unlimited access to course files, 5 years of support/updates/video access, and a 90-day money-back guarantee Test Yourself Complete global case studies, answer quiz questions, and get feedback on your "story" and other interview answers

Gain an Advantage by Quickly Conquering the Two Topics that Matter the Most in IB Interviews:

When you cut away all the fluff, two topics matter more than anything else in investment banking interviews:

1 Your answer to the “Tell me about yourself” or “Walk me through your resume/CV” or “Why are you here?” question.

In my 10+ years of conducting mock interviews, I’ve never once heard a good initial answer to this question.

That’s why it’s so tough to win offers in investment banking interviews – answer that one question wrong in the first 2-3 minutes, and you’re out!

But if you use our templates, and you outline your story and get feedback directly from us , you instantly jump ahead of everyone else in the interview room.

2 Your ability to answer technical questions competently and prove that you can do the work.

Technical knowledge is required to win offers. If you can’t explain the financial statements, valuation, or DCF analysis, you’ve eliminated yourself before the interview even begins.

It’s not possible to learn everything at the last minute because there is a lot of material – but you can focus on the most important concepts and use the quizzes and Excel files in this Interview Guide to make sure you know the ropes.

Our Interview Guide is designed around these two critical topics .

Yes, there’s lot of other stuff in here – fit/behavioral questions and answers, quizzes, practice case studies, industry-specific questions, deal discussions, and more.

But if you answer questions in these two categories like a pro, the rest is icing on the cake .

Highlights of the Investment Banking Interview Guide 4.0 Include:

investment banking case study interview questions

Take a Look at What You’ll Get Immediately After Signing Up…

investment banking case study interview questions

Module 1: Action Plan and Quick Start Guide

investment banking case study interview questions

This module will show you how to get the most out of the Interview Guide with limited time, and what to focus on if you have only a few hours or a few days to prepare.

You get a Quick Start Guide with 4-hour, 2-day, and 1-week-long study plans, as well as checklists of everything you need to do before and after the interview.

You’ll also learn how the recruiting process works and what you must do to be a competitive candidate.

  • 3 Written Guides: Quick Start Guide, Interview Checklist and What to Do Before and After the Interview, and The Recruiting Process and Differences at the MBA Level and in Other Countries.

Module 2: How to Tell Your Story (Resume Walkthrough Tutorials)

investment banking case study interview questions

In this module, you’ll learn how to answer the most important question in any interview: “Tell me about yourself” or “Walk me through your resume/CV” or “Why are you here today?”

If you answer this question effectively, all the other “fit” questions will be easy because you can keep referencing your ‘story’ to answer those questions.

If you don’t answer this question effectively, the rest of the interview is pointless because the interviewer will stop paying attention after 1-2 minutes.

  • 1 Written Guide: How to Tell Your Story in Interviews.
  • 4 Slide Presentations and 4 Videos: Overall Strategy, Undergrads and Recent Grads, Experienced Professionals and MBA-Level Candidates, and IB Exit Opportunities.
  • 5 Templates and Executed Examples for Undergrads and Recent Grads: Engineer to IB, Liberal Arts to IB, Mixed Finance to IB, Previous Banking Experience to IB, and Consulting to IB.
  • 8 Templates and Executed Examples for MBA-Level Candidates and Career Changers: Big 4 and Accounting to IB, Corporate Finance to IB, Experienced Executive to IB, Law to IB, Back and Middle Office to IB, Military to IB, Equity Research or Sales & Trading to IB, and Entrepreneurship to IB.
  • 5 Templates and Executed Examples for IB Exit Opportunities: IB to Private Equity, IB to Hedge Funds, IB to Venture Capital, IB to Other Groups, and IB to Other Banks.

Module 3: Fit Questions and Deal and Market Discussions

investment banking case study interview questions

In this module of the Interview Guide, you’ll learn how to answer “fit” questions (e.g., your strengths, weaknesses, and leadership skills) and how to discuss deals, companies, and markets, including your own deal experience.

Instead of presenting a laundry list of questions and answers, we focus on the strategies behind answering questions and how to prepare your discussions efficiently. We provide sample answers for the key “fit” questions as well as many examples of transaction discussions – for IPOs and equity deals, M&A deals, and debt deals.

You’ll also learn how to present your work if you have non-investment-banking deal experience, such as client work at a Big 4 firm, law firm, consulting firm, or in a credit risk role.

  • 3 Written Guides: “Fit” Questions and Answers, Deal and Market Discussions, and How to Discuss Your Own Deal Experience.
  • 3 Templates and Executed Examples for Deal and Market Discussions: One initial public offering (IPO) example, one leveraged buyout (LBO) example, and one shipping/maritime market discussion example.
  • 3 Templates and Executed Examples for Discussing Your Own Deal Experience: One sell-side M&A example, one buy-side M&A example, and one capital markets (debt/equity) example.

Module 4: Technical Questions and Answers

investment banking case study interview questions

In this module, you’ll master all the technical concepts.

You’ll start by learning about the core concepts required to understand everything else: The time value of money, present value, net present value, discount rates, WACC, and IRR.

Then, you’ll move into accounting and 3-statement modeling, equity value and enterprise value, and valuation and DCF analysis.

The remaining lessons and guides cover M&A deals and merger models, leveraged buyouts and LBO models, and more specialized topics such as private companies, equity and debt capital markets analysis, and industry-specific technical questions.

Each guide has “key rules of thumb” that explain the concepts, as well as interview questions and answers and accompanying Excel files so you can test yourself.

The core sections alone have 578+ pages of instruction and guidance, along with hundreds of annotated diagrams and screenshots, making this the most comprehensive coverage of technical questions available on the planet.

  • 10+ Written Guides: Core Concepts, Accounting and the Three Financial Statements, Equity Value and Enterprise Value, Valuation and DCF Analysis, M&A and Merger Models, Leveraged Buyouts and LBO Models, Industry-Specific Guides (Real Estate; Oil & Gas; Banks and Insurance; and Restructuring), Equity and Debt Capital Markets and Leveraged Finance, and Private Companies.
  • 18+ Excel Files: These files concisely demonstrate all the mechanics, from the fundamental concepts to their actual application to real-life deals.
  • Short Video Walk-Through for Each Guide: These short tutorials summarize the most important points for each topic and explain how to use the written guides, questions and answers, and Excel files efficiently.

Module 5: Interactive Quizzes

investment banking case study interview questions

This module lets you test your knowledge of the key technical topics by completing quizzes on Core Concepts, Accounting, Equity Value and Enterprise Value, Valuation and DCF Analysis, Merger Models, and LBO Models.

These quizzes are intended to be CHALLENGING – even if you have excellent knowledge of the material, you are unlikely to pass with a score of at least 90% on your first attempt.

Once you’ve tested yourself with these quizzes, you can download the full answer keys to verify that you understand the fundamental concepts.

In total, you’ll get to test yourself with 218+ questions across all the topics.

  • 6 Interactive Quizzes: These cover the core technical topics and expand on the questions and answers covered in the technical sections of the Interview Guide in Module 4. There are many trick questions!

Module 6: Case Study Exercises

investment banking case study interview questions

In this module, you’ll get practice completing a variety of case studies and modeling tests given in interviews and at assessment centers in the EMEA region.

The time required for these case studies ranges from 30 minutes up to 2 hours to simulate the time pressure you will encounter in interviews.

The topics covered include 3-statement modeling, valuation and DCF analysis, M&A and merger models, LBO modeling, and credit analysis. There are both qualitative and quantitative case studies, and some exercises mix the two.

We feature companies and deals from all 6 inhabited continents and a variety of industries, including airlines, manufacturing, professional services, retail, and more.

  • 3 Three-Statement Modeling Exercises: A 30-minute case for Illinois Tool Works (U.S.), a 60-minute case for Frank Recruitment (U.K.), and a 90-minute case for Avianca (Colombia).
  • 3 Valuation and DCF Exercises: A 30-minute case for Idea Cellular (India), a 90-minute case for Michael Hill (Australia and New Zealand), and a 30-minute case for Attendo (Nordics).
  • 6 M&A and Merger Model Exercises: A 30-minute case for Steinhoff (South Africa), a 30-minute qualitative M&A deal discussion, a 30-minute deal discussion case for DSV (Europe), a 30-minute assessment center M&A case study, a 60-minute M&A calculations case study, and a 90-minute M&A case study on Starbucks and Krispy Kreme (U.S.).
  • 4 Leveraged Buyout and LBO Model Exercises: A 30-minute “paper LBO” model, a 60-minute case for Diana Shipping (Greece), a 90-minute case for Fromageries Bel (France), and a 2-hour case for My Family Fit (Singapore).
  • 1 Debt vs. Equity Exercise: 60-minute case for Central Japan Railway (Japan).

Hear Directly from Customers Who Achieved Spectacular RESULTS with the Investment Banking Interview Guide:

“i was shocked at just how well the guide prepared me for the questions they asked. it was like they were quizzing me on the guide.”.

investment banking case study interview questions

“Your prep material is by far the best, and you have an amazing intuition for how people learn information. Thank you so much. I am a customer and advocate of BIWS for life.”

investment banking case study interview questions

“What I feel is the real unique selling point of the guide is the fact that it doesn't give you a preset answer but teaches you to formulate your own answers to the questions.”

investment banking case study interview questions

“I'm a law school graduate with no financial background that will be starting at a BB M&A group in July.”

investment banking case study interview questions

“I am an undergraduate junior at MIT who was recently given an offer for a summer internship at Barclays IBD.”

investment banking case study interview questions

“I just wanted to take a moment to say that thanks to your interview guide, I landed an offer this summer at Goldman Sachs's Special Situations Group in New York.”

investment banking case study interview questions

“I've since recommended the interview prep guide to friends in other industries as the gold standard.”

investment banking case study interview questions

“I went through the EMEA IBD summer internship recruiting process last year and ended up with 2 offers. I decided to join JP Morgan's summer internship in London this summer and got a full-time offer.”

investment banking case study interview questions

“I wanted to let you know that I have secured a graduate position at a bulge bracket investment bank. I would not have been able to get through the technical interview questions without the interview guide.”

investment banking case study interview questions

“As an MBA student with absolutely zero finance experience, your IB Interview Guide was the most resourceful guide in my preparation.”

investment banking case study interview questions

“After a year of trying, I finally landed my dream job of working in IB at a big bank.”

investment banking case study interview questions

“I have secured 4 summer associate offers in London in IBD at Goldman Sachs, Morgan Stanley, Credit Suisse, and Nomura.”

investment banking case study interview questions

The BIWS Interactive Learning Portal is Your Roadmap to Fast Understanding and Quick Answers from Our Expert Support Team

investment banking case study interview questions

Can I See Samples of What’s in the Guide?

Sure! Just click on the links below to check out a few samples of what’s inside:

  • Sample “Story” Template and Example – Engineer or Technical Major to IB
  • Sample Chapter from the “Fit” Questions and Answers – Strengths and Weaknesses
  • Sample Technical Guide and Excel Files – Equity Value, Enterprise Value, and Valuation Multiples
  • Sample 30-Minute Case Study – Three-Statement Projection Model for Illinois Tool Works

ADD TO CART 100% Unconditional Money-Back Guarantee

Plus Expert Support From Experienced Investment Bankers

investment banking case study interview questions

Just moments after you enroll, you’ll receive Instant Access to the entire Guide.

Everything is digital, so you do not have to wait for physical products to arrive in the mail, and there are no shipping charges.

But that’s not the best part.

The best part is 365-day-per-year expert support, for a full 5 years after purchase!

If there’s anything at all you don’t understand or need to clarify, just go to the “Question/Comment” area below each and every lesson, and ask your question there.

These comments are monitored and responded to by our expert support team – every one of whom has personal experience working on deals at investment banks, private equity firms, and other finance firms.

That ensures that you’ll get responses from people with deep experience in the field – not a clueless high school temp clutching the “Help Desk” manual.

This personalized, expert support is one of the things that sets Breaking Into Wall Street apart and gets you to your goals faster.

You can often learn just as much from reading other customers’ questions and our responses, as you will from the lessons themselves!

Our 1-on-1 coaching rate is $200+ per hour. But when you invest in the Investment Banking Interview Guide 4.0, personal support is included for FREE .

NOTE: There are some limitations to these support services. For example, we cannot complete models, case studies, or homework assignments for you.

We also cannot provide play-by-play support with an earpiece during interviews.

Finally, we cannot answer questions on topics not covered in the guide, such as sales & trading interview questions or coding/programming interview questions.

We’re happy to answer career-related, qualitative, and technical questions that are related to the course materials.

So, What’s Your Investment in the IB Interview Guide 4.0 ?

To put this in context, let’s look at your Return on Investment in this guide…

The compensation for entry-level investment banking jobs varies from year to year, but it’s safe to say that even entry-level Analysts right out of university earn $150,000 USD at the bare minimum.

And the pay range is often more like $150,000 – $200,000, depending on your region and firm.

At the MBA level, that climbs to $300,000 – $400,000 USD or more (again, just in your first year out of school).

And as you progress, your total compensation only gets higher and higher; top senior bankers earn over $1 million USD annually.

And every top banker had to start at the entry level and get their foot in the door, just like you today.

So, we could charge $500 or $1,000 for this Guide because of the huge payoff potential and the fact it’s flat-out impossible to find out all this information and knowledge any other way (unless you have several years to do full-time research).

But I also want to make this information as accessible as possible to the finance community.

Many customers who sign up for the Interview Guide are so impressed that they go on to do more business with us down the road – so it benefits both of us if I lower the barrier to entry as much as possible.

That’s why I’ve decided to offer the complete Investment Banking Interview Guide, including everything described above, for a one-time payment of just $197.

At $197, the price shouldn’t be an object – the right role at the right firm could make you thousands of times that amount over the course of your career.

And perhaps more importantly: You only get one chance to make a good first impression.

That’s why we focus so heavily on your “story” in this Guide: You can do all the preparation in the world, but if you mess up those first 2-3 minutes of the interview, it’s all over.

How much is it worth to avoid that disastrous, but all-too-common, outcome?

An IB internship at a large bank pays $15,000+ USD, and a full-time role pays 10x that amount (or more), so the answer is “A lot more than $197.”

And just in case you’re not totally convinced this Guide is for you, rest assured that you’re covered by our risk-free 90-Day Money-Back Guarantee:

investment banking case study interview questions

Here’s What Happens Within Minutes of Signing Up

Once you sign up, you’ll immediately have access to the 578+ pages of written technical guides, the 18 templates for pitching yourself in interviews, the “Fit” guide and the deal/market discussion templates, the 18+ Excel files, the 218+ practice quiz questions, the 17 case studies, the video tutorials, and more.

And you’ll have access to our expert support team to ask whatever questions you need, for a full 5 years after purchase.

You’ll also get free updates and new content as we add it.

A Simple Choice…

If you are reading this right now, you’re serious about a career in investment banking, private equity, or hedge funds…

You know the importance of your first impression in that initial 30-minute interview and your performance in the interviews that follow.

Interview success can be the difference between a lucrative career at prestigious firms like Goldman Sachs, Morgan Stanley, KKR, and Blackstone… or the back office.

Sure… you could ‘wing it,’ hope for the best, and take a huge risk with your future, even though you know you are competing against some of the best and brightest…

You could prepare to dominate your competition, gain an unfair advantage, and land one of the limited investment banking opportunities available, by investing in the Investment Banking Interview Guide 4.0.

I know you will make the right choice so that you start landing six-figure job offers with investment banks – places that just might make your friends a bit jealous when they hear about your success!

Click here now to gain Instant Access to the Investment Banking Interview Guide .

To YOUR success,

investment banking case study interview questions

Brian DeChesare Breaking Into Wall Street Founder

P.S. It is never too early to start preparing for the most important interview of your career. Gain Instant Access to the Secure Members’ Area now, and you will receive an email with login instructions within a few short minutes.

In case the Investment Banking Interview Guide 4.0 isn’t everything I say it is, you are covered by our no-questions-asked, 100% risk-free money-back guarantee. You have nothing to lose, and a rewarding career in finance to gain.

P.P.S. Business philosopher Jim Rohn once said, “We must all suffer from one of two pains: the pain of discipline or the pain of regret. The difference is, discipline weighs ounces while regret weighs tons.”

My hope is that you do everything you can to ace your interviews and that you never have to live with the knowledge that there’s something you could have done to succeed, but didn’t.

Sign up now , and I’ll see you on the other side.

add to cart 100% Unconditional Money-Back Guarantee

Get Instant Access To The Investment Banking Interview Guide

investment banking case study interview questions

100% Unconditional Money-Back Guarantee

investment banking case study interview questions

Join 307,012+ Monthly Readers

book image

Get Free and Instant Access To The Banker Blueprint : 57 Pages Of Career Boosting Advice Already Downloaded By 115,341+ Industry Peers.

investment banking case study interview questions

  • Break Into Investment Banking
  • Write A Resume or Cover Letter
  • Win Investment Banking Interviews
  • Ace Your Investment Banking Interviews
  • Win Investment Banking Internships
  • Master Financial Modeling
  • Get Into Private Equity
  • Get A Job At A Hedge Fund
  • Recent Posts
  • Articles By Category

Investment Banking Interview Questions and Answers: The Definitive Guide

Investment Banking Interview Questions and Answers

If you're new here, please click here to get my FREE 57-page investment banking recruiting guide - plus, get weekly updates so that you can break into investment banking . Thanks for visiting!

If you ever Google “investment banking interview questions” , you could easily find yourself depressed .

There are endless books, articles and message board threads where people complain about unfair investment banking interviews, and horror stories about “bad cop” interviewers.

At first glance, it might seem like the interview preparation process for investment banking is next to impossible.

But I’d suggest that everyone is over-complicating it.

Investment banking interviews are not rocket science, and this article will unpack every type of question you’re likely to face in banking interviews (and how to answer them).

Table Of Contents:

Intelligent preparation for interview questions.

  • Investment Banking Interview Question Category 1: Telling Your Story
  • Investment Banking Interview Question Category 2: “Fit” Questions
  • Investment Banking Interview Question Category 3: Deals, Markets, and Companies
  • Finance Questions
  • Accounting Questions
  • Valuation Questions
  • DCF Questions
  • M&A and Merger Model Questions
  • LBO Model Questions
  • What NOT to Worry About: Brain Teasers and Questions to Ask
  • Associate Interview Questions

Let’s start with this summary infographic ( scroll down to go straight to a more detailed analysis):

Investment Banking Interview Questions & Answers Infographic

First, note that this article is about investment banking interview questions – not the overall process, how to win interviews, or what to do before and after the interviews.

For those, see our articles on how to get into investment banking , investment banking recruitment , and the investment banking industry .

Second, I’m going to link and refer back to our existing coverage for many of these questions since the most important ones have dedicated articles on this site.

The key preparation point is this: Rather than memorizing 541,763 questions and answers, you should focus on the main question categories and make sure that you have stories and examples prepared for them.

There are only four types of questions you’ll encounter in interviews at investment banks, and you can prepare for 3 / 4 of them in 1-2 days (or less).

The last category – technical questions – will take more time and effort, but you can save time by focusing on the right topics and ignoring the fluff.

Investment Banking Interview Questions Category 1: Your Story

There is only one question in this category, though it may be phrased in different ways:

  • “Tell me about yourself.”
  • “Walk me through your resume.”
  • “Why are you here today?”
  • “Tell me about your experience.”

Each of these phrases should trigger the “story reflex” in your brain and set your 200-300-word pitch into motion.

This question is the single most important one in any interview, so if you do nothing else to prepare, please take 30-60 minutes to outline your story!

Sample Questions and Answers:

I’ve already listed the main question variations above.

For sample answers , here are a few examples of how you can answer this question like a pro, taken from our article on how to answer the “ Walk me through your resume ” question:

  • Engineer or Technical Major to IB (Word)
  • Engineer or Technical Major to IB (PDF)
  • Big 4/Accounting to IB (Word)
  • Big 4/Accounting to IB (PDF)

And if you’re more of a visual learner, here’s a video tutorial on how to answer the “Walk me through your resume” question:

Investment Banking Interview Questions Category 2: “Fit” Questions

This category includes questions such as:

  • “What’s your greatest strength?”
  • “What’s your greatest failure?”
  • “Tell me about a team activity that did not go as planned.”
  • “Why is your GPA on the low side?”

All questions that are not related to your story, deal/market/company discussions, and technical concepts are in this category.

You can get a wide range of questions here, but you don’t need to memorize a wide range of answers – just come up with 2-3 examples that you can recycle for everything.

We explain how to do that in the article on investment banking fit questions , but in short:

First , you should outline 3 “short stories” – a leadership story, a “success” story, and a “failure” story. These should come from your work experience, but 1-2 can be from school if you’re an undergrad or recent grad.

Next , you should select 3 strengths and 3 weaknesses based on what bankers want to see (hard work, drive, ability to work long hours, attention to detail, financial skills, leadership, etc.).

Possible, not-completely-terrible weaknesses might include your inability to delegate tasks well, second-guessing yourself when making decisions, not managing your time well, or not always speaking up when a teammate has made a mistake.

Finally , you should prepare for the key objections that bankers will raise about your background: Did you not attend top universities? Are your grades on the low side ? Do you not have much work experience? Are you too old? Did you not major in something technical? Are you a job hopper?

Compare yourself to the ideal candidate and pinpoint how you’re different.

Once you’ve done that, you can practice with this video tutorial, and the sample questions that follow:

QUESTION: “What’s your greatest strength, and what’s your greatest weakness?”

ANSWER: Your greatest strength should be easy (see the list above – pick one and support it with a quick story). For your greatest weakness, pick one that’s “real,” but not too damaging for the role.

For example, don’t say you can’t delegate tasks well at the Associate level since you’ll have to do that all the time, especially as you advance.

If you say that you take too long to make decisions, state it and then back it up with your “failure” story such as how it took you too long to remove a team member who wasn’t contributing because you didn’t want to start a conflict – but that slow action ended up hurting the team.

QUESTION: “What feedback did you receive from your most recent internship or job?”

ANSWER: This one is a combined strengths/weaknesses question. It will be almost impossible to describe your 3 strengths and 3 weaknesses in 30-60 seconds, so pick 1 in each category and give a quick story to support it.

For example, you worked long hours and finished a last-minute task for a pending deal in your internship, which resulted in a successful close, but you could have been more proactive when following up on assignments and asking for the next steps.

QUESTION: “Can you describe a team situation where you worked with a difficult team member?”

ANSWER: You could use any of your 3 short stories here, depending on what they’re about.

You just have to say that a person in the group didn’t want to do the work, or wanted to do something unethical, or was competent but couldn’t get along with others.

Then, you convinced the others to side with you – and you gave this difficult team member something that wouldn’t sink the project to prevent further conflicts while still finishing the task.

QUESTION: “You have no experience in an investment bank. Why do you think your skills are relevant to this industry?”

ANSWER: Start by stating, briefly, the skills that are required for investment banking (see the article on investment banking analyst roles ), and then explain how your previous work experience has given you similar skills.

For example, you can point out how you’ve worked with clients in previous jobs, how you’ve had to work long hours and complete analytical/technical tasks, and how you’ve learned about accounting and finance in classes.

QUESTION: “You already have two years of work experience. Why couldn’t you get into an investment bank as an undergrad? Were you a failed candidate?”

ANSWER: No! Even if it’s true, never admit this.

You can answer this one by saying that you got interested in banking very late in the process, when it was too late to get the required sequence of internships (plausible if you went to a non-target school; not believable if you went to Harvard or Wharton).

Or, you could say that you knew about banking but deliberately chose something else – but then you realized you should have done banking, so you’ve been working toward it from the start of your full-time role (it’s much tougher to make this story work).

You must be clear that you didn’t “just” get interested in IB – you’ve been interested for a long time, and you completed specific client work/jobs to move closer over time.

QUESTION: “You’ve changed jobs twice in the past two years, and now you’re trying to switch once again. How do I know you won’t just leave our firm next year?”

ANSWER: Emphasize that you changed jobs twice because it was your original, long-term plan to do so.

You won’t change jobs yet again because working at an investment bank was your plan all along .

For example, you started out in audit, went to a boutique valuation firm to gain client and valuation experience , and now you want to move into banking to work on the entire deal process from beginning to end.

That has been your goal since you started out in audit, but you knew you couldn’t just move directly from audit to IB.

QUESTION: “The person in the room next door has perfect grades from Harvard or Oxford. You had lower grades and went to a state school. Why should I hire you over him?”

ANSWER: Because it’s the person who does the work, not the degree. Also, point out that you had to put in far more effort to get into this room than the other person did.

You’re also motivated to stay in banking for the long term because it’s your actual end goal; almost everyone from “elite schools” wants to get into private equity ASAP, which runs contrary to the long-term commitment that banks are now trying to encourage.

QUESTION: “Can you describe what a banker does in an IPO or M&A deal?”

ANSWER: Review our articles on mergers & acquisitions and initial public offerings (IPOs) .

QUESTION: “Why do you want to be an investment banker?”

ANSWER: This is just the last part of your story. It shouldn’t even be a question if you’ve told your story properly. But just in case, see the “ Why investment banking? ” article.

QUESTION: “Why our bank, specifically?”

ANSWER: See above. If you’ve mentioned something the bank is good at in your story, it shouldn’t even be a question. But just in case, see the “ Why our bank? ” article.

Investment Banking Interview Questions Category 3: Discussing Deals, Markets, and Companies

  • “Tell me about a recent deal.”
  • “Tell me about a deal our bank worked on recently.”
  • “Tell me about a company you’re interested in.”
  • “What makes Market X interesting to you?”

These questions are not that important unless you’ve had extensive deal experience that the interviewers plan to dig into – but they do require extra research and preparation.

We recommend the following steps:

  • Look Up 1 Deal the Bank Has Worked on Recently – Find something from the past ~6 months on the bank’s website or via Google searches. Outline the background, deal rationale, 1-2 financial stats, and your opinion of it. This can be very short because you just need to show that you know something about the bank.
  • Prepare for 1 In-Depth Deal/Market/Company Discussion – You should also prepare for a more in-depth discussion of a deal, and this deal does not have to be one that this bank advised on. For this, you’ll need the background information, deal rationale, a few financial stats, and your opinion of it. If they ask you to discuss a market, pick the market from this deal and make sure you know the approximate market size, key trends/drivers, major competitors, and your opinion of its prospects.
  • (If Applicable) Prepare for 2 Discussions of Your Own Deals – This one is applicable only if you’ve had previous IB, PE, corporate law, or Big 4 experience where you worked on deals or with clients. You should gather the background information, deal motivation, your personal contributions, and the current status for each one you use.

There are many tips on how to research and prepare for these questions in our articles on discussing a recent deal and discussing your own deals .

There isn’t much to say here because the most common questions are listed at the top of this section.

For examples of how to answer these questions, please see the templates and samples in the corresponding articles .

Investment Banking Interview Questions Category 4: Technical Questions and Answers

For this last category, I do not have any magical tips that will get you results in hours instead of weeks or months.

Put simply, to succeed in investment banking interviews, you need to put in the time to study accounting, finance, valuation, and M&A and LBO modeling.

If you don’t, you won’t have a great chance against candidates who are obsessed about becoming an investment banker and have spent months preparing.

We cover all these topics comprehensively in our full Investment Banking Interview Guide , but you can also get good introductions to them in our YouTube channel and the articles on this site:

  • Accounting Concepts (YouTube)
  • Equity Value and Enterprise Value (YouTube)
  • Equity Value vs Enterprise Value – comprehensive article and guide
  • Valuation Metrics and Multiples (YouTube)
  • Discounted Cash Flow (DCF) Analysis (YouTube)
  • M&A and Merger Models (YouTube)
  • Leveraged Buyouts and LBO Models (YouTube)

With limited time , focus on accounting, equity value and enterprise value, and valuation and DCF analysis. They are the most common topics, especially in entry-level interviews.

There are thousands of possible questions to test your technical knowledge, so I will list a few representative examples in each of the main categories.

I will focus on questions and answers that you probably haven’t seen on other sites and other resources, so most of these are in the “more challenging” range:

Example “Finance” Questions in Investment Banking Interviews

“Finance” means concepts such as the Time Value of Money, the Discount Rate, Present Value , and the Internal Rate of Return (IRR).

QUESTION: “How much would you pay for a company that generates $100 of cash flow every single year into eternity?”

ANSWER: It depends on your Discount Rate, or “targeted yield.”

If your Discount Rate is 10%, meaning you could earn 10% per year in companies with similar risk/potential return profiles, you would pay $100 / 10% = $1,000.

But if your Discount Rate is 20%, you would pay $100 / 20% = $500.

QUESTION: “A company generates $200 of cash flow next year, and its cash flow is expected to grow at 4% per year for the long term.

You could earn 10% per year by investing in other, similar companies. How much would you pay for this company?”

ANSWER: Company Value = Cash Flow / (Discount Rate – Cash Flow Growth Rate), where Cash Flow Growth Rate < Discount Rate.

So, this one becomes: $200 / (10% – 4%) = $3,333.

QUESTION: “What might cause a company’s Present Value (PV) to increase or decrease?”

ANSWER: A company’s PV might increase if its expected future cash flows increase, its expected future cash flows start to grow at a faster rate, or the Discount Rate decreases (e.g., because the expected returns of similar companies decrease).

The PV might decrease if the opposite happens.

QUESTION: “What does the internal rate of return (IRR) mean?”

ANSWER: The IRR is the Discount Rate at which the Net Present Value of an investment, i.e., Present Value of Cash Flows – Upfront Price, equals 0.

You can also think of it as the “effective compounded interest rate on an investment” – so, if you invest $1,000 today, end up with $2,000 in 5 years, and contribute and earn nothing in between, the IRR is the interest rate you’d have to earn on that $1,000, compounded each year, to reach $2,000 in 5 years.

Example “Accounting” Questions in Investment Banking Interviews

You don’t need to know accounting in terms of debits and credits, but you do need to know the 3 main financial statements and how they link together very well.

QUESTION: “How do the 3 financial statements link together? Assume the Indirect Method for the Cash Flow Statement.”

ANSWER: To link the statements, make Net Income at the bottom of the Income Statement the top line of the Cash Flow Statement.

Then, adjust this Net Income number for any non-cash items such as Depreciation & Amortization.

Next, reflect changes to operational Balance Sheet items such as Accounts Receivable (AR), which may increase or decrease the company’s cash flow depending on how they’ve changed.

(We might project some of these items, such as AR, with methods such as the Days Sales Outstanding .)

That gets you to Cash Flow from Operations.

Next, reflect investing and financing activities, which may increase or decrease cash flow, and sum up Cash Flow from Operations, Investing, and Financing to get the net change in cash at the bottom.

Link Cash on the Balance Sheet to the ending Cash number on the CFS, and add Net Income to Retained Earnings within Equity on the Balance Sheet.

Then, link each non-cash adjustment to the appropriate Asset or Liability; SUBTRACT links on the Assets side and ADD links on the L&E side.

Link each CFI and CFF item to the matching item on the Balance Sheet, using the same rule as above.

Check that Assets equals Liabilities + Equity at the end; if this is not true, you did something wrong and need to re-check your work.

QUESTION: “A company runs into financial distress and needs cash immediately. It sells a factory that’s listed at $100 on its Balance Sheet for $80. What happens on the 3 statements, assuming a 40% tax rate?”

ANSWER: Income Statement: Record a Loss of $20 on the Income Statement, which reduces Pre-Tax Income by $20 and Net Income by $12 at a 40% tax rate.

Cash Flow Statement: Net Income is down by $12, but you add back the $20 Loss since it’s non-cash. You also show the full proceeds, $80, in Cash Flow from Investing, so cash at the bottom is up by $88.

Balance Sheet: Cash is up by $88, but PP&E is down by $100, so the Assets side is down by $12. The L&E side is also down by $12 because Retained Earnings fell by $12 due to the Net Income decrease, so both sides balance.

QUESTION: “A company buys a factory using $100 of debt. A year passes, and the company pays 10% interest on the debt as it depreciates $10 of the factory. It repays $20 of the loan as well. Walk me through the statements from beginning to end, and assume a 40% tax rate.”

ANSWER: Initially, nothing changes on the IS. The $100 factory purchase shows up as CapEx on the CFS, and the $100 debt issuance shows up on the CFS as well, offsetting it, so Cash does not change at the bottom.

On the Balance Sheet, PP&E is up by $100, and Debt is up by $100, so both sides balance.

Then in the first year, you record $10 of interest and $10 of depreciation on the IS, reducing Pre-Tax Income by $20 and Net Income by $12 at a 40% tax rate.

On the CFS, Net Income is down by $12, but you add back the $10 of depreciation since it is non-cash, and the $20 loan repayment is a cash outflow, so Cash is down by $22.

On the BS, Cash is down by $22, and PP&E is down by $10, so the Assets side is down by $32. On the L&E side, Debt is down by $20 and Retained Earnings is down by $12, so the L&E side is down by $32 and both sides balance.

QUESTION: “What does the Change in Working Capital mean, intuitively?”

ANSWER: The Change in Working Capital tells you if the company needs to spend in advance of its growth, or if it generates more money as a result of its growth.

For example, the Change in Working Capital is usually negative for retailers because they must spend money on Inventory before being able to sell their products.

But the Change in Working Capital is often positive for subscription-based companies that collect cash in advance because Deferred Revenue increases when they do that.

The Change in Working Capital increases or decreases Free Cash Flow , which, in turn, directly affects the company’s valuation.

QUESTION: “What does it mean if a company’s Free Cash Flow is growing, but its Change in Working Capital is increasingly negative each year?”

ANSWER: It means that the company’s Net Income or non-cash charges are growing by more than its Change in WC is declining, or that its CapEx is becoming less negative (i.e., shrinking) by more than the Change in WC is declining.

If a company’s Net Income is growing for legitimate reasons, this is a positive sign. But if higher non-cash charges or artificially low CapEx are boosting FCF, both of those are negative.

Example “Valuation” Questions in Investment Banking Interviews

You need to understand the “big picture” behind valuation, how Equity Value and Enterprise Value differ, and the trade-offs of different multiples and methodologies.

Questions like “How do you value a company?” or “Tell me the 3 basic valuation methodologies” are so basic that banks almost assume you already know them.

QUESTION: “What do Equity Value and Enterprise Value mean, intuitively?”

ANSWER: Equity Value is the value of ALL the company’s Assets, but only to EQUITY INVESTORS (common shareholders).

Enterprise Value is the value of only the company’s core-business Assets, but to ALL INVESTORS (Equity, Debt, Preferred, and possibly others).

For more, please see our tutorial on how to calculate Enterprise Value .

QUESTION: “A company issues $200 million in new shares, and then it uses $100 million from the proceeds to issue Dividends to shareholders. How do Equity value and Enterprise Value change in each step ?”

ANSWER: Initially, Equity Value increases by $200 million because Total Assets increases by $200 million and the change is attributable to common shareholders.

Enterprise Value stays the same because Cash is a non-core-business Asset; you can also say that the increases in Cash and Equity Value offset each other in the Enterprise Value formula.

In the next step, Equity Value decreases by $100 million because Cash, and therefore Total Assets, falls by $100 million and this change is attributable to common shareholders.

Enterprise Value stays the same because Cash is a non-core-business Asset, or because the reduced Cash and reduced Equity Value offset each other.

QUESTION: “What are the advantages and disadvantages of EV / EBITDA vs. EV / EBIT vs. P / E as valuation multiples?”

ANSWER: With EV / EBITDA vs. EV / EBIT, EV / EBITDA is better in cases when you want to completely exclude the company’s CapEx, Depreciation, and capital structure.

EV / EBIT is better when you want to exclude capital structure but partially factor in CapEx and Depreciation. It is common in industries where those items are key value drivers for companies (e.g., manufacturing).

The P / E multiple is not terribly useful in most cases because it’s affected by different tax rates, capital structures, non-core-business activities, and more – so, you often use it in the interest of “completeness” or because you want a multiple that reflects a company’s true bottom line.

Also, it’s important in industries such as commercial banking and insurance where you do need to factor in the interest income and expense.

For more on this topic, please see our guide to EBIT vs. EBITDA vs. Net Income

QUESTION: “Which of the main 3 valuation methodologies will produce the highest valuations?”

ANSWER: Any methodology could produce the highest valuations depending on the industry, period, and assumptions.

But you can say that Precedent Transactions often produce higher values than the Public Comps because of the control premium – the extra amount that acquirers must pay to acquire sellers.

It’s tough to say how a DCF model stacks up because it’s far more dependent on the assumptions and far-in-the-future projections.

So: “A DCF tends to produce the most variable output since it’s so dependent on the assumptions, and Precedent Transactions tend to produce higher values than the Public Comps because of the control premium.”

QUESTION: “How might you select a set of comparable public companies for use in a valuation?”

ANSWER: You screen based on geography, industry, and size. For example, your screen might be “U.S.-based steel manufacturing companies with over $500 million in revenue” or “European legacy airlines with over €1 billion in EBITDA.”

Get more on Comparable Company Analysis in our YouTube channel.

Example “DCF” Questions in Investment Banking Interviews

The DCF is “real valuation”; multiples are just abbreviated ways to express it.

So, you can expect questions on everything from the basic idea to a walk-through to the Discount Rate and Terminal Value calculations.

QUESTION: “Explain the big idea behind a DCF analysis and how it is used to value a company.”

ANSWER: A DCF is an expansion of this formula:

Company Value = Cash Flow / (Discount Rate – Cash Flow Growth Rate), where Cash Flow Growth Rate < Discount Rate

The problem is that that formula assumes the company’s Discount Rate and Cash Flow Growth Rate never change – but in real life, they keep changing until the company reaches maturity.

So, in a Discounted Cash Flow analysis, you divide the valuation into two periods: One where those assumptions change (the explicit forecast period) and one where they stay the same (the Terminal Period).

You then project the company’s cash flows in both periods and discount them to their Present Values based on the appropriate Discount Rate(s).

Then, you compare this sum – the company’s Implied Value – to the company’s Current Value or “Asking Price” to see if it’s valued appropriately.

QUESTION: “Walk me through an Unlevered DCF .”

ANSWER: You start by projecting the company’s Unlevered Free Cash Flows over the next 5-10 years by making assumptions for revenue growth, margins, Working Capital, and CapEx.

Unlevered FCF excludes all financing and non-core-business activities and equals EBIT * (1 – Tax Rate) + D&A +/- Change in Working Capital – CapEx.

Then, you discount the UFCFs to Present Value using the Weighted Average Cost of Capital and sum up everything.

Next, you estimate the company’s Terminal Value using the Multiples Method or the Gordon Growth Method; it represents the company’s value after those first 5-10 years into perpetuity.

You then discount the Terminal Value to Present Value using WACC and add it to the sum of the company’s discounted UFCFs.

Finally, you compare this Implied Enterprise Value to the company’s Current Enterprise Value; you’ll often calculate the company’s Implied Share Price so you can compare that to the Current Share Price as well.

Get more on Unlevered Free Cash Flow in our YouTube channel.

QUESTION: “Explain what WACC means intuitively and how you might calculate each component of it.”

ANSWER: WACC is the expected annualized return over the long term if you invest proportionately in all parts of the company’s capital structure – Debt, Equity, Preferred Stock, and anything else it has.

To a company, WACC represents the cost of funding its operations by using all its sources of capital and keeping its capital structure percentages the same over time. The WACC formula is simple:

WACC = Cost of Equity * % Equity + Cost of Debt * (1 – Tax Rate) * % Debt + Cost of Preferred Stock * % Preferred Stock

You usually estimate the Cost of Equity with Risk-Free Rate + Equity Risk Premium * Levered Beta.

The Cost of Debt and Cost of Preferred can be based on the Yield to Maturity (YTM) of the current issuances, the median rates or YTMs on the issuances of peer companies, or you can take the Risk-Free Rate and add a default spread based on the company’s credit rating after it issues more Debt or Preferred.

(For more, please see our full tutorials on the bond yield , the Current Yield , the Yield to Maturity , the Yield to Call , and the Yield to Worst .)

QUESTION: “A company goes from 20% Debt / Total Capital to 30% Debt / Total Capital. How do its Cost of Equity, Cost of Debt, and WACC change? Assume it only has Debt and Equity.”

ANSWER: As a company uses more Debt, the Cost of Debt and Cost of Equity always increase because more Debt increases the risk of bankruptcy, which affects all investors.

As the company goes from no Debt to some Debt, WACC decreases at first because Debt is cheaper than Equity , but it starts to increase at higher levels of Debt as the risk of bankruptcy starts to outweigh the lower Cost of Debt.

In this case, we can’t tell how WACC will change because we don’t know where we are on this “curve” – but we guess that WACC will likely decrease because 30% Debt / Total Capital is still in a fairly low/normal range for most industries.

QUESTION: “How do you calculate and sanity check Terminal Value in a DCF?”

ANSWER: You apply a Terminal Multiple , such as an EV / EBITDA figure based on the comparable companies, to EBITDA in the final year of the forecast period, or you pick a Terminal FCF Growth Rate and use a variation of the “Company Value” formula:

Terminal Value = Final Year FCF * (1 + Terminal FCF Growth Rate) / (Discount Rate – Terminal FCF Growth Rate)

To check yourself, back into the Terminal FCF Growth Rate implied by the first method and the Terminal Multiple implied by the second method.

If you get, say, a 10% Implied Terminal FCF Growth Rate for a company in a developed country, you’re way off and need to pick a lower multiple that results in a growth rate below the long-term GDP growth rate.

Example M&A and Merger Model Interview Questions

These questions are less important than those in the other technical categories above, but you should still know the basic concepts. We have a full YouTube video tutorial on these questions:

  • Merger Model Interview Questions – Slide Format
  • Merger Model Interview Questions – Sample Excel File

But if you prefer the text version, here’s a sample:

QUESTION: “Walk me through a merger model.”

ANSWER: Start by projecting the financial statements of the Buyer and Seller. Then, you estimate the Purchase Price and the mix of Cash, Debt, and Stock used to fund the deal. You create a Sources & Uses schedule and Purchase Price Allocation schedule to estimate the true cost of the acquisition and its effects.

Then, you combine the Balance Sheets of the Buyer and Seller, reflecting the Cash, Debt, and Stock used, new Goodwill created, and any write-ups. You then combine the Income Statements, reflecting the Foregone Interest on Cash, Interest on Debt, and Synergies (for both revenue and expenses, but if you have to pick one, cost synergies are more important). If Debt or Cash changes over time, the Interest figures should also change.

The Combined Net Income equals the Combined Pre-Tax Income times (1 – Buyer’s Tax Rate), and you divide that by (Buyer’s Existing Share Count + New Shares Issued in the Deal) to get the Combined EPS.

You calculate the accretion/dilution by dividing the Combined EPS by the Buyer’s standalone EPS and subtracting 1.

For more, see our tutorial on the Earnings per Share formula and EPS accretion / dilution .

QUESTION: “A company with a P / E multiple of 25x acquires another company for a purchase P / E multiple of 15x. Will the deal be accretive or dilutive?”

ANSWER: You can’t tell unless it’s a 100% Stock deal. If it is, it will be accretive because the Cost of Acquisition is 1 / 25, or 4%, and the Seller’s Yield is 1 / 15, or 6.7%. Since the Seller’s Yield is higher, it will be accretive.

QUESTION: “Let’s say it is a 100% Stock deal. The Buyer has 10 shares at a share price of $25.00, and its Net Income is $10.  It acquires the Seller for a Purchase Equity Value of $150. The Seller has a Net Income of $10 as well. Assume the same tax rates for both companies. How accretive is this deal?”

ANSWER: The buyer’s EPS is $10 / 10 = $1.00. It must issue $150 / $25.00 = 6 additional shares to do the deal, so the Combined Share Count is 10 + 6 = 16.

Since both companies have the same tax rate and since no Cash or Debt is used, Combined Net Income = $10 + $10 = $20, and Combined EPS = $20 / 16 = $1.25, so the deal is 25% accretive.

QUESTION: “What are the Combined Equity Value and Enterprise Value in this same deal? Assume that Equity Value = Enterprise Value for both the Buyer and Seller.”

ANSWER: Combined Equity Value = Buyer’s Equity Value + Value of Stock Issued in the Deal = $250 + $150 = $400.

Combined Enterprise Value = Buyer’s Enterprise Value + Purchase Enterprise Value of Seller = $250 + $150 = $400.

QUESTION: “Without doing any math, what ranges would you expect for the Combined EV / EBITDA and P / E multiples, and why?”

ANSWER: They should be somewhere in between the Buyer’s multiples and the Seller’s purchase multiples. It’s almost never a simple average because of the relative sizes of the Buyer and Seller – and for P / E multiples, the purchase method also plays a role.

Example LBO Model Interview Questions and Answers

These questions are also less important than the ones in the categories above, but you’ll still be expected to know the big picture behind LBOs. We also have a full YouTube tutorial for these:

  • LBO Model Interview Questions – Slide Format

To understand the main ideas and mechanics, also check our our tutorial on how to build a simple LBO model .

And if you prefer the questions and answers in text, here’s a sample:

QUESTION: “Walk me through a leveraged buyout model.”

ANSWER: In a leveraged buyout, a PE firm acquires a company using a combination of Debt and Equity, operates it for several years, and then sells it. The math works because leverage amplifies returns; the PE firm earns a higher return if the deal goes well because it uses less of its own money upfront (and it earns an even lower return if the deal goes poorly!).

In Step 1, you make assumptions for the Purchase Price, Debt and Equity, Interest Rate on Debt, and Revenue Growth and Margins.

In Step 2, you create a Sources & Uses schedule to calculate the Investor Equity paid by the PE firm.

In Step 3, you adjust the Balance Sheet for the effects of the deal, such as the new Debt, Equity, and Goodwill (see our tutorial for more on how to calculate Goodwill ).

In Step 4, you project the company’s statements, or at least its cash flow, and determine how much Debt it repays each year.

Finally, in Step 5, you make assumptions about the exit, usually using an EBITDA multiple, and calculate the IRR and cash-on-cash multiple .

QUESTION: “What’s an ideal LBO candidate?”

ANSWER: Price is the most important factor because almost any deal could work at the right price (i.e., one that’s low enough) – but if the price is too high, the chances of failure increase substantially.

Beyond that, stable and predictable cash flows are important, there shouldn’t be a huge need for ongoing CapEx or other big investments, and there should be a realistic path to exit, with returns driven by EBITDA growth and Debt paydown instead of multiple expansion .

QUESTION: “A PE firm acquires a $100 million EBITDA company for a 10x multiple using 60% Debt.

The company’s EBITDA grows to $150 million by Year 5, but the exit multiple drops to 9x. The company repays $250 million of Debt and generates no extra Cash. What’s the IRR?”

ANSWER: Initial Investor Equity = $100 million * 10 * 40% = $400 million.

Exit Enterprise Value = $150 million * 9 = $1,350 million.

Debt Remaining Upon Exit = $600 million – $250 million = $350 million.

Exit Equity Proceeds = $1,350 million – $350 million = $1 billion.

This represents a 2.5x multiple over 5 years, and you should know that a 2x multiple over 5 years is a ~15% IRR, while a 3x multiple is a ~25% IRR, so this IRR is approximately 20%.

QUESTION: “You buy a $100 EBITDA business for a 10x multiple, and you believe that you can sell it again in 5 years for 10x EBITDA.

You use 5x Debt / EBITDA to fund the deal, and the company repays 50% of that Debt over 5 years, generating no extra Cash. How much EBITDA growth do you need to realize a 20% IRR?”

ANSWER: Initial Investor Equity = $100 * 10 * 50% = $500.

20% IRR Over 5 Years = ~2.5x multiple (2x = ~15% and 3x = ~25%).

Required Exit Equity Proceeds = $500 * 2.5 = $1,250.

Remaining Debt = $250, so Exit Enterprise Value = $1,500.

Required EBITDA = $150, since $1,500 / 10 = $150. So, EBITDA must grow by 50%.

What NOT to Worry About In Investment Banking Interviews: Brain Teasers and Questions to Ask

Phew. OK, we’re done with that list of sample questions that ended up being surprisingly long.

I’ve seen prospective investment bankers worry about two subjects that do not matter much for traditional IB interviews: brain teasers and the questions you ask the interviewer when he/she asks if you have any questions at the end.

Brain teasers are more likely in sales & trading interviews or consulting interviews, and less likely in banking because they have nothing to do with the job.

So, I wouldn’t recommend spending much time learning how to estimate the number of golf balls that fit in a 747 or how to move water between jugs of different sizes.

If you are worried because you’re interviewing at an elite boutique or a group/firm known for brain teasers, get a book to prepare.

On another note, interviewees tend to obsess over “the right questions” to ask interviewers at the end.

But the truth is, these questions are almost irrelevant unless you say something stupid or inappropriate.

Just ask a question about the person’s background, experience at the bank so far, etc., and don’t devote brain cells to this one.

Finally, Excel tests for interviews are quite unlikely for entry-level roles in the U.S.; they could come up if you’re at a more advanced level or interviewing in a region such as EMEA.

Investment Banking Associate Interview Questions

There are no huge differences for Associate-level candidates , as the same topics and types of questions tend to come up.

The main difference is that you need to be more polished because everyone at this level at an investment bank is articulate and has more real-world experience.

It’s also quite important to focus on a specific industry because they want candidates who can leverage their pre-MBA experience for something useful on the job.

Finally, case studies – sometimes informal verbal ones, sometimes in writing, and sometimes in Excel – are more likely to come up at this level.

To practice, you can look at the many example case studies and solutions in the full Breaking Into Wall Street Investment Banking Interview Guide .

For more, please see our article on the investment banking associate job .

This article is detailed and comprehensive, but the most important point is simple: You cannot “prepare” for the technical questions in an investment banking interview at the last minute.

You can come up with a halfway decent story and reasonable answers to common “fit” questions with limited time – such as a few hours or days before the interview.

But you cannot answer detailed questions about LBO models, the components of Enterprise Value, or how WACC changes under different conditions unless you have solid technical knowledge, which takes time to acquire.

At the minimum , you’ll have to start ~2-3 months in advance to get a good sense for these concepts (assuming no background or limited accounting/finance knowledge).

The other question categories can wait until the last minute, but you can’t cram and master the technical side in that span of time.

If you want to go beyond this article with your preparation, the IB Interview Guide mentioned in several spots above is a good starting point. And if you have more time and want to focus on learning the concepts rather than review/practice, the Core Financial Modeling course will get you there:

course-1

Core Financial Modeling

Learn accounting, 3-statement modeling, valuation/DCF analysis, M&A and merger models, and LBOs and leveraged buyout models with 10+ global case studies.

It teaches accounting and valuation from the ground up and has many examples of 3-statement models, valuations, and M&A and LBO case studies, with a focus on interviews and preparation for internships.

Finally, if you want to have a team of coaches help you with both your behavioral and/or technical interview preparation, our friends at Wall Street Mastermind  might be able to help you out.

They’ve worked with over 1,000 students to help them secure high-paying investment banking jobs out of school (and internships while in school), and their coaches include a former Global Head of Recruiting at three different large banks.

They provide personalized, hands-on guidance through the entire networking and interview process – and they have a great track record of results for their clients.

You can book a free consultation with them to learn more .

investment banking case study interview questions

About the Author

Brian DeChesare is the Founder of Mergers & Inquisitions and Breaking Into Wall Street . In his spare time, he enjoys lifting weights, running, traveling, obsessively watching TV shows, and defeating Sauron.

Free Exclusive Report: 57-page guide with the action plan you need to break into investment banking - how to tell your story, network, craft a winning resume, and dominate your interviews

Read below or Add a comment

18 thoughts on “ Investment Banking Interview Questions and Answers: The Definitive Guide ”

' src=

Hi Brian, I have an interview for a debt and restructuring SA position coming up. Does the LBO and M&A stuff still apply to me, or will they most likely not ask about it since it wouldn’t be relevant to the role.

investment banking case study interview questions

You will still get standard questions on all the normal topics even in restructuring interviews.

' src=

During my MBA I did a boutique IB internship but since they were having team issues I decided to take my big tech offer. It is only 3 month now but I started regretting my decision and reapplying to IB again to Industrial group so unrelated to tech experience. How would you frame the answer to why I went to tech at the time from IB? Would be great to hear your insight. Thanks a lot.

Given the timing here, I don’t think you will sound believable if you claim that taking the Big Tech offer was your plan all along (maybe if you stayed for a year and then applied to IB). So you will probably have to say that you made a mistake and had a poor impression of IB from your internship, but quickly realized that larger banks are quite different and do not have the same types of cultural/team issues you encountered there. You took the Big Tech offer because it seemed better at the time, but you realized you’re not a good fit for whatever reason, so you still prefer finance.

' src=

Hi Brian, for the problem on company buying factory using $100MM in debt, I think you should also include $10MM interest paid in CFO and reduce Interest Payable by $10MM at year end.

No. All these accounting questions assume standard U.S. GAAP treatment of the financial statements, i.e., that CFO starts with Net Income and then adjusts for non-cash items and the Change in WC (as opposed to the IFRS/other treatment where the CFS may use the Direct Method or start with OpInc or Pre-Tax Income or something else rather than Net Income).

Including interest expense therefore makes no sense because it would be double-counting, as it’s already deducted in Net Income on the IS. Interest Payable is pointless because the interest is not going to remain a payable for a year. Most loans have interest payments that are due on monthly, quarterly, or semiannual schedules.

' src=

Hey Brian I just had a quick question. How much do pre-employement, AI driven assessments like Pymetrics or CCAT factor into the recruiting process? If they do significantly matter, do you have any tips on how to prepare for them? Thanks!

They factor in, but no one knows by exactly how much because banks do it differently and never disclose their methods. But they’re normally more of an “elimination” exercise than anything else. Answer poorly enough and you’re out, but a great score doesn’t necessarily help you. All you can do is practice with firms that offer online tests.

' src=

Thank you Brian for your blog! Just one question I would like to know if age plays a key role to get a summer internship as an analyst intern to a BB (London)? I am 26 and just get an offer for a Master at LSE but I am worried that my age could affect my application for the summer 2021 position. Do you think I have still my chance?

It all depends on how much full-time work experience you already have. If it’s beyond 2-3 years, it will be tough to win Analyst/Intern offers. Please see: https://mergersandinquisitions.com/age-investment-banking/

' src=

Hey, was just wondering of you could do an article on Hirevues. They are quite common but there’s only generic advice out there. I’m sure tons of people are stymied by them. Are then any differences between the UK and the US?

There is really not much to say, and most of it’s common sense (wear clothes, look at the camera, do it in a quiet spot so your roommate doesn’t come barging in, etc.). The questions tend to be very simple fit-based ones as well. We may eventually write something on Hirevues, but it almost feels silly.

' src=

Brian, I have been reading your blog for more than a year and it would be a great pleasure to hear your advice. I will try to sum up my background as much as I can and then ask you my question.

Studied Economics in Spain with an overall average of 7/10. Beginning with low grades, during the last two years I studied significantly more and got an average of 9.5/10. During this time I studied one semester in the UK (not in a top ten Uni) and I did an 8-month full time internship in structured finance.

After my degree I did a two year graduate programme in the same bank in which I rotated through different departments. Since I finished this programme 6 months ago I am working in the London branch of this bank managing an SMEs portfolio.I recently got the GMAT with 720.

What do you think it would be my best plan to get into IB in London or NY in a top bank? Directly applying to IB jobs, study an MBA, MSc in Finance? I am looking for the best long term path. Now I am 24.

Thank you very much for your time and all the best.

At this point, I think you would probably need another degree to have a good shot at the top banks. You can sometimes network and win IB offers if you do so right after you graduate, but 2.5 years is a bit too much time for that to work. If you apply ASAP, a top MSc in Finance program could work well, but you’re right on the border of having too much experience for an MSF… so if you wait any longer, you would probably have to consider a top MBA program instead.

Since you have experience at various banks and are not exactly a “career changer,” though, you should probably start by applying to top MSF programs right away. If that doesn’t work or it takes too long or other problems come up, then consider an MBA.

Thank you Brian!! The FT ranking has to be very reliable to find the best MSc in Finance and MBA programmes but in the MSc one for example I don’t see the top USA unis apart from the MIT…what ranking do you think would the best one to find the best graduate programmes?

Ranking I am refering to: http://rankings.ft.com/businessschoolrankings/masters-in-finance-pre-experience-2018

Thank you very much.

See some of the suggestions and comments here: https://mergersandinquisitions.com/investment-banking-masters-programs/

Hi Brian, I think you will know the answer to this. To work in a top PE firm (KKR, TPG) in at least 5 years, would it make sense to directly study an MBA in a top uni and then work between 2-3 years in IB? Thanks, Toni

Generally it’s not a good idea to do that because it’s far more difficult to transition into PE at the Associate level. You’re much better off working as an IB Analyst and then moving in like that. If you have no other good route into IB, then yes, you pretty much have to do the MBA. See: https://mergersandinquisitions.com/investment-banking-associate-exit-opportunities/

Leave a Reply Cancel reply

Your email address will not be published. Required fields are marked *

Got an Investment Banking Interview?

The BIWS Interview Guide has 578+ pages of technical and fit questions & answers, personal pitch templates, 17 practice case studies, and more.

investment banking case study interview questions

  • The Investment Banker Micro-degree
  • The Project Financier Micro-degree
  • The Private Equity Associate Micro-degree
  • The Research Analyst Micro-degree
  • The Portfolio Manager Micro-degree
  • The Restructurer Micro-degree
  • Fundamental Series
  • Asset Management
  • Markets and Products
  • Corporate Finance
  • Mergers & Acquisitions
  • Financial Statement Analysis
  • Private Equity
  • Financial Modeling
  • Try for free
  • Pricing Full access for individuals and teams
  • View all plans
  • Public Courses
  • Investment Banking
  • Investment Research
  • Equity Research
  • Professional Development for Finance
  • Commercial Banking
  • Data Analysis
  • Team Training
  • Felix Continued education, eLearning, and financial data analysis all in one subscription
  • Learn more about felix
  • Publications
  • My Store Account
  • Learning with Financial Edge
  • Certification
  • Masters in Investment Banking MSc
  • Student Discount
  • Find out more
  • Diversity and Inclusion
  • The Investment banker
  • The Private Equity
  • The Portfolio manager
  • The real estate analyst
  • The credit analyst
  • Felix: Learn online
  • Masters Degree
  • Public courses

Investment Banking Case Studies – Job Application

By Ivy Wang |

 Reviewed By Rebecca Baldridge |

November 21, 2022

What are Investment Banking Case Studies

Candidates will normally encounter case studies at the final stage of the application process, likely during an assessment or final-stage interview. Investment banking roles are highly competitive, and candidates must be properly prepared.

Investment banking case studies are commonly used to assess how a job candidate would perform in a real situation by presenting them with a theoretical scenario similar to those encountered on the job.

Most investment banking case study questions center on acquisitions, raising capital, or company expansion. The case may be given to you ‘blind’ on the day of your assessment with only a short amount of time provided for preparation. If the case is likely to involve deep analysis, financial modeling, or a company valuation, you will likely be given the case in advance to give you more time to work on it before the assessment day.

Key Learning Points

  • Investment banking case studies are often used to assess how job candidates would perform in a real situation by presenting them with a theoretical scenario like one they would encounter on the job.
  • While general questions give the interviewer a superficial impression of the candidate’s skills and fitness, case study questions allow the interviewer to assess the candidate’s ability to handle multiple levels of analysis and problem-solving.
  • Case study questions test reasoning and communication skills as well as analytical skills. They are useful for assessing how candidates approach complex issues, make critical judgments, and deliver recommendations.
  • With many case study problems, there will be more than one path to success and more than one possible solution.
  • Investment banks are looking for decisive candidates who can articulate and logically present their solutions and defend their decisions under scrutiny.

What are the Types of Investment Banking Case Studies?

In general, there are two types of case studies during an investment banking assessment, the decision-making case, and the financial modeling case.

Decision-Making Investment Banking Case Studies

Decision-making case studies appear more frequently than modeling case studies. In this type of case study, the candidate must make decisions for their client and provide advice. The client case studies could be based on finding funding sources, or whether a proposed merger should proceed and why or why not.

You should expect these questions to be given to you at the interview. Therefore, you must analyze and present the case within a given time frame. Throughout, you will have 45-60 minutes of preparation time and a 10-minute presentation, followed by a round of Q&A.

These case studies do not involve in-depth analysis of the case, given the short amount of time available.

Decision-Making Case Study Example

One of your clients is a global corporation that manufactures and distributes a wide range of perfumes. They are contemplating ways to expand their business. They may either introduce a new range of fragrances with their current distribution channels or start a completely new company with different stores.

You need to determine which solution is better for the business. To do so, you need to compare the return on investment and decide on a solution. Be ready to support your reasoning.

Modeling Investment Banking Case Studies

Modeling case studies are usually take-home assignments where you must do financial modeling and a simple valuation. Thus, it is more of a modeling test than a case study. The Investment Banker provides an introduction to building models, developing multiple techniques for a comprehensive and practical understanding of the topic.

The modeling case study will either employ a free cash flow to the firm (FCFF) valuation on a company or require a simple merger or leveraged buyout model. You would be expected to analyze the corporations’ valuation multiples to determine whether they are undervalued or overvalued.

Generally, you will be given a few days to complete your analysis. Then, you need to spend 30-45 minutes on the day of the interview presenting your case to the bankers. The analysis will go much deeper than a client case study because you will have more time to work on it.

Modeling Investment Banking Case Study Example

A pharmaceutical company wishes to make an acquisition. It has identified the target company and approached you to determine how much they should pay. You will be provided with the necessary financial information, metrics, and multiples, as well as the buyer and seller company profiles.

To complete the case study, you need to determine if the acquisition is feasible. Second, what would be the structure and synergies of the deal if the buyer has access to capital? You need to use multiples and valuation metrics to determine the price range for the transaction.

Access the three-statement case study example to practice a modeling case study.

How to Prepare for Investment Banking Case Studies

Regardless of which type of case study a candidate is presented, the thought process and deliverables are the same. The best way to prepare is to:

  • Ensure that key business concepts are well understood and that you can use the associated terminology comfortably in a conversation.
  • Learn about various valuation techniques, how to employ them, and how to interpret them. Prepare for case studies by mastering valuation for investment banking with the online investment banker course . Learn how and when to utilize key valuation methodologies and the supporting calculations
  • Make sure you read business news regularly and focus on discussing the details of banking transactions in the news.
  • Read as many case studies as possible so you get the knack of understanding business scenarios and analyzing Especially for modeling and valuation-based case studies, you must be prepared to format your work using PowerPoint and Excel.
  • Check company websites to see if sample case studies are available for reference. Investment banks do not tend to publish case study questions for practice. However, it is possible to formulate your own questions by looking at business scenarios involving possible mergers, valuations, or capital raises.
  • Candidates must practice streamlining their thought process so judgments can be made under time pressure.
  • Read through the scenario carefully before beginning to form an opinion on how the problem should be tackled. This will ensure that no intricacies are missed, and your response addresses all facets of the case.
  • Learn how to stand out from the crowd in your interview with the investment banker interview skills course , designed to prepare you for your interviews and enable you to make a great impression.
  • Investment Banking Case Study Example

1.     Scenario

A magazine publisher is evaluating whether it should sell, continue to grow organically, or make small “tuck-in” acquisitions to maximize shareholder value. It is selecting an investment bank to advise on its options and has requested a presentation from your bank.

2.     Task

Review the company’s financial and market information and create a 30-minute presentation analyzing its options. Recommend a specific course of action – selling the company, continuing to grow organically, or making smaller acquisitions.

3.     Solution

The answer to this case study is rather subjective. You should take a stand and support it with well thought out reasons. Here’s how you should approach it:

  • First, read through everything and get a sense of the industry, where it’s heading, and how much this company might be worth based on comparables. If they don’t give you much information on comparable public companies or precedent transactions, you’ll have to do your own research.
  • Complete a brief valuation using public comps, precedent transactions, and a DCF.
  • Weigh the numbers the valuation gives you, the company’s organic growth prospects, and whether there are any good companies to acquire.
  • Make a decision-it’s usually best to say “sell” unless the industry is growing quickly (over 10% per year), the company is extremely undervalued, or there are acquisition targets that would boost revenue or profit by at least 20-30%.
  • Keep this simple and straightforward-the numbers should back up your reasoning, not take over the entire presentation.
  • You could get much fancier with the analysis and look at the company’s valuation now, 5 years from now, and if it acquires 1 or 2 companies, but that isn’t necessary and it may just make your presentation more confusing.

4.     Sample Answer

If you decide to sell, you can write:

Slide 1: Recommendation to sell and the three key reasons why.

Slide 2: Industry overview – Is it growing?  Shrinking?  Stagnant?

Slide 3: Company’s position in the industry – Leader?  Tier 2 player?  Where is it strong / weak?

Slide 4: What organic growth would look like 5 or 10 years in the future – how much bigger / more highly-valued would the company be?

Slide 5: Potential tuck-in acquisition candidates.

Slide 6: Why neither organic growth nor acquisitions are the answer.

Slide 7: Why selling now produces the greatest shareholder value.

Slide 8: Valuation – Show public comps and precedent transactions.

Slide 9: Valuation – Show DCF output and sensitivity table.

Slide 10: Conclusion – Reiterate that selling now is the best option and that neither organic growth nor acquiring smaller companies will result in a higher valuation 5-10 years from now.

If you come to a different conclusion – for example, that acquisitions are the best strategy, you would reverse the order and list the solutions you’re not recommending first, concluding with the one you are suggesting.

Investment banking case studies are an important element in the interview process, it is an opportunity to showcase your skills and talent to investment bankers. In general, there are two types of case studies, the decision-making case study and the financial modeling case study. Candidates will need to be confident in their valuation skills. They will also need to display a good level of commercial awareness. Presentation skills are also critical.

Share this article

Investment banking case study.

Sign up to access your free download and get new article notifications, exclusive offers and more.

Recommended Product

investment banking case study interview questions

investment banking case study interview questions

Investment Banking Case Study Examples – A Guide

If you are preparing for an investment banking interview, you’ll probably need to conquer a case study interview. because case studies are a very crucial component in the investment banking hiring process. particularly if you have never completed a case study before, that will be very challenging for you to get into the investment banking field. this article has covered everything you need to know about investment banking and potential investment banking case studies. there are also tips and practice investment banking case study questions with examples of how to resolve them..

Investment Banking Case Study Examples (1)

What is Investment Banking?

Investment banks are financial firms that perform a variety of tasks, including underwriting, assisting companies with the issuance of stock and debt securities through initial public offerings or fixed-priced offerings enabling mergers and acquisitions on both the buy side and sell side of the deal, corporate restructuring and many other tasks. 

To efficiently complete these significant deals, a firm turns into an investment banker when it requires finance services. With some of the best benefits in the businesses, it is an extremely competitive industry.

How Does Investment Banking Work?

Investment banking offers services and serves as the middleman between businesses and investors and focuses mostly on shares and stock exchanges. 

Investment banking services help big businesses and organizations in developing a successful investment strategy that includes accurate financial instrument valuation.

When a company conducts an IPO or initial public offering, an investment bank purchases the majority of the shares immediately on the firm’s behalf.

The investment bank, which is now serving as a stand-in for the company then sells these shares on the market. The investment bank improves the company’s revenue in this way while also making sure that all governing rules are observed.

The investment bank makes money by marking up the initial price of shares when selling them to investors, helping the organization in making the most profit possible from this activity.

If a circumstance in the market emerges where the stock becomes overpriced, the investment bank also runs the risk of losing money by selling the stock at a lower price. 

An organization should assess its requirements and carefully consider all of its possibilities before seeking guidance from an investor banker. Before the company visits an investment bank, there are a few crucial considerations including the amount of capital being raised and the level of market competition. When the business has clarity in these areas, it can enlist the assistance of investment bankers to find new businesses to invest in.

  • Financial Modeling vs Investment Banking
  • Guide To Investment Banking
  • Financial Services vs Investment Banking
  • Investment Banking Business Models
  • Career Opportunities In Investment Banking
  • Investment Banking Industry
  • Investment Banking Courses in Delhi
  • Investment Banking Courses in Chennai
  • Investment Courses After 12th

Benefits of Investment Banking

Investment banking assists big businesses in a variety of ways to make crucial financial decisions and make sure they maximize revenues. That’s the reason, Investment banks are a prevalent financial institution among these businesses and even governments.

Here Are Some of the Advantages of Investment Banking:

  • Investment banks effectively manage their client and provide them with the information they require regarding the advantage and disadvantages of investing their money in other businesses or organizations.
  • These banks serve as a bridge between the company and the investor, ensuring a rise in financial capital by helping in major financial transactions like mergers and acquisitions.
  • It conducts an in-depth analysis of the deal and project that will be undertaken by its customer to ensure that the client’s money is invested safely and helps to reduce the risks involved with the mentioned deal or project.

What is Investment Banking Case Study?

You must have solved case studies during your investment banking training. 

Analyzing a business condition is done in case studies during investment banking interviews.

You would be provided with all the necessary data and have adequate time to examine broad case studies. There you would be asked for your opinion on business-related issues.

Your Task Includes,

  • Make the necessary deduction.
  • Investigate the matter, which is typically a client’s business.
  • Give suggestions for resolving the current issue along with an explanation.

Investment banking case studies are frequently used to evaluate a job candidate’s potential performance in real circumstances, where your interviewers would give you a problem and ask for a detailed recommendation.

By presenting them with a hypothetical scenario similar to those experiences while working in the field, your job is simply to analyze the scenario and give them justified reasons. 

Case studies are typically presented at the end of the application process, most frequently at the final interview or during the assessment center.

The majority of questions in investment banking case studies revolve around acquisition, capital raising, or business growth.

  • Financial Modeling Course
  • Digital Marketing Course
  • Technical Writing Course
  • Content Writing Course
  • Business Accounting And Taxation Course
  • CAT Coaching
  • Investment Banking Course
  • Data Analytics Course

What Are the Types of Case Studies?

Take home investment banking case study.

  • You will probably receive the case in advance so you have more time to work on it before the assessment day.
  • In the case of take-home case studies, you are given a few days to work on them, complete your analysis, and showcase your recommendation to the bankers over a 30-45 minutes presentation.
  • It involves a much deeper analysis including merger/LBO modeling, company procedures, and valuation.

On the Spot or Blind Investment Banking Case Study

  • On the day of your assessment center, the case can be presented to you blindly with little time for preparation.
  • These are given to you on the day of your interview and within an hour or two you are supposed to present it on the spot. 
  • The time split for this process would usually be 45-60 minutes of preparation, 10 minutes of presentation followed by a round of question and answer.
  • It would not involve such deep study.
  • Some case studies on investment banking may occasionally be given as a group task, where the employer will use this as an opportunity to examine the candidate’s analytical skills and teamwork qualities.

Why You Should Prepare for Investment Banking Case Study?

The theory behind these case studies is that because the qualification for various professions varies, bankers don’t trust the conventional method of interviewing applicants.

Case studies are preferred by banking recruiters as a better way to evaluate applicants because they show how you should perform in the workplace. 

You don’t need to worry about whether your response is right or wrong in this situation because the interviewer is more interested in how the candidate thinks and how well they can use logic and analysis to come up with an innovative answer to the challenge at that time.

Investment banking case study writers aim to inspire applicants to come up with their ideas and apply critical thinking.

Candidates for these positions must have a variety of skills, but problem-solving ability is one of the most important. 

Recruiters are interested in learning how you would approach difficult circumstances and use your intelligence, education, and professional experience to handle them successfully.

Additionally, candidates get an amazing chance to practice their other abilities including presentation, communication, and interpersonal skills.

These factors make case studies significantly more important than the other methods of evaluating applicants in the investment banking hiring process.

How to Prepare for Case Studies Before Assessment Day?

  • Read as much deal news as you can while preparing and going through the daily market and business news in popular publications.
  • Discover the many valuation methods, how they are calculated, and how they are evaluated then try out your calculations after watching YouTube videos or reading information on valuation methods.
  • You must prepare a structure using PowerPoint and Excel consistently, especially for modeling and valuation-based case studies.
  • Also, improve your familiarity with software like Microsoft Excel so that you can use spreadsheets effectively.
  • You should practice the kinds of questions you might get during your presentation. 
  • Real case study interview questions used by banks might not be available to you.
  • But, knowing that you need to practice, consider contacting a colleague or friend, or mentor you know who has gone through case study rounds for the types of questions they were asked.

How to Solve It and Perform Well During Assessment Day?

  • To solve the case study, take an organized strategy.
  • Before making a conclusion or deciding how to solve the problem, carefully analyze the case and the questions.
  • Professionally prepare Excel and PowerPoint while modeling case studies.
  • Every assentation you make should be supported by solid logical arguments, and the first few points should address that case’s most important issues.
  • Even if is not necessary, it would be advantageous to have a specialized understanding of the industry being studied.
  • Do not beat around the bush as you have limited time and hence be precise as you speak.

Investment Banking Case Study Examples and Answers

The decision-making case and the financial modeling case are two main types of case studies used in investment banking assessments.

Modeling – Investment Banking Case Study

Modeling case studies are typically take-home tasks that require you to perform straightforward valuation and financial modeling.

So rather than being a case study, it is more of a modeling exam.

The investment banker gives an overview of creating models as well as developing a variety of methods for an in-depth and useful understanding of the subject.

The modeling case study will either use a simpler merger or leveraged buyout model or a free cash flow to the business valuation. 

To assess whether the firms are overvalued or undervalued, you would be asked to examine their valuation multiples.

In most cases, you will be given a few days to finish your analysis. Then on the day of the interview, you must spend 30-45 minutes presenting your case to the bankers. 

Because you will have more time to work on it, the analysis will be considerably more in-depth than in a client case or decision-making case study.

Evaluating Strategic Alternative: Case Study 1 

To maximize shareholder value, a magazine publisher is deciding whether to sell, grow organically or make tiny “tuck-in” acquisitions. It is looking for an investment bank to assist it with its alternatives and has asked for a presentation from your company.

Given Materials: 

They would provide you with a firm summary with financial statements and five-year forecasts, a ten-page market analysis with main competitors, minor acquisition candidates, and recent transactions.

  • First, go through everything to get a sense of the industry, where it’s going, and how much this firm is worth in comparison.
  • Complete a quick assessment using publicly available rivals and prior transactions and a DCF.
  • Evaluate the figures provided by the value, the company’s potential for organic growth, and the availability of suitable targets for acquisition.

Decide what to do, in most cases it is advisable to say “Sell” unless the industry is expanding rapidly (Above 10% annually) the company is completely undervalued, or these are acquisition candidates that will increase revenue or profit by at least 20-30%.

After you have come to a decision, you must prepare your presentation and decide what to tell the bankers.

If you are analyzing scenarios like this during a 30-minute presentation, choose 10 slides with 3-4 important themes each and attempt to spend 3-4 minutes on each slide.

If you choose to write “Sell the company”, consider the following steps in preparing a presentation:

  • List the three main reasons for recommending selling
  • Overview of the industry- Is it expanding? Falling off? Or Being Inactive?
  • Position of the company in the industry? Leader or Second level position? Or is it strong or weak?
  • What would organic growth look like in five to ten years? How much larger or more valuable would the company be?
  • Prospective tuck-in acquisition candidates
  • Why organic growth and acquisition are not the answers.
  • Why selling now will generate the most shareholder value
  • Show prior transactions and public comparable valuations
  • Display the DCF output and the sensitivity chart valuation
  • Summary- State again that the best course of action is to sell your company right away and that neither organic development nor the acquisition of smaller firms would increase your company’s valuation in five to ten years.

Must Check,

  • Investment Banking Distance Learning Courses
  • Investment Banking Courses in South Africa
  • Investment Banking Courses After CA
  • Investment Banking Training Institutes in Hyderabad
  • Investment Banking Fresher Jobs
  • Investment Banking Courses in Bangalore
  • Investment Banking Courses in Noida
  • Investment Banking Courses in London
  • Investment Banking Jobs in Mumbai

Decision Making- Investment Banking Case Study

Case studies that include decision-making are more common than case studies that involve modeling.

In this kind of case study, the applicant is required to decide for their client and offer advice.

The client case study can center on locating financial sources or determining whether or not a proposed merger should go forward.

At the interview, you should be prepared for these questions. Because you will have a set amount of time in which to examine and present the case. You will be given a total of 45-60 minutes to prepare and beforehand 10 minutes presentation with a Q&A round.

investment banking case study interview questions

Case Study 1

A customer owns her company fully and wants to release some liquidity while keeping a stake in it (Worth £400 million) what suggestions would you provide the client to get the best possible price?

Given Materials:

A corporate overview and details about the company’s performance over the last three years are provided.

Examine all financial information thoroughly and forecast the company’s organic development.

Consider the breakdown of the present valuation if you are provided with the relevant facts.

Think about the client’s industry and the expected trends for that market.

  • How does the valuation stack up against others in the field?
  • Is the current valuation backed up by reliable industry forecasts?
  • Given the slow development of the industry, would it be wise to give up more equity?
  • Is it expected that this industry will keep growing?

Consider present customer portfolios, projects, etc., while deciding whether any actions could be performed to boost the company’s value.

Think about suggestions for the client’s negotiation strategy:

  • How much equity should they be prepared to give up?
  • What number should the client choose as their actual reserve price, in your opinion?

Case Study 2

A publicly traded firm contacts you in the hope to raise money. Analysts’ expectations were met by recent profits and the latest financial report, but the company’s market values are lowest throughout the year. The management of the company has developed a project that it hopes would significantly boost EBIDTA and is looking to raise funding for it. What should the business do to raise the required capital?

Given material:

A summary of the business and its financial statements will be provided to you to prepare for this question.

You must think about whether the organization should raise debt or stock.

Think about the market capitalization, share count, and share price:

  • How would the company be affected in this environment if it issued fresh shares?
  • In terms of dilution of ownership, would equity financing be an appropriate option?
  • How would the effect currently differ from what it would be if the share price were back to normal?

Then examine the provided financial statements:

  • Would increasing debt be a better course of action if they are actually under management’s predictions?
  • How much they could possibly raise?
  • What potential problems could a debt increase bring about?
  • How could the cost of interest be reduced?

Prepare your presentation by organizing your ideas clearly and go through your questions and thought process to get at your recommendation.

Also Check,

  • Investment Banking Courses in Canada
  • Investment Banking Courses in Germany
  • Investment Banking Training Programs
  • Best Courses For Investment Banking
  • Free Investment Banking Courses
  • Investment Banking Course Fees and Duration
  • Investment Banking Courses Eligibility
  • Investment Banking Training
  • Investment Banking Graduate Programs
  • Investment Banking Courses For Beginners
  • Investment Banking Analyst Jobs

Potential Acquisition: Case Study 3 

A software company is considering a large acquisition. It has chosen the company it wishes to acquire and has contacted a number of investment banks to obtain their thoughts on the transaction and how much they should pay. Based on these presentation, it will choose an advisor and decide what to do.

Two page summaries of the buyers and seller, each containing financial data as well as statistics and multiples for similar organization.

With a recommendation on whether to move forward with the acquisition and if so, how much to pay for the target, create five minute presentation.

For the very first, you should consider this two question to solve this,

  • Should they purchase that target business?
  • What price should they want for the target business?

For an example,

Let’s assume that the comparable companies are trading at EBITDA multiples that range from 4 to 8 times, with the median at 6 times and the 75th percentile at 7 times, respectively. You choose the 25th to 75th percentile range of 5x-7x and apply it to the target company’s $10 million EBITDA since the target company’s profit margins and revenue growth are comparable.

Therefore, the purchase price should range between $50 million and $70 million.

If you have access to a computer, you can also design a DSF, but if you are short of time, keep it straightforward and use multiples.

To answer the question “Should they buy?” take note of the following:

  • Will the buyer be able to purchase the seller with enough cash, debt, or stock issuances?
  • Will the vendor increase the buyer’s revenue and profit?
  • Will the buyer benefit from new consumers, new goods, new markets, or other kinds of benefits as a result of the seller’s acquisition?

After concluding these, you can complete your presentation.

Investment Banking Case Study: FAQs

Q. what is an investment banking case study in short terms.

By presenting candidates with a hypothetical scenario that is comparable to those they might face on the job, investment banking case studies are frequently used to evaluate how the candidate would function in real circumstances.

Q. Which skills are tested in investment banking case study interviews?

Candidates’ analytical and financial skills as well as problem-solving, presentation skills, critical thinking, and interpersonal skills are tested during investment banking interviews.

Q. Is there any way to practice investment banking case studies?

There are various tools, financial modeling online courses, and investment banking textbooks accessible to practice investment banking case studies. Additionally, there are certain career services offered at universities and institutions that provide investment banking programs with case studies.

Investment Banking Case Study: Conclusion

The opportunities to demonstrate your abilities and expertise to investment bankers are provided by investment banking case studies, which are a crucial component of an interview process. 

We have covered some of the investment banking case study examples that will help you in preparation for an investment banking interview.

No doubt it is a very competitive yet tough field to break into but we hope, through this article you achieve the success ladder in the investment banking industry.

investment banking case study interview questions

Author: Swati Varli

Leave a reply cancel reply.

Your email address will not be published. Required fields are marked *

Join Our Investment Banking Demo Class

  • Name This field is for validation purposes and should be left unchanged.

investment banking case study interview questions

Weekend Batch - 13th April 2024

Every Sat & Sun - 10:00 AM - 12:00 PM

7 Seats Left

You May Also Like To Read

Top 7 cyber security courses in chennai, tips to provide great sop writing services, list of content writing tools every content writer must use, simplified seo for successful digital marketing in 2024, general knowledge tips for cat exam, 40 must know digital marketing terms with definition, 7 effective pillars of best branding strategies, investment banking reconciliation – types, and stages, 6 successful marketing strategies of starbucks.

investment banking case study interview questions

  • 100% assured internships
  • Placement Assured Program
  • 500+ Hiring Partners
  • 100% Money Return Policy

Sunday Batch - 07th April 2024

Sunday 10:00 AM - 2:00 PM (IST)

Share Your Contact Details

  • Email This field is for validation purposes and should be left unchanged.

Weekdays Batch - 2nd April 2024

Tues & Thur - 8:00 PM - 9:30 PM (IST)

Saturday Batch - 30th March 2024

Saturday 10:00 AM - 1:00 PM (IST)

  • Comments This field is for validation purposes and should be left unchanged.

Download Course Brochure

  • Phone This field is for validation purposes and should be left unchanged.

Weekend Batch - 21st April 2024

Every Sat & Sun - 10:00 AM - 1:00 PM (IST)

Download Hiring Partners List

Download tools list, request for online demo.

Every Sat & Sun - 10:00 AM - 12:00 PM (IST)

  • Learn From An Expert
  • Steroids To Crack CAT Exam
  • Flip The Classroom Concept
  • Technology Driven

Request to Speak with MBA ADVISOR

  • Select Course * * Select Course Advanced Search Engine Optimization Business Accounting & Taxation Course Business Analytics Master Course Content Writing Master Course Digital Marketing Master Course Data Analytics Master Course Data Science Master Course Financial Modeling Course Investment Banking Course GST Practitioner Certification Course Technical Writing Master Course Tally Advanced Course Other Course
  • ADDITIONAL COMMENT

investment banking case study interview questions

Talk To An Agent

Talk to our agent, download student's success report, request for a callback, start hiring.

  • Company Name *
  • Hiring for * Select Program Content Writer Digital Marketer Data Analyst Financial Modellers Technical Writer Business Accounting & Taxation Search Engine Optimization Investment Banking
  • Attach Document * Max. file size: 256 MB.
  • Company Name * First
  • Select Program Select Program Business Accounting & Taxation Course Content Writing Master Course Digital Marketing Master Course Data Analytics Master Course Financial Modeling Course Search Engine Optimization Technical Writing Master Course
  • Select Members Select Mumbers 1-5 6-20 21-50 51-100 100+
  • Additional Comments

investment banking case study interview questions

I Pass the CFA Exam | Pass the CFA Exam & Other Finance Exams

  • You are here:
  • Home »
  • Investment Banking »

My Investment Banking Interview Questions and Answers: See How I Got My Final Round with Ease

  • By John / Stephanie

investment banking interview questions and answers

The following is a campus interview. It took place in my college in the beginning of the school year.

Investment Banking Interview Questions and Answers: Case Study

Interviewer : Hi Stephanie, I am Derek. Nice to meet you.

Stephanie : Hi Derek, thanks for coming over to Chicago to interview us.

Interviewer : Sure, no problem. I’d like to get out of Manhattan once in a while. I am from the Midwest myself (smile). Anyway, let me give you a short introduction. [Derek talked about where he graduated and that he is a third-year analyst]. OK, Stephanie, can you tell me about yourself?

Note: a subtle appreciation is returned with some useful information (that Derek is from the Midwest and maybe there is something he doesn’t like about Manhattan)

Stephanie : Sure. I am from Canada and towards the end of high school, I decided to study and explore places beyond my home country. Most of my friends went to either West Coast or East Coast, so I decided to go to the Midwest to get the most unique experience.

I was warmly welcome at my School, the University of Chicago where I major in Economics and Public Policy Studies.

I like Economics because I find it an intriguing way to analyze our daily lives in a systematic way, and I like Public Policy Studies because I want to understand how policies around the world drive and affect businesses and daily lives of common people.

I try to put my studies in practice by volunteering in the nearby public school right there (pointing to a direction) where I lead a group of fellow college students to hold scientific experiments with 30 kids every week, and it has been an amazing learning experience .

  • Grab the first commonality – Midwest and talk positively about it in an honest and natural way.
  • Include relevant keywords (marked bold ) in your self-introduction. Give an example you are proud of with specific information (volunteering with scientific experiment) and data (30 kids) to give it more substance. Non-verbal communications (the pointing towards the public school) also helps a lot.
  • End with a positive note (amazing experience) and lead your interviewer to ask a question about it. The answer should be well-prepared.

Interviewer: That’s great. Can you tell me about this volunteering service? I see that you mention it here in your resume.

Stephanie: Certainly. The program was launched by Reese Wilson, a graduate student who has founded a similar program when she was at Yale.

The idea is to provide an opportunity to 8-12 year old children in underprivileged school where scientific experiment is non-existent. As you may agree, the excitement of dissecting an insect or the discovery of how things work in nature is the best way to bring out the passion of learning among the kids.

Because I fully agree with Reese’s vision and I have never missed a session, I am quickly given the responsibility to lead a team of newly-joined volunteers on this program, from fund raising, brain storming scientific experiments with the children, to buying materials, coordinating and implementing the actual experiments… it involves a lot of teamwork and countless stressful moments , but the pay off is priceless.

(Pause and see if interviewer is interested in learning more. He is.)

  • This is a well-prepared question so it is easy to pack it with great keywords and lots of details to make it colorful and interesting.
  • Be specific — The mentioning of Reese Wilson is a subtle way to provide a reference. It’s useful that Reese comes from an Ivy League. Can elaborate by saying how Reese thinks of you as a such-and-such person. It’s as powerful as if Reese is telling the interviewer herself.
  • Here I highlighted the fact that I am pro-active, a fast-learner and someone with commitment; I have good teamwork and leadership skills; and I can handle stressful situations very similar to those in investment banks – all the important attributes for a junior investment banker.
  • For non-quantitative majors, you may want to pick an experience that can highlight your analytical / number-crunching skill.

Interviewer : The children’s stubbornness reminds me of one of my clients, ha ha. Yes, the Senator’s office (scanning the resume). That’s cool. Can you tell me more about it?

Interviewer : It’s great to study at a metropolitan city isn’t it, with all these exciting internships… Now tell me, why are you interested in Investment Banking, instead of, say, a non-profit organization?

Stephanie : I did think about NGOs and international organizations such as World Bank , and so I seek the advice from one of my mentors, Professor Truman who is an authority in international business . After a good conversation, he recommends me to start in a firm with the highest standard and most demanding clients , and after I have the best training under stressful situations , I can almost do anything thereafter.

I understand that investment bankers at [the firm] serve the CEOs of the biggest corporation and without questions these should be the toughest and smartest clients . I can also imagine the analysis and presentations done for them have to be the highest standard in order to get businesses from them. I also like to work with smart people and build a valuable network down the road so all-in-all I think Investment Bank is the best place to start my career.

  • Mention the world-class brand name such as World Bank and a prominent professor to give an impression that you are among the top-tiers.
  • Highlight the characteristics of i-banks and praising the industry as the most sought-after job at the same time.
  • Continue to include keywords associated with the analyst’s job to strengthen the sense of commonality and show that you understand the nature of their jobs.
  • Don’t forget to mention the specific firm you are interviewing with so it won’t sound too cookie-cutter.
  • You may notice that I didn’t sound as if I have a long-term commitment in the firm (I just said it’s an excellent place to start my career). Some people think this is a risky answer, but as a former analyst and interviewer I strong believe that honesty is more important than anything else in this type of questions. More than half of the analyst class leave i-banking after their 2 nd /3 rd year.

By this time, you should have covered all your strength:

  • A couple of specific experiences that highlight your leadership, teamwork and other attributes for a good investment banker.
  • A sincere yet flattering reason of why you choose investment bank as your career.

Interviewer : Excellent Stephanie. Do you have any question for me?

Stephanie : Yes. You asked me why I wanted to get into investment banking. Could you share with me why you are interested in investment banking and what’s your experience so far?

Note: This is one of the best questions because:

  • The interviewer is always interested in sharing his/her experience (otherwise the person won’t volunteer the time to interview in the first place).
  • The interviewer is most likely a good performer in the firm (otherwise he/she won’t be chosen as a representative) so he/she will and love to talk about the i-banking experience.
  • The answer is a good reference for your next rounds of interviews as you will certainly be asked with the same question again.

Interviewer : Anything else?

Stephanie : I understand that you are working in New York Headquarters. Have you thought about opportunities in regional offices, e.g. the Chicago office? What’s the difference between the two?

See how I pick up his casual comment on Manhattan and twist it into a relevant question.

This is another good question because (1) the answer is going to be useful for you; (2) it shows that you have a genuine interest in the firm and is thinking about the specifics.

Interviewer: It’s been great chatting with you. [Derek talks about the process, that I should hear from him within a week]. Here is my business card. Let’s keep in touch.

Stephanie : Thanks a lot for the interview and the sharing. I’ve learned a lot and I look forward to hearing from you soon.

Thanks! Do You have More Tips?

  • The 24 classic (and tough) i-banking interview questions
  • Difference between big, medium-sized and boutique investment banks
  • Most common questions from readers

Related Posts

Investment Banking FAQ: 25 Most Common Questions from Readers

Investment Banking Analyst Program: Info for Sophomores and Juniors

Investment Banking League Tables by Industry 2013

66 Investment Banking Interview Questions & Answers [Actually Asked By Recruiters]

Investment banking – a term that signifies prestige, challenging work, and rewarding compensation. As the driving force behind major financial transactions and corporate decisions, investment banking plays a crucial role in the global economy.

But with great influence comes immense competition , making the road to becoming an investment banker a highly selective process , with the interview stage often being the make-or-break point.

General Overview of Investment Banking Interviews

Investment banking interviews are notoriously rigorous and can range from phone screenings to first-round interviews and final Superdays. They typically cover behavioral and technical aspects, with some firms throwing in market-related questions and brain teasers for good measure. Let’s explore each of these categories.

Preparation for Investment Banking Interviews

To succeed in an investment banking interview, you need more than just a degree in finance or a knack for numbers. Preparation is paramount. You need to familiarize yourself with financial concepts, accounting principles, and investment banking jargon . Resources like investment banking textbooks, online courses, and mock interviews can prove invaluable.

Also, don’t forget to keep up-to-date with financial news and global market trends. Interviewers appreciate candidates who can discuss recent financial events with insight and acumen.

Common Fit/Behavioral Investment Banking Interview Questions

Fit or behavioral questions aim to assess whether you’d mesh well with the team and handle the pressures of investment banking. They often relate to your experiences, interests, and skills.

Here are some common fit questions :

  • Tell me about yourself.
  • Why have you chosen investment banking as a career?
  • Can you describe a situation where you demonstrated leadership?
  • How do you cope with high-stress situations?
  • Describe a situation where you had to work as part of a team.

Let’s dive deeper into the above behavioral questions and provide some examples, anecdotes, and tips for how you could approach these questions –

1. Tell me about yourself

This question is almost guaranteed to be the first one you’ll face in an interview. It may seem straightforward, but it’s your first opportunity to leave a strong impression. You should prepare a concise, compelling pitch that covers your educational background, relevant experience, and why you’re interested in investment banking . Remember to focus on your achievements and skills that make you suitable for the role.

Anecdote : Instead of just listing your qualifications, you could say something like: “During my internship at XYZ Corp, I was able to assist on an M&A deal which reinforced my interest in investment banking and helped me develop my financial modeling skills.”

2. Why have you chosen investment banking as a career?

Your interviewer wants to see your passion for the industry. Discuss what attracts you to investment banking, whether it’s the fast-paced nature of the work, the opportunity to work on high-stakes transactions, or the steep learning curve. Also, make sure your reasons align with the realities of the job – long hours, high stress, and all.

Tip : Show that you’ve done your research about the industry and are aware of the challenges, not just the glamorous parts. You might say, “I am drawn to investment banking for the rigorous analytical work and the chance to gain a deep understanding of businesses across different sectors.”

3. Can you describe a situation where you demonstrated leadership?

Interviewers ask this question to assess your leadership skills and teamwork. Talk about a time when you took the lead on a project or initiative, focusing on the actions you took and the results you achieved.

Anecdote : “In my previous role, I led a team of five to work on a complex project under tight deadlines. I delegated tasks effectively, motivated the team, and ensured we maintained open communication. Our team successfully completed the project on time, resulting in a 15% increase in efficiency.”

4. How do you cope with high-stress situations?

Investment banking is known for its high-stress environment. Your interviewer wants to know that you can handle this pressure. Discuss your stress management strategies such as prioritization, efficient time management, or mindfulness practices.

Tip : Provide an example of a high-stress situation you’ve faced, how you dealt with it, and what you learned from the experience. This showcases your problem-solving skills and resilience.

5. Describe a situation where you had to work as part of a team.

Investment banking involves a lot of teamwork. This question aims to assess your ability to work with others. Share an experience that shows your ability to collaborate, resolve conflicts, and achieve a common goal.

Anecdote : “During a group project at university, our team faced a disagreement over the project’s direction. To resolve this, I suggested we list the pros and cons of each approach and vote on the best one. This helped us move forward and ultimately, we received high praise for our work.”

When answering fit/behavioral questions, your goal should be to showcase your skills, experiences, and personal qualities that make you an excellent fit for investment banking. Be authentic, concise, and structured in your responses, and always tie your answers back to why you’d excel in the role.

Common Technical Investment Banking Interview Questions

The technical component of an investment banking interview is where your financial knowledge is put to the test. Common topics include financial modeling, valuations, M&A, LBOs, and DCF analysis.

Here are some example technical questions:

  • Can you explain the difference between enterprise value and equity value?
  • How do you value a company?
  • Can you walk me through a discounted cash flow (DCF) analysis?
  • How would you model an M&A deal?

Let’s dive deeper into the technical questions you might face during an investment banking interview –

1. Can you explain the difference between enterprise value and equity value?

Enterprise value (EV) and equity value (also known as market capitalization) are two measures used to determine a company’s value. Equity value is the total value of a company’s equity and can be calculated by multiplying the company’s share price by its number of shares outstanding. On the other hand, EV is a measure of a company’s total value, including equity and debt, minus cash and cash equivalents.

Tip : Remember, EV is a more comprehensive measure as it accounts for a company’s entire capital structure. It’s often used in scenarios like acquisitions, where the buyer would need to assume the company’s debts.

2. How do you value a company?

There are several methods to value a company, including the discounted cash flow (DCF) analysis, comparable company analysis (Comps), and precedent transaction analysis.

  • A DCF analysis values a company based on the present value of its projected free cash flows.
  • Comps involves comparing the company’s valuation multiples to those of similar companies in the market.
  • Precedent transaction analysis values a company based on recent M&A transactions involving similar companies.

Anecdote : During my finance coursework, I utilized the DCF method to value a technology start-up as part of a case study. This helped me understand the real-world application of these valuation models.

3. Can you walk me through a discounted cash flow (DCF) analysis?

A DCF analysis involves estimating a company’s future cash flows and then discounting them back to present value using an appropriate discount rate (usually the weighted average cost of capital, or WACC). The sum of these discounted cash flows gives us the enterprise value.

Tip : When walking through a DCF analysis, be sure to discuss key assumptions like revenue growth rates, operating margins, tax rates, and the discount rate. These assumptions can significantly impact the valuation.

4. How would you model an M&A deal?

Modeling an M&A deal involves several steps, including:

  • Determine the purchase price and how the deal will be financed.
  • Adjust the target’s balance sheet to reflect the transaction (add goodwill, write-ups, new debt, etc.).
  • Project financials for the combined company.
  • Calculate financial metrics and ratios to analyze the impact of the transaction.

Anecdote : As an intern, I had the opportunity to assist in building an M&A model for a client in the healthcare industry. This allowed me to understand the strategic and financial implications of M&A transactions.

50 More Technical Questions

Do you want to get ahead of the competition by mastering the questions that industry insiders swear by? Download our Free Comprehensive Guide on 50+ Technical Questions & Answers , which were actually asked in Investment Banking Interviews by top companies!

Technical questions can seem daunting, but with adequate preparation, you can handle them effectively. Make sure to brush up on your financial concepts and be comfortable with valuation methodologies . Try to relate these concepts to real-world scenarios or experiences when possible, as this shows your practical understanding of these principles.

Common Market-Related Investment Banking Interview Questions

Market-related questions probe your understanding of the broader financial landscape and your ability to form opinions based on current events. Staying abreast of the news and being able to articulate how different factors influence each other in the global economy will help you excel in this category.

Examples of these questions might include:

  • How do you see interest rates moving in the next 12 months and why?
  • What do you think about the current state of the stock market?
  • Can you explain how a change in the Federal Reserve’s policy might impact our clients?
  • How do recent economic events impact the M&A landscape?

Let’s dig deeper into some common market-related questions and provide you with more examples, anecdotes, and tips for handling these types of questions.

1. How do you see interest rates moving in the next 12 months and why?

This question tests your understanding of economic indicators and their impact on the market. You should base your answer on current economic trends, monetary policy, inflation rates, and other relevant data.

Tip : Be sure to articulate your thoughts clearly and justify your predictions with sound reasoning. For instance, you might say, “Given the recent signals from the Federal Reserve and current inflation trends, I believe we might see a gradual increase in interest rates over the next 12 months.”

2. What do you think about the current state of the stock market?

For this question, interviewers are looking for your insights on market conditions, trends, and possible future developments. Keep up-to-date with market news and be ready to discuss recent events, such as shifts in market indices, sector performance, or major announcements from central banks.

Anecdote : You could share an experience where a stock market event had an impact on a project or investment you were working on. This shows that you can apply market knowledge in practical scenarios.

3. Can you explain how a change in the Federal Reserve’s policy might impact our clients?

Interviewers use this type of question to assess your understanding of monetary policy and its effects on businesses. Changes in Federal Reserve policy can affect interest rates, which in turn can impact a company’s borrowing costs, capital structure decisions, and overall business strategy.

Tip : When answering, discuss both the direct impacts (e.g., changes to interest expenses for firms with debt) and the indirect effects (e.g., changes to consumer spending, which could affect a company’s revenues).

4. How do recent economic events impact the M&A landscape?

This question requires you to link macroeconomic factors to specific areas of investment banking, such as mergers and acquisitions. Economic events can influence M&A activity in several ways, such as changing the availability of financing, altering the valuation of potential targets, or impacting companies’ strategic decisions.

Anecdote : An example could be discussing how an economic recession led to an increase in consolidation within a specific industry.

To excel in answering market-related questions, you need to stay updated on current events , understand the macroeconomic landscape, and be able to link these factors to the world of investment banking. Also, remember to communicate your thoughts clearly and support your opinions with solid reasoning.

Common Brainteasers in Investment Banking Interviews

While they may seem out of place, brainteasers are sometimes used in investment banking interviews to test your problem-solving skills and how you handle pressure. Here are some examples:

How many square feet of pizza is eaten in the U.S. each year?

How many golf balls can fit in a boeing 747, if you were shrunk to the size of a pencil and put in a blender, how would you get out.

Let’s dig deeper into these brainteasers and provide you with more examples, anecdotes, and tips for handling these types of questions

This is an example of a market sizing question where you are expected to make reasonable assumptions to arrive at an estimate. Start by estimating the average size of a pizza, the frequency at which an average person eats pizza, and the population of the U.S.

Tip : Clearly articulate your thought process and the assumptions you are making. This shows your problem-solving approach and numerical reasoning skills.

This type of question tests your estimation skills and creativity. You’d need to estimate the volume of a Boeing 747 and the volume of a golf ball, then divide the former by the latter.

Anecdote : “During an internship interview, I was asked a similar question about filling a room with basketballs. Though it seemed offbeat at the time, it tested my ability to think on my feet and solve problems under pressure.”

This is an abstract brainteaser designed to assess your problem-solving skills and creativity. There is no right or wrong answer, but you’re expected to come up with a logical solution under pressure.

Tip : Try to stay calm and talk through your thought process clearly. For example, you could suggest tipping the blender over by using the pencil-size body as a lever.

When tackling brainteasers, it’s essential to stay composed, think logically, and articulate your thought process clearly. Interviewers are interested in how you approach the problem rather than the exact answer. Keep in mind that it’s okay to ask for a moment to think before you start answering. This shows that you’re thoughtful and won’t rush into complex problems without due consideration.

How to Close an Investment Banking Interview

Closing the interview effectively can leave a lasting impression. This is often the time when the interviewer asks if you have any questions. Use this opportunity to express interest and show you’ve done your research. Ask thoughtful questions about the role, the team, or recent transactions the firm has handled.

Post Interview Tips

After the interview, it’s good practice to send a thank you email to your interviewer. This shows appreciation for their time and can help you stand out from other candidates. If you haven’t heard back after a couple of weeks, it may be appropriate to follow up on your interview status .

Securing a role in investment banking is no small feat, and the interview process is just one of the many hurdles you’ll encounter on this career path. Remember, success in investment banking interviews isn’t just about regurgitating financial knowledge; it’s about demonstrating a deep understanding of financial principles, showcasing your problem-solving abilities, and proving your cultural fit within the company.

However, these interviews are also an opportunity for you to assess whether the role and the company are the right fit for you. After all, you’ll be investing a substantial amount of your time and energy into this job. Therefore, ensure that you’re as comfortable with them as they are with you.

A career in investment banking is undoubtedly challenging, but with the right preparation and mindset, you can navigate your interview confidently. So, take a deep breath, believe in yourself, and let your passion for investment banking shine through.

Frequently Asked Questions

Interviews typically include fit/behavioral questions, technical questions, market-related questions, and brainteasers to test a variety of skills and knowledge.

Brush up on financial modeling, valuation, corporate finance, and capital markets. Understand financial statements, recent deals, and current finance trends.

investment banking case study interview questions

CA Yash Jain

Bain & Co. 5000+ Students Trained in the field of Investment Banking, FRM & CFA

Leave a Reply Cancel reply

Your email address will not be published. Required fields are marked *

Save my name, email, and website in this browser for the next time I comment.

investment banking case study interview questions

50+ Investment Banking Questions

  • Real Questions from Actual Interviews
  • Expert Answers to Make you Stand out
  • Frequently asked by top Investment Banks
  • Both Practical and Theory

Get the Brochure in your Inbox

Get a call - back, upcoming batches.

  • Investment Banking Program
  • CFA Level 1 Coaching
  • CFA Level 2 Coaching
  • FRM Part 1 Coaching
  • FRM Part 2 Coaching
  • ACCA Coaching Program
  • Investment Banking Online Course
  • CFA Level 1 Online Coaching
  • CFA Level 2 Online Coaching
  • FRM Part 1 Online Coaching
  • FRM Part 2 Online Coaching
  • ACCA Online Course
  • CFA Reviews
  • FRM Reviews
  • ACCA Reviews

InterviewPrep

30 Investment Banking Interview Questions and Answers

Common Investment Banking interview questions, how to answer them, and example answers from a certified career coach.

investment banking case study interview questions

The high-stakes world of investment banking is known for its competitive nature, lucrative rewards, and rigorous interview process. To help you navigate this demanding landscape and make a lasting impression on your potential employer, we have compiled a list of common investment banking interview questions.

1. Can you explain the role of an investment banker in a merger or acquisition?

This question is designed to assess your understanding of the complex responsibilities and functions that an investment banker performs in the context of a merger or acquisition. It’s important for recruiters to know that you not only have the financial knowledge but also the strategic thinking and ability to navigate the intricacies of such transactions. Your response will demonstrate your expertise and showcase the value you could bring to the firm in executing successful deals.

Example: “An investment banker plays a critical role in mergers and acquisitions by providing expert guidance and support to the companies involved. In the initial stages, they help identify potential targets or buyers based on their client’s strategic objectives and financial goals. They conduct thorough due diligence, including analyzing the target company’s financials, operations, and market position, to determine its value and assess any risks associated with the transaction.

Once both parties agree to proceed, the investment banker assists in structuring the deal, negotiating terms, and preparing necessary documentation. This includes determining the optimal financing structure, whether it be cash, stock, or a combination of both, as well as addressing any regulatory requirements. Throughout the process, the investment banker acts as an intermediary between the buyer and seller, facilitating communication and ensuring that all parties are aligned on the deal’s progress.

Post-transaction, the investment banker may continue to provide advisory services, such as assisting with integration efforts or identifying further growth opportunities. Ultimately, the goal of an investment banker in a merger or acquisition is to ensure a smooth, efficient process that maximizes value for their client while minimizing risk.”

2. What is the difference between enterprise value and equity value?

Diving into the financial intricacies of a company is essential in investment banking, and understanding key valuation metrics like enterprise value and equity value is critical. By asking this question, interviewers assess your knowledge of financial concepts and your ability to analyze a company’s worth, which is a fundamental skill for making investment decisions and advising clients on mergers, acquisitions, and other financial transactions.

Example: “Enterprise value and equity value are two distinct valuation metrics used in finance to determine a company’s worth. Equity value, also known as market capitalization, represents the total value of all outstanding shares of a company’s stock. It is calculated by multiplying the current share price by the number of outstanding shares.

On the other hand, enterprise value provides a more comprehensive view of a company’s value by considering its entire capital structure, including debt and cash holdings. To calculate enterprise value, you start with the equity value and then add the company’s total debt and subtract its cash and cash equivalents. This metric gives a better understanding of the true value of a company, especially when comparing companies with different levels of debt and cash on their balance sheets.

While equity value focuses solely on the shareholders’ ownership stake, enterprise value takes into account the interests of all stakeholders, such as debt holders and preferred shareholders, providing a more holistic perspective on a company’s worth.”

3. How do you calculate the weighted average cost of capital (WACC)?

Financial acumen and analytical skills are essential in investment banking. By asking you to explain the calculation of the weighted average cost of capital (WACC), interviewers want to gauge your understanding of this key financial concept. Knowing how to calculate the WACC is vital for evaluating investment opportunities and determining a company’s cost of financing, which ultimately impacts the overall financial performance and decision-making process in investment banking.

Example: “To calculate the weighted average cost of capital (WACC), you need to consider both the cost of equity and the cost of debt for a company, as well as their respective proportions in the overall capital structure. The formula for WACC is:

WACC = (E/V) * Re + (D/V) * Rd * (1 – Tc)

Where: – E represents the market value of equity – D represents the market value of debt – V is the total value of the firm’s capital (V = E + D) – Re is the cost of equity, which can be calculated using models like the Capital Asset Pricing Model (CAPM) – Rd is the cost of debt, typically represented by the yield-to-maturity on the company’s outstanding bonds or loans – Tc is the corporate tax rate

To compute WACC, first determine the proportion of equity (E/V) and debt (D/V) in the capital structure. Then, multiply these proportions by their respective costs (Re and Rd). Finally, adjust the cost of debt component for the tax shield effect by multiplying it with (1 – Tc). Summing up the results gives you the WACC, which reflects the average rate that a company expects to pay to finance its assets.”

4. Describe your experience with financial modeling.

Financial modeling is the backbone of investment banking. It’s the process of creating abstract representations of real-world financial situations in order to make informed decisions about investments, acquisitions, mergers, and other transactions. Interviewers want to know that you have the skills and experience to build, analyze and interpret these models, as it demonstrates your ability to provide valuable insights and contribute to the success of the bank’s business operations.

Example: “During my time as a financial analyst at XYZ Corporation, I gained extensive experience in building and maintaining financial models for various purposes. One of the most significant projects I worked on involved creating a detailed valuation model for a potential acquisition target. This required me to analyze historical financial statements, project future cash flows, and calculate key valuation metrics such as discounted cash flow (DCF) and enterprise value.

I also collaborated with cross-functional teams to gather necessary inputs and assumptions, ensuring that our model accurately reflected the company’s operations and growth prospects. Throughout this process, I honed my skills in Excel, including advanced functions like VLOOKUP, INDEX-MATCH, and pivot tables, which allowed me to efficiently manipulate large datasets and present findings in a clear, concise manner. Ultimately, my work contributed to the successful completion of the acquisition, demonstrating my ability to apply financial modeling techniques to real-world situations in investment banking.”

5. Explain the concept of due diligence in the context of investment banking.

This question is designed to assess your understanding of the critical processes that take place within the investment banking sector. Due diligence is a vital aspect of any financial transaction, ensuring that all information is accurate, and risks are identified and mitigated. Your ability to explain this concept demonstrates not only your knowledge of the industry but also your attention to detail and commitment to thorough research—skills that are highly valued in the world of investment banking.

Example: “Due diligence in investment banking refers to the comprehensive research and analysis conducted on a company or asset before entering into a financial transaction, such as mergers, acquisitions, or issuing securities. The primary goal of due diligence is to identify potential risks, evaluate the target’s financial health, and ensure that all relevant information is disclosed to make informed decisions.

The process typically involves examining various aspects of the target entity, including its financial statements, management team, legal compliance, market position, and competitive landscape. Investment bankers collaborate with other professionals like lawyers, accountants, and industry experts to gather and analyze this data. Ultimately, thorough due diligence helps minimize risk for both the buyer and seller, ensuring a successful transaction and protecting the interests of all parties involved.”

6. What are some common valuation methods used in investment banking?

Exploring your knowledge of valuation methods is important to interviewers because evaluating a company’s worth is a critical part of investment banking. This includes mergers and acquisitions, initial public offerings, and other financial transactions. Being familiar with techniques like discounted cash flow, comparable company analysis, and precedent transactions demonstrates that you have the foundational skills needed to excel in the field of investment banking.

Example: “There are several valuation methods commonly used in investment banking, with three of the most widely employed being discounted cash flow (DCF), comparable company analysis (CCA), and precedent transaction analysis (PTA).

The DCF method involves projecting a company’s future cash flows and discounting them back to their present value using an appropriate discount rate. This approach is based on the principle that the value of a business is equal to the sum of its expected future cash flows.

Comparable company analysis, on the other hand, involves comparing the target company to similar publicly traded companies within the same industry. Key financial ratios and multiples, such as price-to-earnings (P/E) and enterprise value-to-EBITDA (EV/EBITDA), are calculated for each company and then applied to the target firm to estimate its value.

Precedent transaction analysis entails examining previous transactions involving companies similar to the target firm. The acquired companies’ valuation multiples from these deals are analyzed and adjusted to account for differences between the target and precedent firms. These adjusted multiples are then applied to the target company to derive its estimated value.

Each method has its strengths and weaknesses, and often, investment bankers use a combination of these approaches to arrive at a more comprehensive valuation.”

7. Can you discuss a recent M&A deal that caught your attention? Why did it interest you?

Your interviewer wants to gauge your passion for the world of finance and your ability to critically analyze and understand the complexities of mergers and acquisitions. This question reveals your ability to stay current with industry news, your understanding of market trends, and your ability to dissect the rationale behind a deal—all essential skills for an investment banker.

Example: “One recent M&A deal that caught my attention was the acquisition of Slack by Salesforce, announced in December 2020 and completed in July 2021. This deal interested me for several reasons. Firstly, it showcased how technology companies are seeking to expand their product offerings and strengthen their positions in the market, especially during a time when remote work has become increasingly prevalent due to the pandemic.

The strategic rationale behind this acquisition is compelling, as it allows Salesforce to integrate Slack’s communication platform into its existing suite of customer relationship management (CRM) tools. This integration aims to enhance collaboration capabilities for Salesforce users and create a more seamless experience across various business functions. The deal also highlights the growing importance of cloud-based software solutions and the competitive landscape within the industry, with major players like Microsoft and Google continuously expanding their product portfolios.

This M&A transaction demonstrates the ongoing trend of consolidation within the tech sector and emphasizes the need for companies to adapt and innovate to maintain a competitive edge in an ever-evolving market. As someone interested in investment banking, observing such high-profile deals provides valuable insights into the strategies employed by leading firms and the factors driving M&A activity in the technology space.”

8. How would you assess the creditworthiness of a potential client?

Analyzing a client’s creditworthiness is a critical aspect of investment banking, as it helps determine the level of risk involved in extending credit, loans, or making investments. By asking this question, interviewers want to gauge your understanding of key financial indicators, your ability to interpret financial statements, and your overall analytical skills. They’re interested in finding out if you can make informed decisions that protect the bank’s interests while fostering growth and maintaining client relationships.

Example: “When assessing the creditworthiness of a potential client, I would first analyze their financial statements to gain insight into their overall financial health. This includes examining key ratios such as debt-to-equity, current ratio, and interest coverage ratio. These metrics provide valuable information about the client’s ability to meet their short-term obligations, leverage levels, and capacity to service their debt.

After evaluating the financials, I would consider qualitative factors that could impact the client’s creditworthiness. This may involve analyzing the company’s management team, industry trends, competitive landscape, and any regulatory or geopolitical risks associated with their operations. Combining both quantitative and qualitative assessments allows for a comprehensive understanding of the client’s credit profile and helps determine whether extending credit is a prudent decision for the bank.”

9. What factors do you consider when determining whether to recommend debt or equity financing for a client?

When working in investment banking, your ability to make informed recommendations to clients is essential. By asking this question, interviewers want to know that you understand the key factors that should be considered when deciding between debt or equity financing. This demonstrates your knowledge of the industry, your analytical skills, and your ability to make strategic decisions that will benefit your clients and their businesses.

Example: “When determining whether to recommend debt or equity financing for a client, I consider several factors. First, I assess the company’s financial health by examining its balance sheet, cash flow, and profitability ratios. A strong financial position may make debt financing more attractive, as the company can likely secure favorable interest rates and terms.

Another factor is the company’s growth stage and future plans. If the business is in an early stage with high growth potential, equity financing might be more suitable, as it allows investors to share both risks and rewards. Conversely, if the company is well-established and looking to finance specific projects or acquisitions, debt financing could be more appropriate.

I also take into account market conditions and investor sentiment. In a bullish market, equity financing may be more appealing due to higher valuations, while in a bearish market, companies might prefer debt financing to avoid diluting ownership at lower valuations. Ultimately, my recommendation will depend on a comprehensive analysis of these factors and the client’s strategic objectives.”

10. Describe a time when you had to work under tight deadlines on a complex project.

Investment banking is synonymous with high-pressure, fast-paced environments where every minute counts. By asking you to describe a time when you worked under tight deadlines and on a complex project, interviewers aim to gain insight into your ability to manage stress, prioritize tasks, and demonstrate resilience in challenging situations—all of which are essential attributes for a successful investment banker.

Example: “During my time as a financial analyst at XYZ firm, I was assigned to work on the valuation of a mid-sized company that our client was considering acquiring. The project required extensive research and analysis within a tight two-week deadline due to the competitive nature of the deal.

To manage this complex task, I quickly divided the workload into smaller components and prioritized them based on their importance and time sensitivity. I collaborated closely with my team members, ensuring clear communication and efficient delegation of tasks. We worked diligently, often putting in extra hours to meet the deadline.

Despite the challenging timeline, we successfully completed the valuation report, which included detailed financial modeling, industry analysis, and risk assessment. Our thorough analysis enabled our client to make an informed decision about the acquisition, ultimately leading to a successful transaction. This experience taught me the value of effective time management, teamwork, and adaptability when working under pressure in the fast-paced world of investment banking.”

11. Have you ever worked on an initial public offering (IPO)? If so, what was your role in the process?

Inquiring about your experience with an initial public offering (IPO) allows interviewers to gauge your knowledge of the investment banking industry and assess your ability to navigate complex financial transactions. Your response demonstrates your capacity to contribute to high-stakes projects and highlights your understanding of the processes involved in taking a company public, which is a critical aspect of investment banking services.

Example: “Yes, I had the opportunity to work on an initial public offering (IPO) during my time as an analyst at a boutique investment bank. My role in the process was primarily focused on conducting due diligence and preparing the financial models for the company going public.

I collaborated with other team members to gather and analyze historical financial data, industry trends, and comparable company valuations. This information allowed us to develop accurate financial projections and determine an appropriate valuation range for the IPO. Additionally, I assisted in drafting sections of the prospectus, ensuring that all relevant financial information was accurately presented and compliant with regulatory requirements.

Throughout the IPO process, I also participated in meetings with the client’s management team and legal counsel, addressing any concerns or questions they had regarding the financial aspects of the transaction. My involvement in this project not only provided me with valuable experience in the IPO process but also reinforced the importance of teamwork and effective communication when working towards a common goal.”

12. What is the importance of industry analysis in investment banking?

Understanding industry trends and dynamics is crucial in investment banking, as it helps professionals make informed decisions when advising clients, structuring deals, and managing portfolios. By demonstrating your knowledge of industry analysis, you show your potential employer that you have the ability to identify opportunities, assess risks, and provide valuable insights to clients, ultimately contributing to the success of the bank and its clients.

Example: “Industry analysis plays a vital role in investment banking as it helps identify trends, opportunities, and risks within specific sectors. This information is essential for making informed decisions when advising clients on mergers and acquisitions, capital raising, or restructuring activities.

A thorough industry analysis allows investment bankers to understand the competitive landscape, market dynamics, and growth potential of various industries. This knowledge enables them to provide valuable insights to clients regarding potential targets for M&A transactions, suitable financing options, and optimal deal structures. Additionally, by staying updated on industry developments, investment bankers can anticipate changes that may impact their clients’ businesses and proactively suggest strategic adjustments to maintain competitiveness and maximize shareholder value.”

13. How do you stay updated on market trends and news relevant to your clients’ industries?

Staying current on market trends and industry news is vital for investment bankers. Your clients rely on you to provide them with valuable insights and guidance, which means you need to be well-informed and able to make sound decisions based on the latest information. By asking this question, interviewers want to ensure that you are proactive in staying up-to-date and knowledgeable, demonstrating your dedication to providing the best possible advice to your clients.

Example: “Staying updated on market trends and news is essential in investment banking, as it allows me to provide well-informed advice to my clients. I have developed a routine that includes daily monitoring of financial news outlets such as Bloomberg, The Wall Street Journal, and Financial Times. Additionally, I subscribe to industry-specific newsletters and follow relevant social media accounts to ensure I receive real-time updates.

Furthermore, I participate in webinars, conferences, and networking events related to my clients’ industries, which helps me gain insights from experts and peers. This combination of resources enables me to stay informed about the latest developments and anticipate potential opportunities or challenges for my clients, ultimately allowing me to better serve their needs and contribute to their success.”

14. Can you explain the concept of leveraged buyouts (LBOs)?

Financial knowledge and analytical skills are critical in investment banking roles. Interviewers ask this question to evaluate your understanding of complex financial concepts, such as leveraged buyouts (LBOs), as well as your ability to explain them in a clear and concise manner. This helps them assess whether you have the necessary expertise and communication skills to succeed in a client-facing, high-pressure environment.

Example: “Leveraged buyouts (LBOs) are a financial transaction in which an investor or group of investors acquire a controlling stake in a company primarily using borrowed funds. The acquired company’s assets and cash flows serve as collateral for the debt, and its future earnings are used to repay the loans.

The main goal of an LBO is to create value by improving the target company’s operations or financial structure, ultimately leading to a profitable exit through a sale or public offering. This strategy allows investors to achieve high returns on their equity investment while minimizing the amount of capital they need to contribute upfront. However, it also increases the risk due to the higher levels of debt involved, making it essential for the acquiring party to carefully assess the target company’s ability to generate sufficient cash flow to service the debt obligations.”

15. What is the difference between a cash flow statement, income statement, and balance sheet?

Financial knowledge and understanding are at the core of investment banking. By asking this question, interviewers are assessing your grasp of basic financial statements and their purposes. They want to ensure you have a strong foundation in finance, as you’ll be working with these statements regularly to analyze a company’s financial health and make informed recommendations to clients.

Example: “A cash flow statement, income statement, and balance sheet are three essential financial statements that provide insight into a company’s financial health. Each serves a distinct purpose in analyzing different aspects of the business.

The cash flow statement tracks the inflow and outflow of cash within a specific period, typically divided into operating, investing, and financing activities. It helps assess a company’s liquidity and its ability to meet short-term obligations.

The income statement, also known as the profit and loss statement, summarizes a company’s revenues, expenses, and net income over a specified time frame. It provides an understanding of the firm’s profitability by showing how much money it makes after accounting for all costs.

The balance sheet is a snapshot of a company’s financial position at a given point in time. It presents assets, liabilities, and shareholders’ equity, following the equation: Assets = Liabilities + Shareholders’ Equity. The balance sheet offers insights into a company’s solvency and overall financial stability.

Together, these financial statements offer a comprehensive view of a company’s performance and financial health, allowing investors and analysts to make informed decisions.”

16. Describe your experience working with cross-functional teams on deals or projects.

As an investment banker, you’ll often find yourself working alongside diverse teams, including legal, financial, and management specialists. This is essential because deals and projects in this field require an array of expertise for successful execution. Interviewers want to ensure that you have experience collaborating with different departments and can effectively contribute to and navigate complex, cross-functional teams.

Example: “During my time as an investment banking analyst, I had the opportunity to work on a merger deal involving two major companies in the retail industry. This project required close collaboration with cross-functional teams, including legal, accounting, and strategy departments.

Throughout the process, I was responsible for coordinating financial analysis and valuation tasks while ensuring seamless communication between all parties involved. To achieve this, we held regular meetings to discuss progress, address any concerns, and align our objectives. Additionally, I created detailed reports and presentations that were shared with both internal and external stakeholders to keep everyone informed of the deal’s status.

This experience taught me the importance of clear communication, adaptability, and teamwork when working with diverse groups of professionals. It also highlighted the value of understanding each team member’s expertise and leveraging their unique skills to drive successful outcomes in complex deals or projects.”

17. What strategies do you use to build strong relationships with clients?

Strong client relationships are the backbone of investment banking. The ability to build trust and maintain long-term connections with clients is essential for success and business growth in this field. Interviewers want to know if you possess the interpersonal skills, professionalism, and strategic thinking required to foster these relationships and ensure client satisfaction.

Example: “Building strong relationships with clients in investment banking is essential for long-term success and trust. One strategy I use is active listening, which involves giving my full attention to the client’s concerns and needs. This helps me understand their financial goals and risk tolerance better, allowing me to provide tailored advice and solutions.

Another strategy is maintaining regular communication through various channels such as phone calls, emails, or face-to-face meetings. This ensures that clients are kept informed about market developments, potential opportunities, and any changes in their portfolio. Additionally, it demonstrates my commitment to their financial well-being and fosters a sense of reliability.

These strategies have proven effective in establishing rapport and building lasting relationships with clients, ultimately leading to increased satisfaction and loyalty.”

18. How do you handle confidential information in the workplace?

Handling sensitive information with care and discretion is a critical aspect of working in investment banking. Hiring managers need to know that you can be trusted with important financial data, client relationships, and company secrets. Demonstrating your ability to maintain confidentiality and protect valuable information shows that you understand the importance of discretion in the financial industry and can operate professionally in high-stakes situations.

Example: “Handling confidential information is a critical aspect of working in investment banking, and I take this responsibility very seriously. First and foremost, I adhere to the company’s policies and guidelines regarding confidentiality and data protection. This includes using secure communication channels when discussing sensitive information with colleagues or clients and ensuring that any physical documents containing confidential data are stored securely.

Furthermore, I am cautious about sharing information on a need-to-know basis, only disclosing details to relevant parties who require access for their specific roles. Additionally, I make it a point to stay updated on industry best practices and regulations related to data privacy and security, which helps me maintain vigilance in safeguarding confidential information at all times.”

19. What is your approach to managing risk in investment banking transactions?

A key aspect of working in investment banking is the ability to evaluate and manage risk. The interviewer wants to know that you have a solid understanding of risk management principles and can apply them to various transactions. Demonstrating your ability to balance potential rewards with potential risks will show your potential employer that you have the analytical skills and sound decision-making abilities needed in the high-stakes world of investment banking.

Example: “My approach to managing risk in investment banking transactions involves a combination of thorough due diligence and continuous monitoring. First, I conduct comprehensive research on the client’s financial position, industry trends, and market conditions to identify potential risks associated with the transaction. This includes analyzing financial statements, assessing creditworthiness, and evaluating the competitive landscape.

Once the initial assessment is complete, I work closely with other team members to structure the transaction in a way that mitigates identified risks while still meeting the client’s objectives. This may involve adjusting deal terms, incorporating protective covenants, or diversifying the investment portfolio. Throughout the transaction process, I maintain open communication with all stakeholders to ensure any emerging risks are promptly addressed and managed effectively. This proactive approach helps minimize potential losses and supports the overall success of the investment banking transaction.”

20. Can you provide an example of a successful pitch you made to a client?

When it comes to investment banking, the ability to persuade clients and close deals is critical to success. By asking for an example of a successful pitch, interviewers aim to evaluate your communication skills, salesmanship, and ability to build trust with clients. They want to know that you can effectively present information and opportunities that align with the client’s needs, ultimately leading to a beneficial partnership for both parties. Sharing a specific example demonstrates your experience and effectiveness in this essential aspect of the job.

Example: “Certainly, I recall a pitch I made to a mid-sized manufacturing company looking for growth opportunities. They were considering acquiring one of their competitors to expand their market share and product offerings. After conducting thorough research on the target company and analyzing its financials, I identified potential synergies that would result from the acquisition.

During the pitch meeting, I presented my findings in a clear and concise manner, highlighting the strategic benefits of the acquisition, such as increased market share, cost savings through economies of scale, and enhanced product diversification. Additionally, I provided a detailed valuation analysis, demonstrating how the deal could create value for both companies involved.

The client appreciated the depth of our analysis and the compelling case we built for the acquisition. As a result, they decided to move forward with the transaction, and our team was engaged to advise them throughout the process. The acquisition ultimately proved successful, leading to significant growth for the client and solidifying our relationship for future business opportunities.”

21. What is the significance of EBITDA in investment banking?

The importance of EBITDA (earnings before interest, taxes, depreciation, and amortization) lies in its ability to provide a clear picture of a company’s operational performance. In investment banking, EBITDA is a widely-used metric to analyze and compare the financial health of different companies, as it eliminates the effects of financing and accounting decisions. By asking this question, interviewers aim to assess your understanding of financial analysis and your ability to evaluate potential investments effectively.

Example: “EBITDA, or Earnings Before Interest, Taxes, Depreciation, and Amortization, is a widely used financial metric in investment banking to evaluate a company’s operating performance. It serves as an indicator of the company’s ability to generate cash flow from its core operations, excluding non-operating expenses such as interest, taxes, and non-cash items like depreciation and amortization.

The significance of EBITDA in investment banking lies in its usefulness for comparing companies across industries with varying capital structures and tax environments. Since it focuses on operational profitability, it allows analysts and investors to assess a company’s underlying business performance without being influenced by factors unrelated to its core operations. Additionally, EBITDA is often used in valuation multiples, such as EV/EBITDA, which helps determine a company’s relative value compared to its peers. This information can be critical when making investment decisions or advising clients on mergers and acquisitions.”

22. How do you prioritize tasks when working on multiple deals simultaneously?

Time management and prioritization are key skills for investment bankers, who often juggle multiple high-stakes deals at once. When asking this question, interviewers want to see that you can effectively allocate your time and energy, ensuring that each project receives the attention it needs without compromising the quality of your work. They also want to assess your ability to adapt to changing circumstances and reprioritize as needed.

Example: “When working on multiple deals simultaneously, effective prioritization is essential to ensure that each deal progresses efficiently and deadlines are met. My approach involves assessing the urgency, complexity, and importance of each task across all deals.

I start by creating a comprehensive list of tasks for each deal, including their respective deadlines and any dependencies between them. Next, I evaluate the urgency of each task based on factors such as time sensitivity, client expectations, and potential impact on the overall deal process. This helps me identify which tasks require immediate attention and which can be scheduled later.

Once I have a clear understanding of the urgency, I consider the complexity and importance of each task. For instance, tasks that involve critical decision-making or have significant consequences on the deal outcome would be given higher priority. To manage my workload effectively, I also allocate time for regular progress reviews and communication with team members and clients, ensuring everyone stays informed and aligned throughout the process. This structured approach allows me to maintain focus and deliver high-quality work even when managing multiple deals at once.”

23. What is your experience with regulatory compliance in the investment banking sector?

Navigating the complex world of financial regulations is a critical part of investment banking. By asking this question, interviewers want to gauge your understanding of the regulatory environment and ensure that you possess the skills to maintain compliance. Demonstrating your experience in this area helps assure potential employers that you can protect the firm’s reputation, minimize legal risks, and foster a culture of transparency and ethical behavior.

Example: “During my time as an investment banking analyst, I have gained substantial experience in regulatory compliance. My primary responsibility was to ensure that our team adhered to the regulations set forth by governing bodies such as the SEC and FINRA. This involved staying up-to-date with any changes in regulations and implementing necessary adjustments within our processes.

I worked closely with our legal and compliance departments to review transactions and client communications for adherence to applicable rules. Additionally, I participated in internal audits and assisted in preparing documentation for external audits. Through these experiences, I developed a strong understanding of the importance of regulatory compliance in maintaining the integrity of the financial industry and protecting both our clients and the firm from potential risks.”

24. Describe a challenging negotiation you were involved in and how you resolved it.

Navigating complex negotiations is a critical skill in the world of investment banking. Whether you’re dealing with mergers and acquisitions, capital raising, or other financial transactions, your ability to find common ground and reach mutually beneficial agreements is paramount. Interviewers want to gauge your negotiation skills, as well as your ability to think strategically, communicate effectively, and maintain composure under pressure. Sharing a specific example of a challenging negotiation helps to demonstrate your proficiency in these areas.

Example: “During my time as an investment banking analyst, I was involved in a merger negotiation between two mid-sized companies. The main challenge we faced was that both parties had different valuations of their respective businesses and were unwilling to compromise on the deal structure.

To resolve this issue, our team conducted thorough due diligence and financial analysis to establish a fair valuation for each company. We then presented our findings to both parties, highlighting the key drivers behind the valuations and emphasizing the potential synergies they could achieve through the merger. This helped create a common ground for further negotiations.

As discussions progressed, we facilitated open communication between the parties and encouraged them to focus on their long-term strategic goals rather than short-term gains. Eventually, we reached a mutually beneficial agreement by proposing a creative deal structure that allowed both companies to share risks and rewards proportionately. This successful resolution not only demonstrated my ability to navigate complex negotiations but also showcased the importance of effective communication and collaboration in achieving desired outcomes.”

25. What is the role of an investment banker in a debt restructuring process?

Demonstrating your knowledge of the role of an investment banker in a debt restructuring process is essential for showcasing your expertise in this field. Interviewers want to know if you have a thorough understanding of the various responsibilities and tasks an investment banker must undertake, such as advising clients on the best course of action, negotiating with creditors, and designing a feasible financial restructuring plan. This question will also help them assess your ability to contribute effectively in high-pressure situations involving distressed companies.

Example: “An investment banker plays a critical role in the debt restructuring process by acting as an intermediary between the company facing financial difficulties and its creditors. Their primary objective is to negotiate favorable terms for both parties, ensuring that the company can continue operations while satisfying creditor demands.

To achieve this, the investment banker first conducts a thorough analysis of the company’s financial situation, including cash flow, assets, liabilities, and overall debt structure. Based on this assessment, they develop a comprehensive restructuring plan that may involve extending loan maturities, reducing interest rates, or converting debt into equity. The investment banker then presents this proposal to the creditors and facilitates negotiations to reach a mutually agreeable solution.

Throughout the process, the investment banker also provides strategic advice to the company’s management team, helping them navigate complex financial decisions and implement operational changes necessary for long-term success. Ultimately, their expertise and guidance contribute significantly to the successful execution of the debt restructuring process, enabling the company to regain financial stability and focus on growth opportunities.”

26. How do you handle stress and long working hours in the investment banking industry?

Investment banking can be an incredibly demanding and high-pressure field, with tight deadlines and high stakes. Employers want to make sure that you’re mentally equipped to handle this environment—whether it means staying calm under pressure, showing strong resilience, or demonstrating effective time management and prioritization skills. They’re looking for someone who can maintain a high level of productivity and accuracy, even when faced with long hours and challenging situations.

Example: “Handling stress and long working hours in the investment banking industry requires a combination of effective time management, prioritization, and self-care. I’ve developed strategies to maintain my productivity and well-being during demanding periods.

One key aspect is setting realistic goals and breaking tasks into smaller, manageable steps. This helps me stay focused on what needs to be accomplished while avoiding feeling overwhelmed. Additionally, I prioritize tasks based on their urgency and importance, ensuring that critical projects receive the attention they deserve.

To maintain my well-being, I make sure to take short breaks throughout the day to recharge mentally and physically. Even brief moments away from work can help improve focus and reduce stress. Outside of work, I engage in regular exercise and maintain a healthy diet, which contributes to overall resilience and energy levels. Lastly, I find it essential to have a strong support network of colleagues, friends, and family who understand the demands of the industry and provide encouragement when needed.”

27. Can you explain the concept of a fairness opinion?

Investment banking interviews often delve into your understanding of the industry’s technical concepts and your ability to communicate them clearly. A fairness opinion is a key term in the world of mergers and acquisitions, and it’s important for candidates to demonstrate their knowledge of such concepts. By asking about fairness opinions, interviewers can assess your familiarity with the field and your ability to articulate complex ideas to clients and colleagues.

Example: “A fairness opinion is a professional evaluation provided by an investment bank or financial advisory firm, assessing the financial terms of a proposed transaction such as a merger, acquisition, or divestiture. The primary purpose of a fairness opinion is to determine whether the deal’s terms are fair and reasonable from a financial standpoint for the shareholders involved.

The process typically involves analyzing various aspects of the transaction, including valuation multiples, comparable transactions, and discounted cash flow analysis. This assessment helps the company’s board of directors make informed decisions regarding the transaction and provides them with an additional layer of protection against potential legal challenges from shareholders who may argue that the deal undervalues their shares or is not in their best interest.”

28. Describe your experience with syndicated loans.

Interviewers ask this question because they want to gauge your understanding of syndicated loans, a key component of investment banking. Additionally, they want to know if you have experience in structuring, negotiating, and managing these types of loans. Your response will provide insight into your ability to work on complex transactions and collaborate with various stakeholders in the lending process.

Example: “During my time at XYZ Bank, I was actively involved in the syndicated loans team, where we managed several large-scale transactions. My role primarily focused on conducting due diligence and preparing credit analysis reports for potential borrowers.

I worked closely with our origination team to identify suitable lenders for each transaction, taking into consideration their risk appetite and industry preferences. Additionally, I participated in structuring the loan facilities, including determining pricing, tenor, and covenants based on market conditions and borrower requirements.

This experience allowed me to develop a deep understanding of the syndicated loan process, from origination to closing, and enabled me to build strong relationships with various stakeholders, such as borrowers, lenders, and legal counsel. It also taught me the importance of effective communication and collaboration in successfully executing complex financial transactions.”

29. What are some key considerations when advising clients on cross-border transactions?

The complexities and nuances of cross-border transactions require an investment banker to be well-versed in a variety of factors. Interviewers want to ensure that you have a comprehensive understanding of the challenges and opportunities that arise in these situations. They’re interested in your ability to navigate regulatory frameworks, cultural differences, currency fluctuations, and taxation issues, as well as your capacity to identify potential risks and opportunities for your clients. Demonstrating your knowledge in these areas shows that you can provide valuable insights and guidance to clients engaging in international deals.

Example: “When advising clients on cross-border transactions, it’s essential to consider the regulatory environment and potential legal hurdles in both countries involved. This includes understanding local laws, tax implications, and any restrictions or requirements for foreign investments. It is also important to be aware of currency fluctuations and their impact on the transaction value.

Another key consideration is cultural differences between the parties involved. Understanding the business practices, negotiation styles, and communication preferences of each party can help facilitate a smoother transaction process. Additionally, having a strong network of local advisors and experts in the target market can provide valuable insights and support throughout the deal-making process. Ultimately, being well-prepared and knowledgeable about these factors will enable us to provide sound advice and help our clients navigate the complexities of cross-border transactions successfully.”

30. Why did you choose to pursue a career in investment banking, and what motivates you to excel in this field?

Interviewers want to gauge your passion for and understanding of the investment banking industry. They are looking for candidates who are driven not only by financial rewards, but also by the intellectual challenges and the opportunity to make a significant impact on clients and the markets. Demonstrating your motivation to excel in this field helps interviewers assess your long-term commitment and potential for success within their organization.

Example: “I chose to pursue a career in investment banking because I am passionate about the intersection of finance and strategic decision-making. The dynamic nature of this field, with its ever-changing market conditions and complex financial instruments, presents an intellectually stimulating environment that constantly challenges me to learn and grow.

What motivates me to excel in investment banking is the opportunity to make a tangible impact on businesses and economies. As an investment banker, I have the privilege of working closely with clients to help them achieve their goals, whether it’s raising capital for expansion or advising on mergers and acquisitions. This responsibility drives me to continuously hone my skills and stay updated on industry trends so that I can provide the best possible advice and support to my clients. Ultimately, knowing that my work contributes to the success of companies and has a broader economic impact is what keeps me motivated and committed to excellence in this field.”

30 Waitress Interview Questions and Answers

30 loan officer interview questions and answers, you may also be interested in..., 20 must-know sports nutritionist interview questions (with answers), 30 center manager interview questions and answers, 20 operations research analyst interview questions and answers, 30 book editor interview questions and answers.

Top investment banking interview questions with answers

investment banking interview questions

Do you want to start a career in investment banking? Preparing for an investment banking interview can significantly enhance the chances of getting the job. In this regard, below, we outline the most common investment banking interview questions a future investment banker or associate can expect and provide answers to them.

Common types of investment banking interview questions

Interviewers have a variety of questions to ask in an investment banking interview. Naturally, you can’t be sure which ones of the investment banking interview questions and answers will be voiced during the interview.

However, based on extensive research, we provide you with the most common questions and answers first-year students in finance go through during investment banking interviews.

We will divide them into technical and additional questions for better understanding.

It would be a great idea for the candidate to prepare an investment banking interview questions and answers PDF file when getting ready for the interview.

Main technical questions to ask during an investment banking interview

The core of any M&A investment banking interview is to assess the candidate’s expertise in the field with a series of technical questions. Those can be split up into several subcategories.

Accounting questions

  • What are the three types of financial statements?
  • What is working capital, and how is it calculated?
  • Why is it better for a company to issue debt instead of equity?

The list of accounting questions investment banking professionals might ask freshmen during the interview is big. Below, we outline only a few of the most common ones.

1. What are the three types of financial statements?

There are three main financial statement types — income statement, balance sheet, and cash flow statement.

  • The income statement displays the company’s revenues and expenses over a while and ends with net income.
  • The balance sheet illustrates information about the company’s assets and liabilities — cash, inventory, property, and equipment, as well as shareholder’s debt, equity, and accounts payable.
  • The cash flow statement gives the company’s net change in cash. It begins with net income and then shows the company’s cash flows from financing, investing, and operating activities.

2. What is working capital, and how is it calculated?

Working capital is the difference between a company’s current assets (cash, inventories, finished goods, customers’ unpaid bills) and current liabilities (debts and accounts payable). 

Working capital can be positive or negative. 

Positive working capital signals that a company can invest in future growth and activities. 

The company has a negative working capital when its ratio of current assets and liabilities is less than one.

The formula for calculating working capital:

Working capital = Current assets – Current liabilities

3. Why is it better for a company to issue debt instead of equity?

There are a few reasons why issuing debt instead of equity would be preferable:

  • This is a cheaper and less risky way of financing.
  • The company can benefit from tax shields if it has tax-deductible income.
  • Issuing debt instead of equity is profitable if the company has immediately consistent cash flows and can make its interest payments.
  • It often might result in a lower weighted cost of capital.

Enterprise or equity value questions

  • Why do we look at enterprise value and equity value?
  • How to calculate the cost of equity?
  • How to calculate enterprise value?
  • When considering a company’s acquisition what requires more attention from your side — enterprise value or equity value?

Usually, questions on enterprise value and equity value are straightforward. To answer those questions, it’s important to know the theory as well as all the essential formulas.

1. Why do we look at enterprise value and equity value?

Enterprise value and equity value are two common ways to evaluate the business, but each of them shows a slightly different view. 

Equity value shows a snapshot of current and potential future value, while enterprise value provides an accurate calculation of the overall current value of the business. 

Thus, equity value is the figure the public-at-large sees, while enterprise value is the true value of the company. 

It’s important to look at both enterprise value and equity value to get a broader perspective of the company’s potential after the merger or acquisition and define the selling cost more accurately.

2. How to calculate the cost of equity?

To calculate the cost of equity, investment bankers usually use the capital asset pricing model (CAPM):

CAPM = Risk-free rate + Beta * (Expected market return – Risk-free rate)

3. How to calculate enterprise value?

There’s a formula for the enterprise value calculation:

Enterprise value (EV) = Market value of equity + Debt + Minority interest + Preferred stock – Cash

4. When considering a company’s acquisition, what requires more attention from your side: Enterprise value or equity value?

Enterprise value. This is the number an acquirer needs to pay, and it often includes debt repayment.

Valuation questions

  • Name the three main valuation methodologies.
  • What are the other valuation methods?
  • Why would you use multiple valuations of a single company?

Those who want to build a career in investment banks need to demonstrate above-average knowledge in valuation. Below are only the most common investment banking internship interview questions on valuation.

1. Name the three main valuation methodologies

These include comparable companies, precedent transactions, and discounted cash flow analysis.

2. What are the other valuation methods?

The most popular are liquidation valuation, replacement value, leveraged buyout analysis, the sum of the parts, future share price analysis, and M&A premium analysis.

3. Why would you use multiple valuations of a single company?

Each valuation method is based on different assumptions and will yield different values.

Usually, the precedent transaction and discounted cash flow method demonstrate higher valuations than the comparable companies method does. 

Discounted cash flow (DCF) questions

  • What is a discounted cash flow?
  • What is a weighted average cost of capital, and how do you calculate it?
  • How to calculate unlevered free cash flows for the DCF analysis?

DCF questions during the Investment banking recruiting process usually are not limited to the basis of constructing a DCF model. Candidates should also understand the cost of equity, WACC, terminal value, and other concepts. 

Below are only a few most common DCF questions you should expect during an investment banking interview.

1. What is a discounted cash flow? 

Discounted cash flow, or DCF, is a valuation method used to assess the profitability of the potential investment opportunity. It is based on the present value of the company’s cash flows and the present value of its terminal value. With this approach, you discount the value of future cash flows.

2. What is a weighted average cost of capital, and how do you calculate it?

The weighted average cost of capital (WACC) demonstrates the company’s overall cost of raising capital. It also represents the risk of the potential investment in a target company.

To calculate WACC, investment banking experts use this formula:

Cost of Equity * (% Equity) + Cost of Debt * (% Debt) * (1 – Tax Rate) + Cost of Preferred * (% Preferred)

3. How to calculate unlevered free cash flows for the DCF analysis?

For this, investment bankers use a dedicated formula:

Free cash flows = Operating profit * (1 – tax rate) + Depreciation and Amortization – changes in the net working capital – capital expenditures

Merger model questions

  • What’s the difference between a merger and an acquisition?
  • When is the acquisition considered dilutive?
  • What are synergies and their main types?

For entering the corporate finance industry, prospective investment bankers and associates should understand the concept of mergers and acquisitions. However, you don’t need to understand it at the M&A banker level.

1. What’s the difference between a merger and an acquisition?

These two types of deals differ in the size of the buying and selling sides. 

A merger generally happens between two companies that are almost of the same size, and together they form a new joint entity. 

Acquisition usually refers to the deal where the buying company is substantially bigger than the target (selling) company, and it’s always about a takeover of one entity by another.

Note: Learn more about the key difference between merger and acquisition in our dedicated article.

2. When is the acquisition considered dilutive? 

The deal is dilutive when the acquiring company’s earnings per share (EPS) decrease after the deal’s closure. 

3. What are synergies and their main types?

Synergies in mergers and acquisitions happen when an acquiring company gets more value out of the deal than it was predicted.

There are two types of synergies:

  • Revenue synergies

This type of synergy happens when a combined company gets an opportunity to sell products to new customers or sell new products to current customers, and, as a result, the revenue increases.

  • Cost synergies

Cost synergy occurs when a combined company can consolidate property, lay off certain employees, or shut down physical stores in some locations and, as a result, saves cash, which results in increased revenue.

Leveraged buyout (LBO) model questions

  • What is a leveraged buyout?
  • What variables have the most impact on an LBO model?
  • What is the tax shield in an LBO?

As with any category of investment banking interview questions, the candidate should not only demonstrate the knowledge of basic LBO concepts but also prove he understands how different variables can influence the output.

Below are the top three most commonly asked LBO questions during the investment banking interview.

1. What is a leveraged buyout?

A leveraged buyout is an acquisition of another company with the help of borrowed money to meet the deal’s cost. As a rule, LBO comes with a ratio of 90% debt to 10% equity.

2. What variables have the most impact on an LBO model?

Purchase and exit multiples affect the returns of a model the most. After this, the amount of used debt, revenue growth, and EBITDA margins have a substantial impact as well.

3. What is the tax shield in an LBO?

A tax shield in LBO takes place when the interest expense a company pays on debt is taxable, which helps this company to save money on taxes and, as a result, increase its cash flows.

Additional questions

Among the additional investment banking interview questions are usually those that are less connected to the theoretical and technical part that entails such aspects as preparing a company for an IPO . Such questions are often regarded as behavioral or fit.

Behavioral questions

  • Why investment banking?
  • What qualities should investment bankers have?

The main goal of behavioral questions is to clarify the candidate’s soft skills and personality traits.

Below are only a couple of examples of fit questions you can hear during investment banking interviews.

1. Why investment banking?

The key recommendation here is to tailor your answer to the particular company you’re interviewed for. 

The answer should include your previous professional investment banking experience (if any) and a logical connection to the position you’re applying for.

2. What qualities should investment bankers have?

Before naming a list of qualities a person should have to work in an investment bank, make sure you possess at least 80% of them:

  • Strong communication skills
  • Attention to detail
  • Time management skills
  • Analytical mindset
  • Ability and willingness to multitask
  • Ability to meet deadlines of multiple tasks
  • Determination
  • Strong work ethic

Note: To learn more about investment banking pitchbook read our dedicated article.

The possible investment banking interview questions a candidate can hear during investment banking interviews are numerous. They also depend on the position and the firm they are applying for. 

However, there are some basic questions almost all interviewers ask. They can be branched out into technical and additional ones.

  • Technical questions aim to check the theoretical knowledge of a candidate. This category usually includes questions on accounting, enterprise/equity value, valuation, discounted cash flow, merger model, and leveraged buyout. 
  • Additional questions target the candidate’s soft skills. They aim to determine how applicants would fit the team and whether they share the same values. Such questions are often regarded as behavioral or fit questions.

Guide on going public

One of the primary roles of investment bankers is to advise and intermediate Initial Public Offers (IPOs). 

With this in mind, the M&A Community, in collaboration with iDeals, produced the IPO consideration stage whitepaper, supporting professionals in better understanding the key considerations around going public.

Download the whitepaper here . 

Other insights

investment banking case study interview questions

What are SPAC warrants: Key points and considerations

investment banking case study interview questions

How to Prepare for UBS Investment Banking Interviews

If you're looking to land a job in UBS investment banking, you'll need to prepare thoroughly for the interview process.

Posted May 11, 2023

investment banking case study interview questions

If you have an upcoming UBS investment banking interview, congratulations! You've made it through the initial screening process and have caught the attention of one of the world's most prestigious financial institutions. But before you can land the job, you need to ace the interview. Here are some tips on how to prepare for a UBS investment banking interview:

Understanding the UBS Investment Banking Interview Process

Before diving into interview preparation, it's important to understand the UBS investment banking interview process. Typically, you'll go through several rounds of interviews, including a phone or video interview, a one-on-one interview with a hiring manager, and potentially a case study or group presentation. Each round will test different skills and attributes, so it's important to prepare for each one individually.

It's also important to note that UBS values diversity and inclusion in their hiring process. They actively seek out candidates from a variety of backgrounds and experiences, as they believe this leads to a more innovative and successful team. During the interview process, you may be asked about your experiences working with diverse teams or how you would approach working with colleagues from different backgrounds.

Researching UBS Investment Banking Culture and Values

Just like any other job, you want to make sure that UBS is a good fit for you. Research the company's values and culture to ensure that they align with your own. This will not only help you answer questions in the interview but also show your genuine interest in working there.

One way to research UBS's culture and values is to look at their mission statement and company website. This can give you insight into their overall goals and priorities. Additionally, you can reach out to current or former employees to get a better understanding of what it's like to work at UBS. This can be done through networking events, LinkedIn, or even reaching out to people in your personal network who may have connections at the company.

Crafting Your Personal Narrative for UBS Investment Banking Interviews

Your personal narrative should highlight your experience, skills, and accomplishments while demonstrating why you're the perfect fit for the job. Tailor your narrative to UBS's values and the specific role you're applying for. Practice it until you feel confident and comfortable speaking about yourself and your achievements.

When crafting your personal narrative, it's important to remember that UBS is looking for candidates who can bring a unique perspective and approach to their work. Consider highlighting any experiences or skills that set you apart from other candidates and demonstrate your ability to think creatively and solve complex problems.

Additionally, don't be afraid to show your personality in your personal narrative. UBS values candidates who are not only skilled and experienced, but also personable and easy to work with. Incorporating anecdotes or personal stories into your narrative can help showcase your personality and make you more memorable to interviewers.

Building a Strong UBS Investment Banking Resume and Cover Letter

Your resume and cover letter are often your first chances to make a good impression on a recruiter. Make sure they are polished, concise, and highlight your most relevant experiences and skills. Use specific data points to demonstrate your impact and accomplishments in previous roles.

When crafting your resume and cover letter for UBS Investment Banking, it's important to tailor your application to the specific role you are applying for. Research the job description and company culture to ensure that your application aligns with their values and requirements.

In addition to highlighting your experiences and skills, consider including any relevant extracurricular activities or volunteer work that demonstrate your leadership abilities and commitment to the industry. This can help set you apart from other applicants and showcase your well-roundedness.

Developing Your Technical Skills for UBS Investment Banking Interviews

Investment banking requires a strong understanding of finance and accounting. Brush up on technical skills like financial modeling, valuation, and accounting principles. There are many online resources available to help you practice and improve your skills.

In addition to finance and accounting, it is also important to have a solid understanding of the industry and market trends. Stay up-to-date with the latest news and developments in the investment banking industry by reading industry publications and attending conferences and events.

Another important aspect of developing your technical skills is to practice your communication and presentation skills. Investment bankers often need to present complex financial information to clients and colleagues in a clear and concise manner. Consider taking public speaking courses or practicing your presentation skills with friends or colleagues.

Practicing Common UBS Investment Banking Interview Questions

Practice answering common interview questions, both behavioral and technical. Know how to answer questions about your strengths, weaknesses, and leadership style. Prepare for technical questions related to valuations, financial statements, and investment strategies.

It is also important to research the company and the specific role you are interviewing for. Understand the company's values, mission, and recent projects. This will show your interest and dedication to the position. Additionally, be prepared to ask thoughtful questions about the company and the role during the interview.

Another helpful tip is to practice with a friend or mentor. Have them ask you common interview questions and provide feedback on your answers. This will help you feel more confident and prepared for the actual interview.

Preparing for Behavioral and Situational UBS Investment Banking Interview Questions

Behavioral and situational questions are designed to assess your problem-solving and critical-thinking skills. Prepare to answer questions about conflicts, challenges, or difficult scenarios you've faced in previous roles. Be sure to demonstrate how you overcame these challenges and what you learned from the experience.

Another important aspect to consider when preparing for behavioral and situational UBS investment banking interview questions is to research the company and its values. This will help you understand the type of culture and work environment you may be entering, and allow you to tailor your responses accordingly. Additionally, it's important to practice your responses to common behavioral and situational questions with a friend or mentor, so that you can refine your answers and feel more confident during the interview.

Finally, it's important to remember that behavioral and situational questions are not meant to trick you or catch you off guard. Rather, they are designed to give the interviewer insight into your thought process and problem-solving abilities. So, be honest and authentic in your responses, and don't be afraid to ask for clarification if you need it. With the right preparation and mindset, you can ace your UBS investment banking interview and land your dream job.

Navigating the Case Study Portion of the UBS Investment Banking Interview

During a UBS investment banking interview, you may be given a case study to solve. Remember to keep your approach structured and highlight your ability to solve complex problems. Make sure to show your strong financial modeling and strategic thinking skills.

When solving the case study, it is important to communicate your thought process clearly and concisely. This will demonstrate your ability to work well under pressure and effectively communicate with team members and clients. Additionally, be prepared to defend your assumptions and conclusions with data and analysis.

It is also important to remember that the case study is not just about finding the right answer, but also about demonstrating your ability to think creatively and outside the box. Don't be afraid to propose unconventional solutions or approaches, as long as they are backed up by sound reasoning and analysis.

Tips for Following Up After Your UBS Investment Banking Interview

After the interview, follow up with a polite thank-you email. Use this opportunity to reiterate your interest in the position and highlight any notable takeaways from the interview. This will leave a positive impression on the interviewer and show your professionalism and enthusiasm for the job.

In addition to sending a thank-you email, consider sending a handwritten note to the interviewer. This personal touch can make a lasting impression and show that you are willing to go above and beyond to stand out from other candidates.

It's also important to continue networking with UBS employees and attending industry events. This demonstrates your commitment to the field and can lead to future job opportunities or connections. Keep in touch with your interviewer and express your interest in any future openings that may arise.

Common Mistakes to Avoid During Your UBS Investment Banking Interview

Avoid common interview mistakes such as being late, dressing inappropriately, or failing to research the company beforehand. Make sure to focus on your key strengths and achievements, and avoid rambling or being overly negative about previous experiences. Always remember to stay professional and positive throughout the interview.

Additional Resources for Preparing for UBS Investment Banking Interviews

There are many books, online courses, and resources available to help you prepare for UBS investment banking interviews. Utilize these resources to hone your skills and techniques and ensure you're fully prepared for the interview process.

By following these tips and thoroughly preparing for each stage of the UBS investment banking interview process, you'll increase your chances of landing the job. Remember to stay confident, professional, and passionate about your goals and experience.

One great resource for preparing for UBS investment banking interviews is to network with current or former UBS employees. They can provide valuable insights into the interview process and what the company is looking for in candidates. Additionally, attending industry events and conferences can help you stay up-to-date on the latest trends and developments in investment banking.

Another helpful tip is to practice your interview skills with a friend or mentor. This can help you feel more comfortable and confident during the actual interview. Additionally, consider recording yourself during practice interviews to identify areas where you can improve your communication and presentation skills.

Browse hundreds of expert coaches

Leland coaches have helped thousands of people achieve their goals. A dedicated mentor can make all the difference.

Browse Related Articles

investment banking case study interview questions

May 11, 2023

How to Prepare for J.P. Morgan Investment Banking Interviews

If you're preparing for J.P. Morgan investment banking interviews, this article is a must-read.

investment banking case study interview questions

June 8, 2023

How to Prepare for Credit Suisse Investment Banking Interviews

If you're looking to land a job in investment banking, you'll need to be prepared for the rigorous interview process at Credit Suisse.

investment banking case study interview questions

January 10, 2024

How to Prepare for Morgan Stanley Investment Banking Interviews

Are you preparing for a Morgan Stanley investment banking interview? Look no further! Our comprehensive guide will provide you with the tips and strategies you need to ace your interview and land your dream job.

investment banking case study interview questions

How to Prepare for Deutsche Bank Investment Banking Interviews

If you're preparing for an investment banking interview with Deutsche Bank, this article is a must-read.

investment banking case study interview questions

How to Prepare for Barclays Investment Banking Interviews

Are you gearing up for Barclays investment banking interviews? Look no further! Our comprehensive guide covers everything you need to know to ace the interview process.

investment banking case study interview questions

How to Prepare for Bank of America Investment Banking Interviews

If you're aspiring to work in investment banking at Bank of America, this article is a must-read! Learn how to prepare for your interviews with tips and insights from industry experts.

How to Prepare for HSBC Investment Banking Interviews

Are you aspiring to become an investment banker at HSBC? Our comprehensive guide on how to prepare for HSBC investment banking interviews will equip you with the necessary skills and knowledge to ace your interview.

investment banking case study interview questions

Leveraging Case Studies for Investment Banking Interview Preparation

This article explores the importance of case studies in investment banking interviews and provides tips on how to effectively leverage them for preparation.

investment banking case study interview questions

December 4, 2023

How to Prepare for Goldman Sachs Investment Banking Interviews

Looking to land a job in investment banking at Goldman Sachs? Our comprehensive guide on how to prepare for Goldman Sachs investment banking interviews will give you the insider tips and strategies you need to ace your interview and stand out from the competition.

investment banking case study interview questions

How to Prepare for Lazard Investment Banking Interviews

If you're looking to land a job in investment banking, you'll want to read this article on how to prepare for Lazard Investment Banking interviews.

investment banking case study interview questions

How to Prepare for Citigroup Investment Banking Interviews

Are you looking to land a job in investment banking at Citigroup? Our comprehensive guide on how to prepare for Citigroup investment banking interviews will give you the edge you need to succeed.

investment banking case study interview questions

May 16, 2023

How to Prepare for Redfin Product Management Behavioral Interviews

Are you preparing for a Redfin Product Management behavioral interview? Our article provides valuable tips and insights to help you ace the interview process.

IMAGES

  1. How to Succeed at Investment Banking Case Study Interviews Free eGuide

    investment banking case study interview questions

  2. Top 28 Investment Banking Interview Questions & Answers

    investment banking case study interview questions

  3. Investment Banking Analyst Interview (2021) Questions and Answers

    investment banking case study interview questions

  4. Investment Banking Interview Questions

    investment banking case study interview questions

  5. Investment Banking Interview Questions & Answers

    investment banking case study interview questions

  6. Investment Banking Interview Questions and Answers for software testing

    investment banking case study interview questions

VIDEO

  1. Banking case study / Project Explanation

  2. How to Ace the Stryker Case Study Interview Questions

  3. Investment Banking Mock Interview

  4. Investment Banking case studies

  5. Leverage 401k For Velocity Banking: Case Study on HOW TO DO IT

  6. Unlocking Financial Freedom through Velocity Banking Strategies- Full Video Linked

COMMENTS

  1. Investment Banking Case Studies

    Case studies are commonly used in these interviews to assess a candidate's ability to apply financial and analytical skills to real-world situations. In addition, they test the candidate's critical thinking, problem-solving, and presentation abilities, essential skills for investment banking analysts and associates.

  2. Investment Banking Interview Questions and Answers

    101 common technical, fit, behavioral, and logic investment banking interview assessments with sample answers. IB Interview First Impressions - Carrying Yourself Common First Investment Banking Interview Questions: Crafting Your Story 30 Common Investment Banking Technical Questions WSO's Bonu

  3. Investment Banking Case Study Interviews

    The case study questions are not necessarily about finding a correct answer, but rather the thought process and analytical skills used to tackle a problem. What to Expect During a Case Study Interview. Case studies for investment banking interviews come in two forms: Take-home case studies; Blind or on-the-spot case studies; Take-Home Case Studies

  4. The Complete Guide to Investment Banking Interview Questions

    The most common case study you'll see in an investment banking interview process would be a very light DCF model with little complexity. This is something you might only see at a very small boutique firm, and won't see at all at larger firms or bulge-bracket banks - they simply don't have the time or capacity to review case studies on a ...

  5. The 400 Investment Banking Interview Questions

    Understanding the Investment Banking Interview Process. The investment banking interview process typically consists of multiple rounds, including phone screenings, technical interviews, behavioral interviews, and case study interviews. Each round aims to evaluate different aspects of a candidate's abilities and suitability for the role.

  6. Investment Banking Interviews

    Common Investment Banking Interview Questions. There are four main categories of interview questions in investment banking interviews, and you must be able to answer each one confidently. They are: Category 1: Telling Your Story. Category 2: "Fit" Questions. Category 3: Deals, Markets, and Companies. Category 4: Technical Questions and Answers.

  7. Essential Reading for Investment Banking Interviews

    The Investment Banking Interview Guide ("The Red Book") 1,000 interview questions & answers. Brought to you by the company that works directly with the world's top investment banks and PE firms. Understand what to expect in an investment banking interview including the format, qualitative and technical questions, and networking.

  8. Investment Banking Interview Guide 4.0

    The Investment Banking Interview Guide from BIWS is the World's Most Popular interview preparation tool for aspiring investment bankers and finance professionals. ... the 218+ practice quiz questions, the 17 case studies, the video tutorials, and more. And you'll have access to our expert support team to ask whatever questions you need, for a ...

  9. Investment Banking Interview Questions & Answers

    Investment Banking Interview Questions Category 4: Technical Questions and Answers. For this last category, I do not have any magical tips that will get you results in hours instead of weeks or months. Put simply, to succeed in investment banking interviews, you need to put in the time to study accounting, finance, valuation, and M&A and LBO ...

  10. 25 Investment Banking Interview Questions (With Answers)

    Introductory Interview Questions. Investment banking interviews begin the same way as any other interview: getting to know one another. The interviewer will likely give you an overview of the company and the position, and ask you some questions about yourself. 1. Walk me through your resume.

  11. Investment Banking Case Studies

    Investment banking case studies are an important element in the interview process, it is an opportunity to showcase your skills and talent to investment bankers. In general, there are two types of case studies, the decision-making case study and the financial modeling case study. Candidates will need to be confident in their valuation skills.

  12. Investment Banking Case Study Examples

    Evaluating Strategic Alternative: Case Study 1. Example: To maximize shareholder value, a magazine publisher is deciding whether to sell, grow organically or make tiny "tuck-in" acquisitions. It is looking for an investment bank to assist it with its alternatives and has asked for a presentation from your company.

  13. Top Investment Banking Interview Questions (2024)

    Companies with negative profits and EBITDA will have meaningless EBITDA multiples. As a result, Revenue multiples are more insightful. Q. Two companies are identical in earnings, growth prospects, leverage, returns on capital, and risk. Company A is trading at a 15 P/E multiple, while Company B trades at 10 P/E.

  14. Leveraging Case Studies for Investment Banking Interview Preparation

    Case studies provide a glimpse into how you handle stress and manage your time effectively. Additionally, they give interviewers a chance to evaluate your teamwork and collaboration skills, as you may be required to work with other candidates to solve the case. Overall, case studies are an essential component of investment banking interviews ...

  15. My Investment Banking Interview Questions and Answers: Case Study

    Investment Banking Interview Questions and Answers: Case Study. Interviewer: Hi Stephanie, I am Derek. Nice to meet you. Stephanie: Hi Derek, thanks for coming over to Chicago to interview us. Interviewer: Sure, no problem. I'd like to get out of Manhattan once in a while. I am from the Midwest myself (smile).

  16. 66 Investment Banking Interview Questions [Actually Asked ...

    100% Placement Assistance | 50+ Case Studies | Most extensive curriculum . View Course. Common Market-Related Investment Banking Interview Questions. Market-related questions probe your understanding of the broader financial landscape and your ability to form opinions based on current events. Staying abreast of the news and being able to ...

  17. Breaking Into Investment Banking: Top 25 Questions Asked in Interviews

    Common interview questions in investment banking include why you are interested in the job, why you are a good fit for the role, and why the bank should hire you. ... Case Study Analysis: How to Approach Various Scenarios Presented During Interviews. Investment banking interviews often include case study questions to assess your problem-solving ...

  18. 33 Investment Banking Interview Questions (With Answers)

    3 investment banking interview questions with sample answers Studying potential questions and answers before your interview increases your ability to provide valuable and concise responses to the hiring manager. Here are a few examples of questions and sample answers to reference for an upcoming investment banking interview: 1.

  19. The Ultimate Guide: Top 50 Questions Asked in Investment Bank

    Get ready to ace your investment bank interviews with our comprehensive guide to the top 50 questions asked. Investment bank interviews are notoriously challenging, and it can be difficult to know what to expect. In this comprehensive guide, we will provide you with all the information you need to ace your next interview.

  20. 30 Investment Banking Interview Questions and Answers

    4. Describe your experience with financial modeling. Financial modeling is the backbone of investment banking. It's the process of creating abstract representations of real-world financial situations in order to make informed decisions about investments, acquisitions, mergers, and other transactions.

  21. Investment Banking Interview Questions with Answers

    Below are only the most common investment banking internship interview questions on valuation. 1. Name the three main valuation methodologies. These include comparable companies, precedent transactions, and discounted cash flow analysis. 2.

  22. How to Prepare for Goldman Sachs Investment Banking Interviews

    Avoid coming across as boastful or negative and refrain from discussing inappropriate or irrelevant topics during the interview. Additionally, don't put all your eggs in one basket; interviews with Goldman Sachs are extremely competitive, so it's crucial to apply to other firms, too, and prepare for multiple interviews.

  23. How to Prepare for UBS Investment Banking Interviews

    Navigating the Case Study Portion of the UBS Investment Banking Interview. During a UBS investment banking interview, you may be given a case study to solve. Remember to keep your approach structured and highlight your ability to solve complex problems. Make sure to show your strong financial modeling and strategic thinking skills. When solving ...